Cardiovascular QBANK (UWORLD)

Réussis tes devoirs et examens dès maintenant avec Quizwiz!

-Age: 28 years professional endurance athlete -Comes to the office after his team physician detected a heart murmur. The patient has no symptoms and has no exercise limitations. Medical history is nonsignificant. -BP (110/66), pulse (54). Physical examination shows a soft systolic murmur best heard at the left 2nd intercostal space in the supine position. The murmur disappears when the patient is upright but reappears after brief exercise. Which of the following cardiovascular adaptations best explains the patient's finding? a)Decreased pulmonary vascular capacitance b)Decreased vagal tone c)Increased stroke volume d)Increased systemic vascular resistance e)Increased ventricular wall thickness

Answer: Choice C (Increased stroke volume) -This professional athlete most likely has the cardiovascular adaptations of athlete's heart. The soft systolic murmur likely represents a pulmonic flow murmur, a common finding due to the high stroke volume that develops with intensive endurance training -The adaptations of athlete's heart differ depending on whether intensive endurance training (e.g., long-distance running) or resistance training (e.g., heavy weightlifting) is pursued. With endurance training, systemic vascular resistance (SVR) is reduced (due to vasodilation in skeletal muscle) and blood volume is increased, leading to increased LV volume load. To compensate, LV eccentric hypertrophy (i.e., LV wall lengthening) develops, which increases LV cavity size to increase diastolic filling capacity and facilitate increased stroke volume and cardiac output. RV cavity enlargement also takes place to increase diastolic filling and accommodate the increase in cardiac output; because the RV and LV are arranged in series and contract at the same rate, RV stroke volume must increase to match LV stroke volume and prevent backup of blood flow -An increase in stroke volume is maintained at rest and allows for adequate cardiac output at low resting heart rate (e.g., <55/min). The anterior location of the pulmonic valve and slightly smaller valve area compared to the aortic valve often creates a pulmonic flow murmur due to the high stroke volume. The murmur is best heard at the left 2nd intercostal space in the supine position and may disappear when venous return is decreased with standing Choice A= Pulmonary vascular capacitance is increased with endurance training, allowing the pulmonary circulation to accommodate high flow while minimizing the afterload on the RV Choice B= Baseline vagal tone is increased with endurance training, facilitating low resting HR Choices D and E= Intensive resistance training increases SVR during exercise (due to vascular compression by vigorously contracting muscles), which increases LV pressure load and leads to mild concentric hypertrophy (i.e., LV wall thickening). Unlike pathologic concentric LV hypertrophy, the LV cavity remains normal in size and diastolic function is maintained. LV stroke volume and ejection fraction remain normal or are slightly increased, and the RV is unaffected. A flow murmur is not expected

-Age: 69 years man -Brought to the ED with sudden onset palpitations and dyspnea. Medical history: significant for hypertension and GERD. ECG reveals a HR of 120/min (irregular irregular), narrow QRS complexes, and no organized P waves. Which of the following physiological factors most likely determines the ventricular contraction rate in this patient? a)Atrial muscle depolarization rate b)Atrioventricular node refractory period c)Bundle branch conductivity d)Purkinje system pacemaker activity e)Sinoatrial discharge rate f)Ventricular muscle refractory period

Answer: Choice B (AV node refractory period) -This patient's ECG findings are diagnostic for atrial fibrillation with rapid ventricular response. Atrial fibrillation is the most common cardiac arrhythmia and is characterized by absent P waves, irregularly irregular R-R intervals, and narrow QRS complexes. Rapid, irregular, low amplitude fibrillatory waves are also frequently seen and represent chaotic, continuous atrial depolarizations. -Atrial fibrillation is initiated by aberrant electrical impulses that arise within regions of heightened atrial excitability (most often the pulmonary veins). Once triggered, atrial fibrillation induces electrical remodeling of the atria with the development of shortened refractory periods and increased conductivity. This facilitates the creation and persistence of multiple ectopic foci and reentrant impulses within the atria, increasing the risk and chronicity of subsequent episodes (atrial fibrillation begets atrial fibrillation) -Ventricular response in atrial fibrillation is dependent on the transmission rate of abnormal atrial impulses through the AV node. Each time the AV node is excited, it enters a refractory period during which additional atrial impulses cannot be transmitted to the ventricles; consequently, the majority of atrial impulses never reach the ventricles (Choice A). The average ventricular rate in atrial fibrillation usually ranges between 90-270bpm. Because atrial excitation is chaotic, the ventricular rate is irregular with no set intervals between contraction. Choice C= Bundle branch conductivity determines the duration of the QRS complexes, which are widened when there is a bundle branch block. It does not affect the ventricular contraction rate. Choice D= The purkinje system can assume pacemaker activity in patients with severe bradycardia (<40bpm), but it would not generate impulses in atrial fibrillation since the ventricular rate is typically elevated Choice E= During atrial fibrillation, the atria are in a state of chaotic, continuous depolarization that is independent of sinoatrial activity Choice F= The ventricular muscle refractory period does not limit the ventricular contraction rate during AF (the ventricular heart rate can approach 300/min in ventricular tachycardia)

-Age: 45 years woman -Recently immigrated to the US is hospitalized with exertional dyspnea and fatigue. She has no significant past medical history and takes no medications. BP (110/80), HR (90, regular). After cardiopulmonary examination, the physician suspects mitral stenosis. Which of the following is the most useful measure for assessing the degree of mitral stenosis? a)A2-to-opening snap time interval b)Audible S3 c)Audible S4 d)Diastolic murmur intensity e)Presystolic accentuation of the murmur

Answer: Choice A (A2-to-opening snap time interval) -On auscultation, the best indicator of mitral stenosis (MS) severity is the length of time between S2 (specifically the A2 component, caused by aortic valve closure) and the opening snap (OS). The OS occurs due to abrupt tensing of the valve leaflets as the mitral valve reaches its maximum diameter during forceful opening -As MS worsens, left atrial pressures increase due to impaired movement of blood into the LV. Higher pressure causes the valve to open more forcefully; as a result, the A2-OS interval becomes shorter as left atrial pressure increases. The current standard for diagnosis and determination of MS severity is measurement of mean transvalvular pressure gradients via 2-D Doppler echocardiography Choices B and C= Left-sided S3 and/or S4 gallops are generally absent in MS, since left ventricular filling is normal or decreased. When MS is severe enough to produce pulmonary hypertension, patients may develop right-sided heart failure. As a result, a right-sided S3 and/or S4 can occur. However, these right-sided gallops arise only in severe, end-stage MS and are not useful for differentiating between mild and moderate MS Choice D= Diastolic rumble intensity in MS does not correlate well with stenosis severity. This is due to murmur intensity being dependent on both the transvalvular pressure gradient and the amount of blood flowing through the valve. In very severe stenosis, the diastolic murmur may be completely absent due to greatly reduced flow across the valve Choice E= Presystolic accentuation of the MS murmur occurs due to increased transvalvular flow associated with left atrial contraction. This accentuation is heard across a wide range of MS severity. More significant than the degree of presystolic accentuation is its presence or absence. When MS becomes severe enough to precipitate atrial fibrillation, presystolic accentuation of the MS murmur disappears

-Age: 58 years man -Comes to the ED due to headache. History: long-standing hypertension and medication nonadherence. BP on arrival (231/135); 20 mins after IV hydralazine is administered= BP (145/95). Over the following 24 hours, serum creatinine increases from 1.1 to 1.6mg/dl -Urine sediment demonstrates abundant granular casts. Which of the following processes best explains this patient's acute kidney injury? a)Alteration of the blood pressure-flow relationship in renal arterioles b)Drug-induced glomerular injury due to immune complex formation c)Drug-induced renal interstitial inflammation d)Drug-induced renal tubular injury due to crystal deposition e)Prerenal azotemia due to a global reduction in cardiac output

Answer: Choice A (Alteration of the blood pressure-flow relationship in renal arterioles) This patient with chronic hypertension has hypertensive urgency. He received hydralazine, a fast-acting antihypertensive agent. After abrupt normalization of BP, he developed acute tubular necrosis (ATN) (granular casts). This sequence of events is consistent with normotensive ischemia (i.e., inadequate renal perfusion despite a normal BP) Blood flow autoregulation allows organs to receive the same perfusion (flow) across a wide range of pressure. In the kidneys, blood flow autoregulation involves 2 key mechanisms: -Myogenic response: Afferent glomerular arterioles reflexively constrict when they sense greater stretch forces (high BP) -Tubuloglomerular feedback: High arterial pressure causes hyperfiltration (increased GFR), increasing sodium and chloride delivery to the macula densa, which secretes vasoactive mediators (e.g., adenosine) to constrict the afferent arterioles The autoregulation response protects organs from dangerous, persistent BP elevation. In chronic hypertension, there is a baseline constriction of afferent glomerular arterioles. This causes the entire renal autoregulation curve to be shifted to the right; a hypertensive patient's kidneys receive less blood flow at any given BP relative to those of a healthy patient without hypertension. Quickly lowering the BP to normal (e.g., 145/95) causes a steep drop in blood flow, leading to ischemic ATN; as a result, BP should be lowered gently, targeting above-normal values (e.g., 25% reduction over several hours) Choice B= Drug-induced lupus, a rare adverse effect of hydralazine, causes acute kidney injury (AKI) due to immune complex-mediated glomerulonephritis. However, an active urine sediment (e.g., red cells, white cells, casts) and other systemic signs of lupus (e.g., rash, pleuritis) would be expected Choice C= Drug-induced allergic interstitial nephritis is an eosinophilic AKI usually triggered by beta-lactam antibiotics and NSAIDs. It occurs after continual exposure (e.g., several days), and eosinophiluria is usually present Choice D= Some drugs (e.g., acyclovir, indinavir) can precipitate in the urine, causing AKI due to crystalluria. Urine sediment may show tubular necrosis (e.g., granular casts), but crystals are usually also seen. Choice E= Decreased cardiac output can cause prerenal AKI, which may evolve to ischemic ATN. However, treatment with hydralazine lowers the systemic vascular resistance, which reduces cardiac afterload. This will lead to an increase in overall cardiac output.

-Age: 64 years man -Dies suddenly while playing tennis. In the preceding months, the patient experienced fatigue and some exertional dyspnea. Autopsy= rupture of an unsuspected ascending aortic aneurysm. -Heart exam: septal thickness of 1.1cm (normal <1.1), posterior wall thickness of 1.1cm (normal <1.1), and an internal left ventricular diameter of 6.8cm (normal: 3.5-5.9). No focal myocardial scarring is seen. Which of the following is the most likely cause of the cardiac findings seen in this individual? a)Aortic regurgitation b)Aortic stenosis c)Hypertrophic cardiomyopathy d)Obstructive coronary artery disease e)Restrictive cardiomyopathy

Answer: Choice A (Aortic regurgitation) -This individual's autopsy reveals a dilated left ventricular (LV) cavity with relatively thin ventricular walls, a finding consistent with eccentric hypertrophy. Eccentric hypertrophy results from the addition of myocardial contractile fibers in series in response to chronic volume overload. The adaptation allows the LV to increase stroke volume and maintain cardiac output; however, LV wall stress increases, resulting in eventual decompensation and the development of heart failure. Common causes of eccentric hypertrophy include cardiomyopathy, ischemic heart disease, and chronic aortic or mitral valve regurgitation -In developed countries, aortic root dilation is one of the most common causes of chronic aortic regurgitation. This individual with an ascending aortic aneurysm likely had aortic root dilation with consequent chronic aortic regurgitation, leading to LV volume overload and eccentric hypertrophy Choice B= Aortic stenosis leads to pressure overload of the LV and the development of concentric hypertrophy (thickening of the LV walls via the addition of myocardial contractile fibers in parallel). However, the ventricular cavity size is typically normal or reduced (not increased) Choice C= Hypertrophic cardiomyopathy is an inherited cardiomyopathy that involves localized thickening of the LV septal wall and resulting LV outflow tract obstruction. Although sudden cardiac death can occur (mostly in young athletes), the uniform thickness of this patient's ventricular walls and enlarged chamber size is more consistent with eccentric hypertrophy due to aortic regurgitation Choice D= Obstructive coronary artery disease can lead to eccentric hypertrophy via myocardial contractile dysfunction and consequent LV volume overload (i.e., ischemic heart disease). However, chronic aortic regurgitation is more likely in this individual with ascending aortic aneurysm and no evidence of myocardial scarring (scarring would suggest previous MI) Choice E= Restrictive cardiomyopathy involves no change or a slight decrease, rather than an increase, in the LV cavity size.

-Age: 46 years man -Comes to the office for an annual examination. He has an uncomfortable heartbeat sensation at night that he tries to decrease by sleeping on his right side. -The patient has had mild shortness of breath with exertion over the last 6 months, but he has no chest pain. He was told during a wellness check approximately 2 years ago that he had a heart murmur. The patient has no other medical problems. Physical examination reveals bounding pulses and carotid pulsations that are accompanied by head bobbing. Which of the following is the most likely diagnosis? a)Aortic regurgitation b)Aortic stenosis c)Atrial septal defect d)Coarctation of the aorta e)Mitral regurgitation f)Mitral stenosis g)Pulmonary stenosis h)Tricuspid regurgitation

Answer: Choice A (Aortic regurgitation) -This presentation is most suggestive of aortic regurgitation. The inability of aortic valve leaflets to effectively close during diastole leads to regurgitation of blood back into the LV cavity with an increase in LV end-diastolic volume and wall stress. The resultant chamber enlargement and eccentric hypertrophy increase total stroke volume, which is often felt as a sense of pounding or an uncomfortable feeling of heartbeat (especially when lying on the left side). -Physical examination reveals an early "blowing" diastolic decrescendo murmur best heard at the left sternal border in the 3rd or 4th intercostal space. The precordial impulse is hyperdynamic and displaced laterally and downward. Bounding femoral and carotid pulses, marked by abrupt distention and quick collapse ("water-hammer" pulse), are the result of the wide pulse pressure. Some patients exhibit head bobbing with carotid pulsations (de Musset sign) due to transfer of momentum from the larger LV stroke volume to the head and neck. Significant systolic pulsations may also be noticed in other organs (e.g., liver, spleen, retina) and the fingertips. Choice B= Patients with severe aortic stenosis have a characteristic arterial pulse- small pulse amplitude (pulsus parvus) with a delayed peak and slower upstroke of the arterial pulse (pulsus tardus) due to diminished stroke volume and prolonged ejection time. Choice C= The presence of an ASD leads to left-to-right intracardiac shunting, which can cause a hyperdynamic right ventricular impulse. It does not cause any significant change in arterial pulse character Choice D= Coarctation of the aorta causes systolic hypertension in the upper extremities along with characteristic diminished and/or delayed femoral pulses (brachial-femoral delay) Choice E= Arterial pulse, pulse pressure, and forward stroke volume remain normal in patients with chronic (compensated) mitral regurgitation Choice F= Arterial pulses are reduced in volume and amplitude in patients with mitral stenosis due to decreased LV end-diastolic volume and stroke volume Choices G and H= Tricuspid regurgitation and/or pulmonary stenosis are right-sided valvular lesions and do not cause any specific arterial pulse pattern on physical examination

-Age: 64 years man -Dies suddenly while playing tennis; in the preceding months, he experienced fatigue, some exertional dyspnea. -Autopsy= left ventricular septal thickness of 1.6cm (normal <1.1), posterior wall thickness of 1.6cm (normal <1.1), and an internal left ventricular diameter of 3.2cm (normal; 3.5- 5.9) -No focal myocardial scarring is seen. Which of the following is the most likely cause of the cardiac findings seen in this individual? a)Aortic stenosis b)Dilated cardiomyopathy c)Mitral regurgitation d)Obstructive coronary artery disease e)Rheumatic mitral stenosis

Answer: Choice A (Aortic stenosis) -This individual's autopsy reveals thickened left ventricular walls and a small left ventricular cavity consistent with concentric hypertrophy -Concentric hypertrophy results from the addition of myocardial contractile fibers in parallel in response to chronic pressure overload. The adaptation allows for increased contractile force but causes impaired diastolic filling and can lead to heart failure with preserved ejection fraction (diastolic failure). Common causes of concentric hypertrophy include prolonged systemic hypertension and aortic stenosis -Patients with severe aortic stenosis typically experience exertional symptoms; including fatigue, shortness of breath, angina, and presyncope or syncope. Sudden cardiac death can result from the inability to increase cardiac output with exertion due to left ventricular outflow obstruction Choice B= Dilated cardiomyopathy results from a primary insult to cardiomyocytes (e.g., viral infection, chemical toxicity, inherited disease) that causes myocardial contractile dysfunction, left ventricular volume overload, and eventual eccentric hypertrophy Choice C= Severe mitral regurgitation causes LV volume overload and eccentric hypertrophy Choice D= Obstructive coronary artery disease can lead to eccentric hypertrophy via myocardial contractile dysfunction and consequent LV volume overload (i.e., ischemic heart disease). The development of concentric hypertrophy Choice E= Rheumatic mitral stenosis causes impaired diastolic filling of the LV. It does not increase the pressure or volume load on the LV and does not cause ventricular hypertrophy

-Age: 43 years man -Being evaluated for occasional retrosternal chest pressure that develops with moderate exertion and sometimes occurs when resting. -He doesn't use alcohol, tobacco, or illicit drugs. The patient has an extensive family history of coronary artery disease. Temp. (36.7C), BP (124/72), pulse (81), RR (14). Physical examination shows no abnormalities. -Coronary angiography shows mild luminal irregularities but no significant obstructive lesions. Acetylcholine infusion during the procedure results in dilation of epicardial coronary vessels. A reaction involving which of the following amino acids is most likely responsible for the observed dilation? a)Arginine b)Aspartate c)Glutamate d)Tryptophan e)Tyrosine

Answer: Choice A (Arginine) -The vascular endothelium plays an important role in the vasodilation mediated by acetylcholine, bradykinin, serotonin, substance P, and shear forces. These stimuli activate specific membrane receptors present on endothelial cells, leading to an increase in cytosolic calcium levels. This causes activation of endothelial nitric oxide synthase (eNOS), which synthesizes nitric oxide from arginine, NADPH and O2. -Nitric oxide then diffuses into the adjacent smooth muscle cells, where it activates guanylyl cyclase and increases formation of cyclic GMP. High levels of cyclic GMP activate protein kinase G, which causes a reduction in cytosolic calcium levels and relaxation of vascular smooth muscle cells -The availability of arginine for synthesizing nitric oxide depends on several factors, including exogenous food intake, endogenous synthesis, intracellular storage and degradation, and the presence of asymmetrical dimethylarginine (an endogenous analog of arginine that works as a competitive inhibitor of eNOS)

-Age: 67 years man -Brought to the ED due to progressive shortness of breath. He has had decreasing exercise tolerance for several months, and over the last week he could barely walk to the bathroom without becoming short of breath -The patient has been sleeping in a semirecumbent position recently. Medical history: type 2 diabetes, hyperlipidemia, and an MI 3 years ago -He takes no medications; the patient is an ex-smoker with a 30-pack-year history. BMI (30). Oxygen saturation= 92% on room air. The apical impulse is palpated in the 6th intercostal space along the anterior axillary line. -There is 3+ bilateral pitting edema in the legs. Which of the following is likely increased? a)Arteriolar resistance b)Lung compliance c)Mixed venous O2 content d)Renal perfusion e)Stroke volume

Answer: Choice A (Arteriolar resistance) This patient with progressive dyspnea, orthopnea, lower extremity swelling, and a laterally displaced point of maximal impulse has decompensated heart failure. Heart failure occur due to either a progressive decline in myocardial contractility (systolic dysfunction) or impaired ability of the ventricular walls to relax during filling (diastolic dysfunction). Both etiologies eventually lead to decreased cardiac output and inadequate oxygen delivery to the tissues, inducing the following neuroendocrine compensatory mechanisms in an effort to restore organ perfusion: -Increased sympathetic output: baroreceptors sense decreased perfusion and stimulate epinephrine and norepinephrine release to increase HR and contractility. Vasoconstriction is also triggered, increasing arteriolar resistance and raising afterload -RAAS activation: decreased renal perfusion (Choice D) leads to increased secretion of renin and increased circulating levels of angiotensin II, a potent vasoconstrictor that further increases afterload. Adrenal secretion of aldosterone also increases, which stimulates Na+ and water retention and increases circulating blood volume (preload) Although these compensatory mechanisms initially improve organ perfusion; they are overall maladaptive. A vicious cycle is generated as the failing heart is unable to pump against increased afterload, and volume retention creates venous congestion in the kidneys that reduces GFR and further impairs diuresis. Atrial and ventricular stretch triggers the release of natriuretic peptides that stimulate vasodilation and fluid diuresis to counteract the vicious cycle; however, these effects are outweighed by the downward-spiraling negative effects of sympathetic drive and RAAS Choice B= LV failure leads to increased pulmonary venous pressure and transudation of fluid into the lung parenchyma, which stiffens the alveolar walls and decreases lung compliance Choice C= With reduced cardiac output, the tissues become starved for O2 and increase O2 extraction from the available capillary blood. This leads to a reduction in mixed venous O2 Choice E= Stroke volume is generally maintained early in heart failure (due to compensatory mechanisms) and decreases as decompensation takes place

-Age: 62 years obese woman -Brought to the hospital due to acute-onset chest pain and shortness of breath. History: hyperlipidemia, diet-controlled type 2 diabetes, peripheral artery disease. -2 years ago, she had an ischemic stroke but had no residual neurologic deficits. The patient is a current smoker (30-pack-year history) -After initial evaluation, she is taken to the cardiac catheterization lab where she has a cardiac arrest due to ventricular arrhythmia and can't be resuscitated -Autopsy= thrombotic occlusion of the left anterior descending artery. In addition, it shows 95% atherosclerotic narrowing of the proximal right renal artery but no significant stenosis of the left renal artery. -Compared with the right kidney, this patient's left kidney is more likely to demonstrate which of the following? a)Arteriolar wall thickening b)Atrophic tubules c)Cortical thinning d)Interstitial fibrosis e)Small crowded glomeruli

Answer: Choice A (Arteriolar wall thickening) -This patient's autopsy findings are consistent with severe, unilateral renal artery stenosis (RAS), a condition that occurs most commonly in older individuals due to atherosclerosis. Hypoperfusion of the stenotic (right) kidney stimulates renin release by juxtaglomerular apparatus cells. Renin converts angiotensinogen to angiotensin I, which is then converted by angiotensin-converting enzyme (ACE) to angiotensin II, a potent vasoconstrictor that causes systemic hypertension -Unilateral RAS causes morphologic changes that markedly differ in the stenotic (right) kidney and the nonspecific (left) kidney. Narrowing of the renal artery in the stenotic (right) kidney leads to hypoperfusion and renal parenchymal ischemia, which manifests as diffuse cortical thinning, tubular atrophy, interstitial fibrosis, and small crowded glomeruli (Choices B, C, D and E). Juxtaglomerular apparatus enlargement can also occur due to chronic stimulation and increased renin release -In contrast, the contralateral, nonstenotic (left) kidney is exposed to high BP and therefore demonstrates typical signs of hypertensive nephrosclerosis, which is characterized by arteriolar wall thickening due to hyaline arteriolosclerosis (hyalinization of the arterioles due to extravasation of plasma proteins) and hyperplastic arteriolosclerosis (concentric smooth muscle cell proliferation in response to pressure; "onion-skinning")

-Age: 78 years woman -Comes to the ED due to a 3-day history of intermittent but progressive substernal chest pain. Medical history includes hypertension, hyperlipidemia, but the patient has had poor adherence to her medication and outpatient follow-up -She has a 30-pack-year smoking history. On admission, ECG reveals normal sinus rhythm with ST-segment elevation and pathologic Q waves in leads I, aVL, and V2 to V5. The patient refuses percutaneous coronary intervention and is treated medically. -On the fifth day of hospitalization, she experiences severe dyspnea, sweating and hypotension. Physical examination shows raised jugular venous pressure, no heart murmurs, and clear lungs. She rapidly loses consciousness and becomes pulseless, and the subsequent resuscitation attempt is ineffective. Which of the following most likely contributed to this patient's death? a)Blood accumulation in the pericardial space b)Complete rupture of the papillary muscle c)Rapid reperfusion of the ischemic myocardium d)Rupture of the interventricular septum e)True aneurysm of the ventricular wall

Answer: Choice A (Blood accumulation in the pericardial space) -This patient was initially admitted with chest pain and ST-segment elevation in the anterolateral ECG leads, findings consistent with an ST-elevation myocardial infarction (MI) of the LV. Several days later, her abrupt-onset dyspnea, and hypotension were most likely due to LV free wall rupture, a potential mechanical complication of transmural MI -LV free wall rupture usually occurs within 5 days but can occur up to 2 weeks following MI. During this time, the infarcted myocardial wall is substantially weakened by coagulative necrosis, leukocyte infiltration, and enzymatic lysis of myocardial connective tissue and may be unable to withstand the shear stress generated by the high-pressure LV. Delayed reperfusion (or absent reperfusion as in this patient), increases myocardial damage, further weakening the LV wall and increasing the risk of rupture -With LV rupture, blood abruptly fills the pericardial space, resulting in hemopericardium and acute cardiac tamponade with rapidly increased intrapericardial pressure. At onset, patients may experience dyspnea, diaphoresis, and sometimes chest discomfort. Diastolic filling of the relatively low-pressure right-sided heart chambers is abruptly restricted (causing jugular venous distention), and blood flow through the pulmonary circulation to the left side of the heart drops precipitously (the lungs remain clear). Cardiac output profoundly decreases, resulting in hypotension and obstructive shock that rapidly leads to cardiac arrest. Choices B and D= Papillary muscle rupture, causing acute mitral regurgitation, and interventricular septum rupture, causing a ventricular septal defect, can occur acutely or within 5 days of MI. Both defects typically cause a new holosystolic murmur and lead to LV failure with pulmonary edema. Cardiogenic shock with hypotension commonly develops, but lung crackles would be expected on examination Choice C= Ischemic reperfusion injury can occur when blood flow is abruptly restored to infarcted myocardium, and it may rarely lead to cardiac arrest (via precipitation of ventricular fibrillation or profound myocardial contractile dysfunction). However, it is extremely unlikely in this patient who received no reperfusion therapy Choice E= LV aneurysm formation is typically a delayed mechanical complication of MI, occurring after several weeks or months. Patients typically develop gradually progressive heart failure or angina.

-Age: 80 years man -Healthy, participated in a longitudinal study over the past 30 years to study the effects of aging on overall cardiovascular status. -He has undergone annual testing with echocardiography and his left ventricular ejection fraction has remain stable throughout the 30 years. Which of the following changes is most likely responsible for the preserved left ventricular ejection fraction in this patient? a)Cardiomyocyte hypertrophy b)Increased arterial reserve capacity c)Increased left ventricular compliance d)Increased left ventricular diastolic volume e)Stable ventricular myocyte number

Answer: Choice A (Cardiomyocyte hypertrophy) -The cardiovascular system undergoes an array of age-related physiologic changes. Over time, cardiomyocytes are lost to both apoptosis and necrosis (i.e., cardiomyocyte dropout), resulting in an overall reduced cardiomyocyte number. The loss of ventricular cardiomyocytes (Choice E) weakens contractile strength. In addition, the aorta and other large arteries stiffen with age, resulting in reduced arterial compliance (i.e., reduced arterial reserve capacity (Choice B) and increased LV afterload. To compensate for the cardiomyocyte loss and increased afterload, cardiomyocyte hypertrophy (i.e., mild concentric LV hypertrophy) occurs, allowing for maintenance of LV contractility and ejection fraction -Cardiomyocyte dropout also affects cardiac conduction cells, contributing to decreased maximal heart rate and slightly decreased resting heart rate. Decreased responsiveness to adrenergic stimuli with age also contributes to reduced maximal heart rate Choices C and D= The mild concentric LV hypertrophy that occurs with age results in reduction in LV compliance and LV end-diastolic volume. However, because these changes are relatively slight, stroke volume and cardiac output are mostly preserved.

-Age: 75 years man -Brought to the ED by paramedics following a high-speed motor vehicle accident. Several attempts are made to resuscitate him but are unsuccessful. The patient had no significant past medical history and took no medications. -He was known to be in good health and tolerant of moderate levels of physical activity. Electrocardiogram findings at his last office visit showed no abnormalities. At autopsy, heavy calcifications of the aortic valve are seen. Which of the following most likely preceded the aortic valve changes observed in this patient? a)Cell necrosis b)Extracellular amyloid deposition c)Hypercalcemia d)Intracellular hemosiderin accumulation e)Pathologic cell hypertrophy

Answer: Choice A (Cell necrosis) -This patient's aortic valve calcifications most likely represent dystrophic calcification affecting an aging aortic valve. Dystrophic calcification is considered a hallmark of cell injury and death, occurring in all types of necrosis (e.g., coagulative, fat, caseous, liquefactive) in the setting of normal calcium levels. -Grossly, dystrophic calcium deposits are seen as fine, gritty, white granules or clumps. On hematoxylin and eosin staining, these deposits typically appear as dark-purple, sharp-edged aggregates. Deposits that develop lamellated outer layers are described as psammoma bodies (shown in picture) -Dystrophic calcification in aged (or damaged) cardiac valves is thought to be the result of endothelial and fibroblast death secondary to chronic hemodynamic stress (can be accentuated by valvular abnormalities) or atherosclerotic inflammation. Subsequent release of cellular degradation products into the valvular interstitium then promotes calcification and thickening of the valve leaflets and annulus. -These changes are often benign in elderly adults (aortic sclerosis); however, over time, progressive valvular stiffening can lead to outflow obstruction (calcific aortic stenosis) Choice B= Extracellular amyloid deposition occurs in patients with amyloidosis. Amyloidosis can cause restrictive cardiomyopathy and heart failure but is not directly associated with calcification of the aortic valve. Choice C= Hypercalcemia is associated with metastatic calcification affecting normal tissues and organs. In this process, calcium deposition typically occurs in more alkaline tissues involved in acid excretion, such as the kidneys, lungs, systemic arteries, and gastric mucosa Choice D= Intracellular hemosiderin accumulation is common in patients who have hemolytic anemia or who undergo frequent blood transfusions. It is not directly associated with calcification of the aortic valve. Choice E= Pathologic cell hypertrophy of the LV cardiomyocytes (i.e., LVH) can occur as the result of severe aortic stenosis

-Age: newborn girl -Evaluated in the nursery for a heart murmur. The infant was born at 39 weeks gestation via spontaneous vaginal delivery following an uncomplicated pregnancy -She has been breastfeeding well and has urinated 3 times and stooled once. Approximately 1 hour after delivery, a 2/6 systolic murmur could be heard at the left upper sternal border. At age 8 hours, the murmur could be heard during both systole and diastole -Examination at age 24 hours revealed no murmur. Which of the following is most likely responsible for the disappearance of this patient's murmur? a)Closure of an arterial left-to-right shunt b)Closure of an arterial right-to-left shunt c)Closure of an atrial left-to-right shunt d)Closure of an atrial right-to-left shunt e)Closure of a ventricular left-to-right shunt f)Closure of a ventricular right-to-left shunt

Answer: Choice A (Closure of an arterial left-to-right shunt) -The changes in the infant's heart murmur throughout the first day of life are most likely explained by the evolution of a patent ductus arteriosus (PDA) -During fetal life, blood flows right to left through a PDA from the pulmonary artery to the aorta. This is due high pulmonary vascular resistance (PVR) caused by the constricted pulmonary arteries and low systemic vascular resistance (SVR) caused by the low resistance of the placenta. However, after delivery, increased oxygen tension and decreased carbon dioxide tension in the lungs causes pulmonary vasodilation and reduced PVR. At the same time, umbilical cord clamping disconnects the placenta from the infant's systemic circulation, leading to increased SVR. These changes decrease pulmonary pressure and increase systemic pressure, reversing right-to-left blood flow across the PDA to left-to-right shunting -Just after birth, pulmonary diastolic pressure is still high relative to systemic diastolic pressure, so the murmur is heard best (or only) during systole. As the PVR continues to fall, systemic pressure exceeds pulmonary pressure throughout the cardiac cycle, and the classic machine-like continuous murmur is heard. When the PDA closes (usually 24-48 hours after birth), the murmur disappears Choices B, D and F= After birth, right-to-left shunting (i.e., Eisenmenger syndrome) may eventually develop through a PDA, an ASD, or a VSD. However, postnatal right-to-left shunting usually causes cyanosis (not noted in this patient), and the pressure gradient is not typically large enough to cause a murmur Choice C= Left-to-right flow through an ASD does not typically generate a murmur itself. The characteristic murmur of ASD, a systolic murmur at the left upper sternal border, is caused by increased blood flow through the pulmonic valve. However, the murmur does not change to a continuous murmur Choice E= Left-to-right flow through a VSD can cause a holosystolic murmur at the left sternal border, but the murmur does not have a diastolic component

question 2 of 2 -In the same midesophageal position, the probe is rotated so that it now faces posteriorly. Which of the following structures will be best visualized with the probe's new orientation? a)Descending aorta b)Pulmonary artery c)Pulmonary veins d)Superior vena cava e)Tricuspid valve

Answer: Choice A (Descending aorta) -The aorta has 4 major divisions: the ascending aorta, the aortic arch, the descending thoracic aorta, and the abdominal aorta. The ascending aorta lies posterior and to the right of the main pulmonary artery. The aortic arch travels above the right pulmonary artery and the left bronchus. The brachiocephalic, left common carotid, and left subclavian arteries (in that order) originate from the superior aspect -The descending thoracic aorta travels down the anterior surface of the vertebral column, becoming the abdominal aorta as it crosses the diaphragm. As it descends, the aorta moves from the left side of the vertebral column toward the midline; at the level of the cardiac chambers, the descending aorta lies posterior to the esophagus and the left atrium. This permits clear visualization of the descending aorta during transesophageal echocardiography (TEE), allowing for detection of abnormalities such as dissection or aneurysm -The other answer choices describe structures located anterior to the esophagus Choice B= The main pulmonary artery ascends anteriorly and to the left of the ascending aorta and is directed toward the left shoulder. After the pulmonary artery bifurcates, the right pulmonary artery travels horizontally under the aortic arch posterior to the superior vena cava, and the left pulmonary artery courses superiorly over the left main bronchus Choice C= The superior and inferior pulmonary veins arise bilaterally from each lung and enter the left atrium. The proximal 2-3cm of the pulmonary veins contain cardiac muscle within the media and function like sphincters during atrial systole Choice D= The superior vena cava (SVC) is formed behind the right first costal cartilage by the merger of the right and left brachiocephalic veins. It returns blood from the head, neck, and upper extremities to the right atrium of the heart. Mediastinal neoplasm can compress the SVC and result in SVC syndrome Choice E= The tricuspid valve is located between the RA and RV and is composed of 3 valve leaflets, the annulus, supporting chordae tendineae, and the papillary muscles. It is commonly infected (endocarditis) in IV drug users

-Age: 60 years man -Comes to the office with a 6-month history of exertional chest pain that remits with rest. His other medical conditions include hypertension, diabetes mellitus, and hypercholesteremia -An exercise stress test is positive for inducible ischemia. Cardiac catheterization shows 80% occlusion of the right coronary artery and 60% occlusion of the left coronary artery. The first step in the pathogenesis of this patient's coronary artery disease most likely involved which of the following cell types? a)Endothelial cells b)Interstitial fibroblasts c)Macrophages d)Mast cells e)Pericytes f)Platelets g)Smooth muscle cells

Answer: Choice A (Endothelial cells) -This patient's presentation is consistent with coronary artery atherosclerosis. The pathogenesis of atherosclerosis likely begins with endothelial cell injury. Chronic endothelial cell injury may result from hypertension (and related hemodynamic factors), hyperlipidemia, smoking, and diabetes mellitus. Such injury leads to endothelial cell dysfunction and/or exposure of subendothelial collagen (endothelial cell denudation) -Endothelial cell dysfunction results in monocyte and lymphocyte adhesion and migration into the intima (Choice C), while exposure of subendothelial collagen promotes platelet adhesion (Choice F). Growth factors produced by monocytes and platelets stimulate medial smooth muscle cell (SMC) migration into the intima (Choice G). At the same time, increased vascular permeability allows LDL cholesterol into the intima, where it is phagocytosed by the accumulating macrophages and SMCs to produce lipid-laden foam cells (fatty streak) -The continued release of cytokines and growth factors result in a chronic inflammatory state within the underlying intima. This promotes further deposition of LDL cholesterol within the intima and stimulates SMC proliferation with increased production of collagen and proteoglycans. Necrosis of foam cells results in release of toxic oxidized LDL into the extracellular matrix, perpetuating a cycle of injury. The lesion eventually organizes into a core of lipid debris surrounded by monocytes and lymphocytes covered by a fibrous cap with intermixed SMCs (fibrofatty atheroma) Choice B= Fibroblasts do not contribute significantly to atheroma formation. The fibrous tissue, including the fibrous cap, of atheromas is synthesized by SMCs that have migrates to the intimal layer in which the plaque forms Choice D= Mast cells are not significantly involved in the pathogenesis of atherosclerosis Choice E= Pericytes are pluripotent cells that surround the smallest blood vessels (especially postcapillary venules). Because atherosclerosis affects the large elastic arteries and large- and medium-sized muscular arteries, these cells do not play a significant role

-Age: 32 years man -Evaluated in the ED due to fever, night sweats, and chills over the last several days. The patient has been using IV drugs recently as he is "stressed out". He has been otherwise healthy with no medical problems. -Temp. (38.3C), BP (120/80), pulse (105, regular). Further evaluation reveals aortic valve endocarditis with an intracardiac abscess and small fistula formation between the aortic root and right ventricle. -Doppler ultrasound interrogation of the fistula will most likely reveal which of the following blood flow patterns? a)Flow from the aortic root to the right ventricle continuously b)Flow from the aortic root to the right ventricle only in diastole c)Flow from the aortic root to the right ventricle only in systole d)Flow from the right ventricle to the aortic root continuously e)Flow from the right ventricle to the aortic root only in diastole f)Flow from the right ventricle to the aortic root only in systole

Answer: Choice A (Flow from the aortic root to the right ventricle continuously) -This patient with aortic valve endocarditis has developed an intracardiac fistula between the aortic root and RV. Aortocavitary fistulas are an uncommon complication of bacterial endocarditis caused by extension of the infection from the valve to the adjacent myocardium. -Echocardiography with doppler analysis can be used to detect and quantify shunts in patients with intracardiac fistulas. During the normal cardiac cycle, central aortic pressure (e.g., 120/80) is higher than the RV pressure (e.g., 25/5) during systole and diastole. Consequently, in patients with Aortocavitary fistula, Doppler interrogation will most likely demonstrate continuous blood flow from the higher-pressure aortic root to the lower-pressure RV (left-to-right cardiac shunt (Choices B and C). This can lead to a continuous murmur heard on cardiac auscultation. Choices D, E and F= Pressure in the aortic root is always greater than in the RV throughout systole and diastole; therefore, blood flow from the RV to the aortic root would not be expected in this patient.

-Age: 32 years man -Comes to the ED with chest pain that started earlier in the day. The pain is midline and sharp and increases with deep breaths but decreases when the patient leans forward. -He had a mild respiratory illness a week ago. Other medical history is unremarkable. The patient is al lifetime nonsmoker and has no family history of early-onset heart attack, sudden death, or cardiomyopathy. BP (120/70), pulse (110, regular). Which of the following physical examination findings is most expected in this patient? a)Friction rub b)Kussmaul sign c)Pericardial knock d)Pulsus paradoxus e)S3

Answer: Choice A (Friction rub) -The chest pain of acute pericarditis is sharp and pleuritic and characteristically decreases when the patient sits and leans forward, which decreases pressure on the parietal pericardium. -Fibrinous/serofibrinous pericarditis is the most common type of pericarditis, and a pericardial friction rub (described as high pitched, leathery and scratchy) is the most specific physical finding. Fibrinous pericarditis may be caused by MI, rheumatologic disease (e.g., SLE), uremia, or viral infection. This patient's history of a recent upper respiratory illness suggests a viral etiology. Choice B= During inspiration, jugular venous pressure normally decreases as blood is pulled into the right side of the heart. Kussmaul sign is a paradoxical increase in jugular venous pressure on inspiration. It occurs because of impaired right-sided diastolic filling in conditions such as constrictive pericarditis, restrictive cardiomyopathy, and tricuspid stenosis. Although constrictive (chronic) pericarditis can result from prolonged pericardial inflammation (over months to years), this patient's presentation is consistent with acute pericarditis, which is not associated with impaired right-sided diastolic filling Choice C= A pericardial knock is a brief, high-frequency, precordial sound heard in early diastole (shortly after S2) in patients with constrictive pericarditis. It occurs earlier than S3 and may be confused with the opening snap of mitral stenosis Choice D= Pulsus paradoxus is a drop in SBP >10mmHg during inspiration. It represents an exaggeration of the normal reduction in SBP during inspiration. It can be seen in cardiac tamponade, constrictive pericarditis, and severe asthma or COPD Choice E= An S3 typically results from reverberation of the ventricular walls during rapid filling of an overfilled ventricle. It is usually associated with heart failure as occurs with dilated cardiomyopathy, severe mitral regurgitation, or severe aortic regurgitation. A right-sided S3 can occur with right-sided heart failure

-Age: 58 years man -Dyspnea and chronic exertional angina is evaluated for coronary revascularization. History: hypertension, type 2 diabetes, hypercholesteremia. BP (130/80), pulse (72, regular) -Cardiopulmonary examination is normal with the exception of a 4th heart sound. Echocardiogram reveals hypokinesia of the anterior wall of the LV and a LV ejection fraction of 35%. The patient undergoes coronary artery bypass grafting. -Repeat echocardiogram 10 days after the surgery shows that hypokinesia is no longer evident and LVEF has increased to 50%. Which of the following best explains the changes in the cardiac contractility and wall motion seen in this patient? a)Hibernating myocardium b)Ischemic preconditioning c)Post-infarction myocardial scarring d)Reperfusion injury e)Ventricular remodeling

Answer: Choice A (Hibernating myocardium) -Myocardial hibernation refers to a state of chronic myocardial ischemia in which both myocardial metabolism and function are reduced to match a concomitant reduction in coronary blood flow (due to moderate/severe flow-limiting stenosis). -This new equilibrium prevents myocardial necrosis. Chronically hibernating myocardium demonstrates decreased expression and disorganization of contractile and cytoskeletal proteins, altered adrenergic control, and reduced calcium responsiveness. These changes lead to decreased contractility and LV systolic dysfunction. However, coronary revascularization and subsequent restoration of blood flow to hibernating myocardium improves contractility and LV function Choice B= Ischemic preconditioning is a phenomenon in which brief repetitive episodes of myocardial ischemia, followed by reperfusion, protect the myocardium from subsequent prolonged episodes of ischemia. In clinical terms, repetitive episodes of angina prior to MI can delay cell death after complete coronary occlusion and therefore provide greater time for myocardial salvage with coronary revascularization Choice C= Post-infarction myocardial scarring refers to the replacement of the myocardium with collagen and fibrous tissue. This leads to complete loss of myocardial contractile function, and cardiac contractility would not be expected to improve with coronary revascularization Choice D= Reperfusion injury refers to further myocardial cell damage due to restoration of blood flow after a sudden ischemic event. The clinical manifestations of reperfusion include arrhythmias, microvascular dysfunction with myocardial stunning, and myocyte injury and death Choice E= Ventricular remodeling refers to a change in the structure (Cardiac mass or dimensions) or function of the heart in response to cardiac injury or hemodynamic changes (pressure/volume overload). Remodeling can occur with long-term myocardial hibernation, but it would take weeks to months (not days) to recover following reperfusion injury

-Age: 67 years man -Brought to the ED by his son after a syncopal episode. The son was helping his father clean out the garbage when the father said he felt dizzy. -As the son was helping him into a chair, the patient lost consciousness. He woke up spontaneously about a minute later without any disorientation or confusion. ECG demonstrates bradycardia with a regular rhythm and narrow QRS complexes. -There is complete desynchronization between the P waves and QRS complexes. Which of the following locations is most likely pacing this patient's ventricles a)His bundle b)Left bundle branch c)Left ventricular muscle d)Purkinje system e)Sinoatrial node

Answer: Choice A (His bundle) -The electrical impulses in the myocardial conduction system are normally initiated by the SA node at a rate of 60-100bpm. These impulses are then transmitted through the atria to the AV node, then on to the His bundle, bundle branches, Purkinje fibers, and ventricular myocardium. Most of the conduction system has its own intrinsic pacemaker, which is normally suppressed by the more rapid SA node pacemaker but triggers when a signal from further up the conduction system is not received. -This patient's ECG shows complete dissociation of P waves and QRS complexes consistent with third-degree (complete) AV block. In third degree AV block, electrical impulses coming from the SA node are blocked before being transmitted to the ventricles; the point of blockade usually is within the AV node or His bundle. The cells located immediately distal to the blockade (e.g., in the His bundle) never receive the impulse from the SA node and therefore begin generating their own pacemaker impulse that is transmitted to the ventricles. -On ECG, the SA node impulses continue to march out as P waves, and the His bundle impulses generate QRS complexes (an escape rhythm) as the intrinsic rate of the His bundle pacemaker (e.g., 40-60/min). Because the atria and the ventricles are not communicating, the P waves and QRS complexes have no relation to one another Choices B and D= When electrical impulses are initiated below the AV node and His bundle, the heart rate typically slows to 25-40/min (based on the intrinsic pacemaker rate of the bundle branches and Purkinje system). On ECG, the QRS complexes are typically wide compared to the narrow QRS complexes initiated by the AV node and His bundle Choice C= The ventricular myocardium does not have an intrinsic pacemaker and typically must receive electrical impulses from the Purkinje system Choice E= When the SA node is initiating impulses through a normal conduction system, the heart rate is 60-100bpm. ECG shows P waves quickly followed by a narrow QRS complex, representing normal atrial depolarization followed by normal ventricular depolarization (normal sinus rhythm)

-Age: 33 years woman -Recently immigrated to the US is brought to the ED with severe shortness of breath and hemoptysis. Physical exam= diastolic murmur. Chest x-ray= severe pulmonary vascular congestion and edema -She admitted to the hospital, treated with diuretics, and begins to feel better. However, during her hospitalization, she develops right-sided hemiparesis. -Which of the following additional findings in this patient would be most suggestive of combined disease involving the mitral and aortic valves rather than exclusive mitral involvement? a)Increased left ventricular diastolic pressure b)Increased pulmonary artery pressure c)Increased pulmonary capillary wedge pressure d)Reduced pulmonary vascular compliance e)Right ventricular dilation f)Tricuspid regurgitation

Answer: Choice A (Increased left ventricular diastolic pressure) -This patient's physical examination findings are suggestive of mitral stenosis (MS). In isolated MS, cardiac and pulmonary pressure proximal to the stenotic mitral valve are markedly elevated. However, diastolic pressure in the LV is usually near normal or even decreased with severe stenosis. Increased LV diastolic pressure in a patient with MS suggests concomitant aortic valve dysfunction -Rheumatic heart disease almost always affects the mitral valve, but both the mitral and aortic valves are affected in about 25% of cases. Aortic valve involvement typically involves combined aortic stenosis and regurgitation, both of which can increase LV diastolic pressure. Rheumatic deformation of the aortic valve can also predispose to infective endocarditis even when the valve is functionally normal -This patient's right-sided hemiparesis is most likely due to embolism, which could have originated from an atrial mural thrombus secondary to atrial dilation from MS or an endocarditis-related valvular vegetation (less likely) Choices B and D= Severe, isolated MS can cause increased pulmonary artery pressure (pulmonary hypertension) via backward transmission of elevated pulmonary capillary wedge pressure. Longstanding pulmonary hypertension can result in reduced pulmonary vascular compliance due to endothelial remodeling and capillary obliteration Choice C= Pulmonary capillary wedge pressure (an estimate of left atrial pressure) will increase with any significant left-sided valvular pathology (i.e., both isolated MS and combined aortic and mitral valve disease) Choices E and F= RV dilation can occur in MS when the resulting pulmonary hypertension is severe enough to cause right heart failure. Tricuspid regurgitation can occur as a complication of right ventricular dilation

Question 1 of 2 -Age: 52 years man -Brought to the ED for evaluation of fever, chills, and malaise. Cardiopulmonary examination reveals a new holosystolic heart murmur that radiates toward the axilla -Blood cultures are obtained, and the patient undergoes transesophageal echocardiography. -The ultrasound probe is placed on the midesophagus facing anteriorly, and the cardiac chambers are interrogated. Which of the following chambers is closest to the probe? a)Left atrium b)Left ventricle c)Right atrium d)Right ventricle

Answer: Choice A (Left atrium) -The patient's presentation suggests acute endocarditis, with confirmation pending the results of the blood cultures and cardiac imaging. Transesophageal echocardiography (TEE) uses ultrasound waves generated from within the esophagus to produce clear images of the neighboring cardiac structures. The left atrium makes up the majority of the heart's posterior surface, with the esophagus passing immediately posterior to the heart. Therefore, the esophagus lies within close proximity to the left atrium. This allows the left atrium, atrial septum and mitral valve to be particularly well visualized on TEE -Due to its proximity, conditions that result in left atrial enlargement (e.g., mitral stenosis or regurgitation) can cause dysphagia through external compression of the esophagus Choice B= The LV forms the majority of the inferior (diaphragmatic) surface of the heart and the left border of the heart on frontal chest x-ray Choice C= The right atrium, along with the superior vena cava, forms the right lateral cardiac border on frontal chest x-ray Choice D= The RV forms the anterior (sternal) surface of the heart and the majority of its inferior border on frontal chest x-ray

-Age: 52 years man -Comes to the office with concerns about a "heart problem". 3 years ago, was told he had a heart murmur. Currently, no specific complaints, but he mentions that he has become tired more easily in the past year -He has no chest pain or lower extremity swelling or significant medical history. The patient was healthy as a child and received all his childhood vaccinations. BP (145/90), pulse (80, regular). Physical exam: holosystolic murmur best heard at the apex of the heart that radiates to the axilla. The lungs are clear on auscultation and abdominal examination is unremarkable. Which of the following is the best indicatory of the severity of this patient's valve disease? a)Presence of audible S3 b)Presence of audible S4 c)Presence of mid-systolic click d)S2-to opening snap time interval e)Splitting of S2 accentuated by inspiration

Answer: Choice A (Presence of audible S3) -A holosystolic murmur at the apex with radiation to the axilla is consistent with mitral regurgitation (MR). The murmur is generated by regurgitant blood flow from the LV back to the LA during systole. This results in elevated pressure and blood volume in the left atrium, which increases the amount of blood reentering the LV during diastole, an audible S3 gallop occurs when the LV is unable to accommodate the excess blood flow -An S3 is generated by the sudden cessation of blood flow into the LV during the passive filling phase of diastole. A higher volume of blood flow or a more dilated LV is more likely to produce an S3, and although an S3 can be a normal finding in healthy young adults, it is usually a pathologic finding in older adults that is classically associated with heart failure. In patients with MR, an S3 is the best indicator of severe MR with left-sided volume overload, the absence of an S3 can be used to exclude severe chronic MR Choice B= An S4 is a low-frequency diastolic sound that occurs during the atrial kick of ventricular diastole, and it reflects blood colliding with a stiff ventricular wall. It often indicates pathology such as hypertrophic cardiomyopathy or concentric left ventricular hypertrophy (e.g., due to hypertension or aortic stenosis). Choice C= A mid-systolic click is characteristic of mitral valve prolapse. It is caused by sudden tensing of the chordae tendineae as they are pulled taut by the valve leaflets ballooning into the left atrium. The timing of the click varies with LV volume; it occurs earlier in systole with physical maneuvers that decrease LV volume Choice D= An opening snap is an early diastolic sound heard after S2 in patients with mitral or tricuspid stenosis. The S2-to-opening snap interval is the time between the closure of the aortic valve and the abrupt halting of leaflet motion during opening of a stenotic mitral valve, a shorter S2-to-opening snap interval is indicative of more severe mitral stenosis Choice E= Pulmonic valve stenosis delays pulmonic valve closure leading to a widened splitting of S2. The splitting is further accentuated by the increase in venous return to the right side of the heart that occurs with inspiration

Question 2 of 2 -This patient is tachypneic and unable to speak in full sentences. Examination reveals prolonged expiration and prominent bilateral wheezing. Heart sounds are normal. -Chest imaging= normal-sized heart and hyperinflated lungs with a flattened diaphragm. Bedside echocardiogram reveals no intrapericardial fluid accumulation or pericardial thickening. Which of the following physiologic changes is most likely to provide immediate relief in this patient? a)cAMP accumulation in smooth muscle cells b)Cell membrane stabilization of mast cells c)Inhibition of eosinophil degranulation d)Interrupted histamine receptor firing in epithelial cells e)Opening of calcium channels in smooth muscle cells

Answer: Choice A (cAMP accumulation in smooth muscle cells) -This patient's presentation with dyspnea, tachypnea, prolonged expiration, and bilateral wheezing is suggestive of acute exacerbation of obstructive pulmonary disease (asthma or COPD). Acute obstructive pulmonary exacerbations are primarily treated with beta-adrenergic agonists, which produce relaxation of bronchial smooth muscle by stimulating beta-2 adrenergic receptor. This receptor is a Gs protein-coupled receptor that activates adenylyl cyclase and increases intracellular cAMP concentrations -Asthma and COPD exacerbation are the most frequent causes of pulsus paradoxus in the absence of significant pericardial disease (pericardial effusion or constrictive pericarditis). There is a small normal variation in intrathoracic pressure with respiration, which drops 2-5mmH below the atmospheric pressure during inspiration. In patients with severe asthma or COPD exacerbation, this drop in intrathoracic pressure becomes greatly exaggerated and is transmitted to extrathoracic structures. This leads to an excessive drop in BP with inspiration that is detected as pulsus paradoxus Choice B= Cromolyn sodium inhibits mast cell degranulation and release of histamine and leukotrienes. It is used occasionally for management of chronic asthma but is not effective for immediate relief of acute asthmatic or COPD exacerbations Choice C= Eosinophil degranulation would be impaired by corticosteroids, which play an important role in treatment of acute asthma or COPD exacerbations. However, beta agonists provide immediate relief, whereas corticosteroids would take several hours to a few days to take effect Choice D= Histamine is one of several mediators released in response to a hypersensitivity reaction that can lead to bronchial smooth muscle contraction. However, antihistamine therapy is not an effective treatment for asthma or COPD exacerbation Choice E= Calcium channel agonists are not used in acute treatment of asthma or COPD exacerbation. On the contrary, increased calcium influx could potentially worsen bronchial smooth muscle contraction

-Age: 35 years woman -Comes to the office due to solid food dysphagia for the past several months. The patient has a sensation of food getting stuck in the mid-thorax but has no pain or reflux. She also has palpitations and dyspnea on exertion. -The patient immigrated to the US from India 1 year ago and was treated in her home country for rheumatic heart disease. Barium esophagram demonstrates extrinsic compression on the mid-esophagus. Enlargement of which of the following is most likely causing this patient's dysphagia? a)Left atrium b)Left ventricle c)Pulmonary artery d)Right atrium e)Right ventricle

Answer: Choice A (left atrium) -This patient with a history of rheumatic heart disease has likely developed mitral stenosis/regurgitation (e.g., palpitations, exertional dyspnea), leading to left atrial enlargement and extrinsic compression of the esophagus (e.g., solid food dysphagia) -The posterior surface of the heart is mostly formed by the left atrium, which lies directly over the esophagus. Longstanding mitral stenosis/regurgitation can lead to progressive left atrial enlargement, further displacing the LA posteriorly and causing external compression of the mid-esophagus with dysphagia (cardiovascular dysphagia). On rare occasions, left atrial enlargement can also cause compression of the left recurrent laryngeal nerve, leading to voice hoarseness and chronic cough Choice B= The LV makes up most of the left surface of the heart, which abuts the left lung Choice C= The pulmonary artery emerges from the RV and branches into the right and left pulmonary arteries below the aortic arch Choices D and E= The anterior (sternocostal) surface of the heart is mostly formed by the right atrium superiorly and the RV inferiorly. Blood from the coronary sinus and inferior and superior venae cavae drain into the right atrium, which then drains blood into the RV through the tricuspid valve.

-Age: 43 years man -Comes to the ED due to acute-onset shortness of breath. He has no medical conditions and does not use alcohol, tobacco, or illicit drugs. -The patient works as a box handler at a warehouse and has excellent exercise tolerance. Family history is insignificant. BP (98/60), pulse (112, regular). A short systolic murmur is heard at the apex. Bilateral crackles are present on lung auscultation. Bedside echocardiography reveals a flail posterior mitral leaflet due to chorda rupture and severe mitral regurgitation -Which of the following best reflects this patient's left ventricular hemodynamics compared to the normal heart? a)Afterload decrease, preload decrease, ejection fraction decrease b)Afterload decrease, preload increase, ejection fraction increase c)Afterload increase, preload decrease, ejection fraction decrease d)Afterload increase, preload decrease, ejection fraction increase e)Afterload increase, preload increase, ejection fraction increase

Answer: Choice B (Afterload decrease, preload increase, ejection fraction increase) -This patient has acute mitral regurgitation (MR) due to a spontaneous chordae tendineae rupture. In acute MR, the LA is of normal size and compliance and is suddenly exposed to a large volume of regurgitant flow from the LV. This leads to very high LA pressures and subsequent pulmonary edema -In addition, because the LV must accommodate the regurgitant blood volume along with normal pulmonary venous return, there is increased LV end-diastolic volume (preload). Meanwhile, the incompetent mitral valve forms a low-resistance regurgitant pathway for blood to flow into the LA during ventricular systole, decreasing LV afterload. The decreased afterload leads to increased LV ejection fraction (via increased stroke volume); however, much of the stroke volume is lost to regurgitation into the LA, resulting in overall decreased forward stroke volume and reduced cardiac output (manifesting as hypotension and cardiogenic shock) -In chronic MR, compensatory LA enlargement allows the LA to receive the regurgitant volume at lower filling pressures, preventing pulmonary edema from developing. Chronic volume overload also causes the LV to undergo substantial enlargement due to eccentric hypertrophy. Early on, the larger ventricular volume helps maintain forward stroke volume even in the setting of substantial regurgitant flow. However, with prolonged hemodynamic overload, progressive eccentric remodeling of the LV becomes maladaptive, resulting in an overwhelming increase in wall stress with eventual contractile dysfunction. The impaired contractility culminates in decompensation of chronic MR with reduced forward stroke volume (decreased cardiac output) and increased left-sided filling pressures (pulmonary edema)

-Age: 80 years man -At autopsy, the heart has a small left ventricular cavity and thickening of the left ventricle walls, which measure 1.7cm (normal: 0.7-1.1) at the posterior and septal wall locations. Microscopic examination shows diffusely enlarged cardiomyocytes, with prominent nuclei evident at low power. Increased expression of which of the following substances most likely contributed to these pathologic findings? a)Adenosine b)Angiotensin II c)Calcitonin gene-related peptide d)Interleukin-1 e)Nitric oxide synthase

Answer: Choice B (Angiotensin II) -The pathology evaluation of this patient's heart shows uniformly thickened LV walls (resulting in decreased ventricular cavity size) and enlarged cardiomyocytes with highly visible nuclei, which are consistent with concentric LV hypertrophy. This condition commonly results from chronic hypertension, which leads to increased local expression of the vasoconstrictors angiotensin II and endothelin in the heart; these 2 mediators likely have a prominent role in the development of LVH -Although all medications that lower systemic BP (e.g., calcium channel blockers, thiazide diuretics) can reduce concentric LVH, angiotensin II receptor blockers provide the greatest reduction in remodeling, suggesting a role of angiotensin II in the development of LVH that is independent of BP. Endothelin receptor blockers (e.g., bosentan) are not typically used to treat systemic hypertension, but they may reduce RV hypertrophy when used to treat pulmonary arterial hypertension -Local angiotensin II production also likely has a prominent role in the development of eccentric LVH in response to LV volume overload Choice A= A compensatory increase in adenosine expression occurs in response to myocardial ischemia and reperfusion of the heart. Adenosine dilated blood vessels, especially coronary arteries, to improve circulation; it does not contribute to LVH Choice C= Calcitonin gene-related peptide (CGRP) is a neuropeptide that is a potent vasodilator, and it may have a role in protecting the heart from heart failure and LVH. CGRP blockers are used to prevent migraines by reducing the size of abnormally dilated blood vessels in the brain Choice D= IL-1 is a proinflammatory cytokine implicated in atherosclerosis development; high levels are expected in coronary artery disease but not in LVH. IL-1 blockers can be used to treat autoimmune inflammatory conditions (e.g., rheumatoid arthritis) Choice E= Nitric oxide synthases is an enzyme that catalyzes the production of nitric oxide (a potent vasodilator). Nitric oxide is protective against LVH, and relatively low levels of nitric oxide synthase are expected with LVH

-Age: 2 months boy -Found unresponsive in his crib. He had previously been well and had a normal physical examination at his 2-month visit approximately 1 week earlier. -An autopsy is performed to determine the cause of death. The pathologist concludes that the infant likely died of sudden infant death syndrome. -The autopsy shows an incidental finding of bicuspid aortic valve. If this patient had survived, he would have been at greatest risk for which of the following events? a)Aortic dissection in his 30s b)Aortic stenosis in his 50s c)Atrial fibrillation in his 40s d)Endocarditis in his 20s e)Severe aortic stenosis in infancy f)Ventricular arrhythmia in childhood

Answer: Choice B (Aortic stenosis in his 50s) -Bicuspid aortic valves (right and left aortic cusp fusion) occur in approximately 1-2% of live births, making it one of the most common congenital heart defects. The abnormally shaped valve experiences increased hemodynamic stress, which accelerates the normal aging process and causes premature atherosclerosis and calcification of the aortic valve. -These deposits begin accumulating as early as adolescence and lead to aortic stenosis in >50% of affected patients. Symptomatic aortic stenosis develops on average around age 50 (10 years earlier than the average onset of senile calcific aortic stenosis in patients with normal aortic valves) Choice A= Patients with bicuspid aortic valves have a higher risk of aortic dissection. However, the absolute risk of aortic dissection is still <1%. As with other complications of bicuspid aortic valves, the risk increases with advancing age. Choice C= Atrial fibrillation has not been associated with bicuspid aortic valves Choice D= Patients with bicuspid aortic valves are at increased risk for infective endocarditis. The risk is 2-3% and rises with age due to increasingly turbulent blood flow across the abnormal value Choice E= In early childhood, isolated bicuspid aortic valves usually are asymptomatic, without any hemodynamically significant degree of aortic outflow tract obstruction or murmurs. In contrast, congenital aortic stenosis usually produces a systolic murmur as a result of severe subvalvular, valvular, or supravalvular anatomic deficits Choice F= The most common cause of ventricular fibrillation at age <30 is hypertrophic cardiomyopathy. This condition is not associated with bicuspid aortic valves.

-Age: 72 years man -Comes to the office due to palpitations, fatigue, and dyspnea for the past 2 weeks. The patient experienced an episode of atrial fibrillation 4 years ago during hospitalization for pneumonia; the episode spontaneously resolved 2 days later. Since then, he has had 3 episodes of symptomatic atrial fibrillation that self-terminated within a week. -Other medical history includes hypertension, type 2 diabetes, and chronic heart failure. BP (130/84), pulse (110). The lungs are clear to auscultation. Cardiac examination demonstrates tachycardia with irregularly irregular heartbeats. ECG confirms atrial fibrillation. The patient's ventricular rate is controlled with beta blocker therapy, but the arrhythmia fails to resolve as in previous episodes. -Which of the following factors is most likely involved in the progression of this patient's arrhythmia? a)Abnormal atrioventricular nodal pathway b)Atrial structural remodeling c)Cardiac autonomic neuropathy d)Conduction system fibrosis e)Enhanced sinoatrial node automaticity

Answer: Choice B (Atrial structural remodeling) This patient's progression from paroxysmal atrial fibrillation (resolving spontaneously within 7 days) to persistent atrial fibrillation (lasting >7 days) is most attributable to atrial remodeling. The development of atrial fibrillation is primarily driven by both structural and electrical conduction changes in the atrial myocardium that predispose to the initiation and maintenance of electrical reentrant circuits and/or ectopic foci: -The structural component of atrial remodeling likely involves comorbidities that lead to chronic atrial stretching and dilation. These include hypertension, heart failure, and mitral valve disease. Physiologic age-related atrial dilation also contributes. -The electrical conduction component likely involves age-related changes as well as previous exposure to atrial fibrillation. During atrial fibrillation, the conduction system undergoes changes that facilitate propagation of the arrhythmia; over time it becomes more difficult to reverse (i.e., atrial fibrillation begets atrial fibrillation) As atrial remodeling progresses, the atria become increasingly susceptible to electrical triggers (e.g., ectopic foci in the pulmonary veins) that initiate fibrillation and more likely to maintain persistently disorganized electrical activity Choice A= An abnormal pathway in the AV node is the typical cause of AV nodal reentrant tachycardia. This arrhythmia most commonly affects young patients with an otherwise normal heart. Choice B= Cardiac autonomic neuropathy, which can develop in patients with Parkinson disease or poorly controlled diabetes mellitus, typically leads to an impaired sympathetic cardiac response. This often results in exercise intolerance and orthostasis but does not significantly contribute to cardiac arrhythmias Choice D= Conduction system fibrosis (often age-related) is a primary contributor to bradyarrhythmias, including sinus bradycardia (e.g., sick sinus syndrome) and AV block Choice E= Enhanced SA node automaticity causes sinus tachycardia (e.g., physiologic with exercise). During atrial fibrillation, electrical signals in the SA node are suppressed by widespread disorganized electrical activity throughout the atria.

-Age: 52 years man -Comes to the office due to a chronic cough for the last 3 weeks. The patient says, "I haven't been coughing up phlegm, but sometimes I notice some blood on the tissue when I cough particularly hard". He has smoked 2 packs of cigarettes daily for the past 30 years and drink 3-4 cans of beer on weekends. -He works as a welder on an assembly line and says his father died of lung cancer at age 70. Physical examination shows right-sided face and arm swelling and engorgement of subcutaneous veins on the same side of the neck. Which of the following veins is most likely obstructed in this patient? a)Axillary b)Brachiocephalic c)External jugular d)Internal jugular e)Subclavian f)Superior vena cava

Answer: Choice B (Brachiocephalic) -This patient has symptoms consistent with an obstructed right brachiocephalic (innominate) vein. This may be the result of external compression by an apical lung tumor or thrombotic occlusion as can occur when a central catheter has been in place for an extended period. The right brachiocephalic vein is formed by the union of the right subclavian vein and the right internal jugular vein. -The right external jugular vein drains into the right subclavian vein, so obstruction of the right brachiocephalic vein will also cause venous congestion of structures drained by the external jugular vein. It is important to note that the right brachiocephalic vein also drain the right lymphatic duct, which drains lymph from the right upper extremity, the right face and neck, the right hemithorax, and the right upper quadrant of the abdomen Choices A and E= The subclavian vein is the continuation of the axillary vein. Both drain blood from the upper extremity. Blockage at any of these 2 sites would cause unilateral arm swelling without associated facial swelling Choices C and D= The external jugular vein drains the scalp and portions of the lateral face. The internal jugular vein drains the brain and superficial face and neck. Obstruction of the internal jugular veins would not cause arm swelling Choice F= SVC syndrome results when the SVC is obstructed. The signs and symptoms of SVC syndrome are similar to those of obstructed right brachiocephalic (innominate) vein, except that both sides of the face, neck, and chest, and both arms would be involved

-Age: 44 years man -Comes to the office due to increasing shortness of breath. While climbing the stairs to his second-floor apartment, the patient must now stop halfway to catch his breath. -He has a history of nonischemic cardiomyopathy and chronic heart failure with reduced ejection fraction. After evaluation, treatment with a new medication is begun that improves his symptoms by increasing urine output and decreasing peripheral vascular resistance. The medication works by inhibiting a metalloprotease to prolong the action of endogenous polypeptides. These polypeptides are most likely secreted by which of the following cell types? a)Adrenal zona glomerulosa cells b)Cardiomyocytes c)Hepatic stellate cells d)Myocardial Purkinje fibers e)Renal juxtaglomerular cells f)Vascular endothelial cells

Answer: Choice B (Cardiomyocytes) In response to myocardial stretching due to volume overload, atrial and ventricular cardiomyocytes secrete atrial natriuretic peptide (ANP) and brain natriuretic peptide (BNP). These peptides are subsequently broken down and inactivated by the metalloprotease neprilysin. This patient with chronic heart failure was most likely given a neprilysin inhibitor (e.g., sacubitril), allowing for enhanced activity of ANP and BNP Once secreted, ANP and BNP bind to natriuretic peptide receptors to activate guanylate cyclase and form cyclic GMP, which stimulates diuresis and peripheral vasodilation to help alleviate volume overload. Several organs and tissues are affected as follows: -Kidneys: The afferent arterioles are dilated, and the efferent arterioles are constricted, raising the GFR and increasing urinary excretion of sodium and water. Renin secretion from the juxtaglomerular cells is inhibited (Choice E), counteracting RAAS activity -Adrenal glands: Secretion of aldosterone from the zona glomerulosa cells is inhibited (Choice A), further counteracting RAAS activity and bolstering the increase in sodium and water excretion -Blood vessels: Arteriolar and venular smooth muscle relaxes, producing vasodilation. There is also increased capillary permeability, leading to fluid extravasation into the interstitium and a decrease in circulating blood volume Choice C= Hepatic stellate cells (perisinusoidal cells of Ito) may help regulate hepatic blood flow via release of local mediators; however, these cells do not play a significant role in heart failure Choice D= Myocardial Purkinje fibers facilitate the organized transport of electrical impulses from the sinoatrial node through the atria, AV node , and ventricles; however, these fibers are not known to have endocrine function Choice F= Vascular endothelial cells regulate systemic blood flow via release of vasodilators, such as nitric oxide and prostacyclin, and vasoconstrictors, such as endothelin and thromboxane A2. However, these local mediators are not a significant target of therapy for chronic heart failure

-Age: 53 years woman -Comes to office due to progressive distention and tightness in her right lower extremity for the past year. She has had no shortness of breath, fever, or skin rash -The patient was diagnosed with melanoma of the right thigh approximately 2 years ago and underwent surgical resection -2 of her inguinal lymph nodes tested positive for metastasis, and she subsequently underwent inguinal lymphadenectomy and received adjuvant therapy. Recent imaging revealed no evidence of recurrent malignancy -History: hypertension, hyperlipidemia. Vital signs are normal; exam: clear lung sounds, normal heart sounds without murmurs or gallops. No jugular venous distention -Right lower extremity is significantly larger in diameter than the left. Edema is present up to the thigh, with mild skin indentation after application of pressure. The skin overlying the area is thickened and dry. Labs including CBC, serum metabolic panel and urinalysis are normal -Treatment of this patient's symptoms should include? a)Blockade of estrogen receptors b)Compression to increase lymphatic flow c)Dissolution of intravascular thrombus d)Inhibition of vascular endothelial growth factor e)Reduction of intravascular volume

Answer: Choice B (Compression to increase lymphatic flow) -The patient with chronic swelling and thickened skin of the right leg most likely has chronic lymphedema. Lymphedema can be congenital, but it most commonly results from an acquired disruption of lymphatic drainage that allows for accumulation of lymphatic fluid in the interstitial tissue -Malignancy and its treatment (e.g., radiation, lymphadenectomy) are frequent underlying causes; other causes include chronic inflammation (e.g., recurrent cellulitis, connective tissue disease) and parasitic infection (i.e., filariasis due to Wuchereria bancrofti). Obesity is often a strong contributing factor -Lymphedema usually presents with swelling, heaviness, and discomfort in one or more extremities. In early disease, the edema is pitting, and the skin remains soft. However, over time there is progressive deposition of subcutaneous collagen and adipose tissue, which leads to firm, dry, and thickened skin and nonpitting edema. Treatment is difficult and typically involves conservative management to increase lymphatic drainage via compression bandages or physiotherapy (i.e., manual lymphatic drainage) Choice A= Selective estrogen receptor modulators (e.g., tamoxifen) block estrogen receptors in breast tissue and can be useful in treating breast cancer. Breast cancer may present with upper extremity lymphedema, but lower extremity lymphedema would be extremely unusual Choice C= Anticoagulants (e.g., heparin) facilitate dissolution of acute deep venous thrombosis (DVT); however, acute DVT is less likely without active malignancy and would not explain this patient's year-long duration of symptoms Choice D= Inhibition of vascular endothelial growth factor is useful in the treatment of lymphangiosarcoma, which is a rare complication of lymphedema. Lymphangiosarcoma typically presents as purple maculopapular skin lesions, which are not present in this patient. Choice E= Diuretics reduced intravascular volume and are useful in treating lower extremity edema that results from elevated venous hydrostatic pressure in true volume overload states (e.g., heart failure, cirrhosis). However, they are ineffective and contraindicated for lymphedema because lymphedematous fluid cannot be easily mobilized form the interstitial space into the vasculature. When used for lymphedema, diuretics are likely to cause intravascular volume depletion and consequent acute kidney injury.

-Age: 4 years boy -Brought in by his parents for evaluation of a fever that has persisted for the past 5 days. He also has been more irritable than usual and has 2 or 3 episodes of vomiting. The patient has no prior medical problems and takes no medications -He has received all recommended vaccinations. He traveled to China last year to visit his grandparents and cousins but has not traveled outside of the country this year. -Temp. (38.9C). Physical exam: bilateral conjunctival infection with no exudates. His tongue is bright red and lips are cracked. Nonpitting edema is present on his hands and feet. Which of the following complications is this patient at greatest risk for developing? a)Aortic dissection b)Coronary artery aneurysm c)Monocular blindness d)Mononeuritis multiplex e)Proliferative glomerulonephritis f)Pulmonary cavitation

Answer: Choice B (Coronary artery aneurysm) Kawasaki disease is a vasculitis of medium-sized arteries that affects young children (usually age <5 years). It occurs most commonly in patients of Asian ethnicity. The diagnosis of Kawasaki disease is based on a fever for >5 days plus 4 of the following findings: -Bilateral non-exudative conjunctival injection (erythema) -Cervical lymphadenopathy -Mucositis: erythema of the palatine mucosa, fissured erythematous lips, "strawberry tongue" -Extremity changes" edema of hands and feet, erythema of palms and soles, desquamation of the fingertips (periungual) -Rash: polymorphous (usually urticarial) erythematous rash on the extremities that spreads centripetally to the trunk A serious complication of Kawasaki disease is coronary artery inflammation leading to the development of coronary artery aneurysm Choice A= Dissecting aneurysms of the aorta are associated with long-standing hypertension and Marfan syndrome Choice C= monocular blindness is a complication of temporal arteritis Choice D= Mononeuritis multiplex is a peripheral neuropathy of multiple individual nerves. It is associated with a variety of diseases, including granulomatosis with polyangiitis (Wegener's) and polyarteritis nodosa Choice E= Acute proliferative GN can occur after group A streptococcal skin/throat infections. Although symptoms such as fever, "strawberry" tongue, and desquamation can occur in scarlet fever, this patient's bilateral nonexudative conjunctivitis, swelling of the hands and feet, and persistent high-grade fever are more suggestive of Kawasaki disease Choice F= Pulmonary cavitations can occur following resolution of pulmonary TB, histoplasmosis, or sarcoidosis

-Multinational research institute conducting experiments on human circulatory physiology enrolls a healthy 30-year-old male volunteer to assess the oxygen consumption rate at various organs. During the study, the blood oxygen content of the aorta and several other vessels is measured at rest. The greatest difference in these measurements will most likely be between the aorta and which of the following blood vessels? a)Brachial vein b)Coronary sinus c)Internal jugular vein d)Portal vein e)Pulmonary artery f)Renal vein

Answer: Choice B (Coronary sinus) Arterial blood supply to the cardiac muscle (myocardium) is provided by the right and left coronary arteries arising directly from the aortic root. Most cardiac venous blood drains into the right atrium via the coronary sinus, with the remainder draining directly into the other chambers of the heart. Three specific features distinguish cardiac circulation from blood flow to skeletal muscle and viscera. -1) The LV is perfused only during diastole. Myocardial contraction during systole leads to compression of the coronary vessels and disruption of blood flow. Wall tension is highest near the endocardium, making the subendocardial region the most prone to ischemia -2) Myocardial oxygen extraction is very high. The heart has a capillary density far exceeding that of skeletal muscle. Oxygen extraction from arterial blood is very effective within the heart as the resting myocardium extracts 60-75% of oxygen from blood. This amount is higher than that extracted by any other tissue or organ in the body. As a result, the cardiac venous blood in the coronary sinus, before it reaches the right atrium and mixes with blood returning from the systemic circulation, is the most deoxygenated blood in the body (Choices A, C, D and F) -3) myocardial oxygen demand and coronary blood flow are tightly coupled. Because oxygen extraction by the resting heart is already very high, there is little capacity to increase myocardial oxygen extraction during periods of increased oxygen demand (e.g., during exercise). Therefore, increased oxygen delivery to the heart can be achieved only through increased coronary blood flow. Adenosine and nitric oxide are the most important vasodilators responsible for increasing coronary flow Choice E= Deoxygenated systemic blood returning via the vena cava contains more oxygen than coronary sinus venous blood. Because the pulmonary artery contains an admixture of blood returning via coronary sinus and systemic venous circulation, its oxygen content is also higher than pure coronary venous return

-Age: 46 years woman -Comes to the ED due to progressive dyspnea over the last 2 days. The patient's exercise tolerance has decreased dramatically, and she had to sleep in a sitting position last night. She has no history of cardiovascular disease. -The patient does not use tobacco or alcohol. Family history is unremarkable. BP (110/65), pulse (105, regular). Apical impulse is hyperdynamic. Cardiac auscultation reveals a diminished S1 and an apical holosystolic murmur radiating to the axilla. Diffuse pulmonary crackles are heard bilaterally. There is no peripheral edema. -ECG shows sinus tachycardia but is otherwise unremarkable. Which of the following would most likely increase the ratio of forward flow to regurgitant flow volume in this patient? a)Decreasing heart rate b)Decreasing systemic vascular resistance c)Decreasing venous return to the left ventricle d)Increasing left ventricular contractility e)Increasing venous return to the left ventricle

Answer: Choice B (Decreasing systemic vascular resistance) This patient's dyspnea, orthopnea, and crackles on lung auscultation are consistent with decompensated left-sided heart failure; the apical holosystolic murmur radiating to the axilla suggests mitral regurgitation (MR) as a contributing factor. In patients with MR, some of the blood in the LV is pumped forward through the aortic valve (forward stroke volume), and some is forced backward through the incompetent mitral valve (regurgitant stroke volume). The amount of blood flowing down each pathway is determined by the relative contribution of the resistance of each pathway to the total left ventricular afterload -Resistance to forward flow is primarily determined by the pressure in the aorta (systolic BP) -Resistance to regurgitant flow is determined by the mitral valve orifice size during systole and the degree of left atrial compliance. In chronic MR, the left atrium becomes more compliant, and the lower left atrial pressures facilitate greater regurgitant flow Left atrial pressure remains relatively constant over the short term, but aortic pressure can vary significantly with changes in systemic vascular resistance. A reduction in systemic vascular resistance leads to reduced systemic BP and an increase in the ratio of forward to regurgitant blood flow. Pharmacologic vasodilators (e.g., nitroprusside) therefore help to increase forward cardiac output and reduce pulmonary congestion in patients with MR Choices A and E= A decrease in heart rate leads to an increase in venous return to the left ventricle (preload) due to increased diastolic filling time. This causes an absolute increase in both forward and regurgitant blood flow (stroke volume increases in accordance with the Frank-Starling mechanism), but the ratio of forward to regurgitant flow is unlikely to significantly change Choice C= A reduction in venous return to the LV decreases stroke volume with absolute reductions in both forward and regurgitant flow, but is unlikely to significantly affect the ratio of forward to regurgitant flow Choice D= An increase in LV contractility increases stroke volume and leads to an increase in both forward and regurgitant blood flow. However again, as the relative resistance of each pathway is unchanged, the ratio of forward to regurgitant blood flow remains essentially the same

-Age: 68 years man -Comes to the office due to exertional shortness of breath and fatigue, which have progressed over the past year. History: hypertension, lifetime nonsmoker -His father died of a "heart attack" at age 70. BP (144/74), pulse (72, regular). Cardiac auscultation reveals a 3/6 ejection-type, late-peaking systolic murmur and a barely audible S2. The murmur diminishes in intensity during the straining phase of the Valsalva maneuver. Which of the following processes underlies the patient's current condition? a)Cardiomyocyte hypertrophy and disarray causing asymmetric septal thickening b)Differentiation of valve fibroblasts into osteoblast-like cells c)Myxomatous valve-thickening and elastin fragmentation d)Subendocardial granulomatous lesions with fibrinoid necrosis and subsequent fibrosis e)Thrombus formation on inflamed valvular endothelium

Answer: Choice B (Differentiation of valve fibroblasts into osteoblast-like cells) -This patient's soft S2 and late-peaking systolic ejection murmur that decreases in intensity with maneuvers that decrease LV blood volume (e.g., abrupt standing, Valsalva straining phase) are consistent with aortic stenosis, which most commonly occurs due to age-related calcific aortic valve disease (CAVD) -The early pathogenesis of CAVD is analogous to that of arterial atherosclerosis. The endothelium lining the aortic side of the aortic valve cusps is exposed to the same high pressure and turbulent blood flow as the aortic vascular endothelium. As occurs with atheroma development in the vascular endothelium, these mechanical forces (along with smoking, hyperglycemia, and hyperlipidemia) over time cause damage to the aortic valve cusp endothelium, triggering endothelial dysfunction and the onset of a similar atherosclerotic process. There is subendothelial lipid deposition and infiltration of inflammatory cells (e.g., macrophages, T lymphocytes) followed by the release of inflammatory mediators (e.g., IL-1 beta, TGF-beta1) -Subsequently, there is increased production of proteins involved in tissue calcification (e.g., osteopontin). Fibroblasts differentiate into osteoblast-like cells, leading to aberrant bone matrix deposition with progressive valvular calcification and stenosis Choice A= Cardiomyocyte hypertrophy and disarray with asymmetric septal thickening occur in hypertrophic cardiomyopathy. As with aortic stenosis, patients have a harsh crescendo-decrescendo systolic murmur; however, the murmur increases (rather than decreases) in intensity with maneuvers that reduce LV blood volume Choice C= Myxomatous valve thickening and elastin fragmentation is typically seen in mitral valve prolapse. Patients have a nonejection systolic click and a mid-to-late systolic murmur that occurs earlier in systole with decreases in LV blood volume Choice D= Subendocardial granulomatous lesions with fibrinoid necrosis and subsequent fibrosis is seen in patients with rheumatic heart disease. Patients with rheumatic aortic stenosis typically have mitral valve involvement as well, which would be evidenced by a murmur of mitral stenosis or regurgitation Choice E= Inflammation of valvular endothelium with deposition of sterile platelet thrombi is seen in nonbacterial thrombotic endocarditis (also known as marantic, verrucous, or Libman-Sacks endocarditis). The mitral or aortic valve is typically involved, usually leading to regurgitation rather than stenosis.

-Age: 35 years man -Comes to the office with progressive fatigue, dyspnea on exertion, and lower extremity edema over the last 2 weeks. Preceding these symptoms, he had an episode of fever, runny nose, and myalgias that resolved after several days. -The patient has no other medical history, takes no medications, and has no significant family history. He occasionally drinks alcohol but doesn't smoke. BP (112/74), pulse (88, regular). Physical examination reveals jugular venous distention, bibasilar crackles on lung auscultation, and 2+ pitting edema involving the lower extremities. Which of the following echocardiographic findings is most likely to be seen in this patient? a)Concentric left ventricle thickening with abnormal diastolic relaxation b)Dilated ventricles with abnormal systolic ventricular function c)High systolic pressure gradient across the aortic valve d)Regional wall motion abnormality e)Systolic anterior motion of the mitral valve

Answer: Choice B (Dilated ventricles with abnormal systolic ventricular function) -This patient's presentation with dyspnea, lower extremity edema, jugular venous distention, and pulmonary crackles is consistent with decompensated heart failure. In a young patient who develops heart failure following a symptomatic viral prodrome (e.g., fever, sinus congestion, myalgias), dilated cardiomyopathy (DCM) due to viral myocarditis should be suspected. A number of viruses have been implicated in causing DCM including coxsackievirus, adenovirus, and influenza virus. It is believed that an inadequate immune response to the viral infection allows virus to infect and persist inside cardiomyocytes, resulting in damage due to a direct cytotoxic effect or a destructive autoimmune reaction. -Depletion of cardiomyocytes with subsequent fibrosis causes weakened myocardial contractility (systolic dysfunction), volume overload, and ventricular dilation (eccentric hypertrophy) as surviving myocytes respond to increased wall stress. Other causes of DCM include genetic disease, pregnancy, infiltrative disease (e.g., late manifestation of amyloidosis or hemochromatosis), and drugs/toxins (e.g., anthracyclines, chronic alcohol disease) Choice A= Concentric LV hypertrophy with thickened walls and abnormal diastolic relaxation is characteristic of hypertensive heart disease due to long-standing uncontrolled hypertension. Initially, the LV cavity is reduced in size and there is normal systolic function; however, over time, high afterload pressures can cause progressive systolic dysfunction, LV cavity enlargement, and heart failure. Choice C= High systolic pressure gradient across the aortic valve is present in aortic stenosis; concentric LV hypertrophy can occur due to LV pressure overload. Aortic stenosis can develop in relatively young patients due to calcification of a bicuspid aortic valve; however, a crescendo-decrescendo systolic murmur is expected on examination Choice D= Regional wall motion abnormality is suggestive of ischemic heart disease (e.g., MI). Contractile function is impaired in the damaged portion of myocardium, resulting in systolic dysfunction with LV volume overload, eccentric hypertrophy, and LV cavity enlargement. Ischemic heart disease is unlikely in this young patient without significant risk factors for cardiovascular disease. Choice E= In patients with hypertrophic cardiomyopathy, systolic anterior motion of the mitral valve may worsen LV outflow tract obstruction. Clinical features of hypertrophic cardiomyopathy include exercise-induced syncope (due to outflow obstruction) and sudden cardia death (due to ventricular arrhythmia) in young athletes

-Age: 32 years man -Comes to the office due to progressive dyspnea, and chest discomfort. He has no prior medical problems but has mild respiratory illness 2 weeks earlier that resolved spontaneously. -His father has a history of MI. Echocardiogram shows pericardial fluid accumulation with late diastolic collapse of the right atrium. Which of the following physical examination findings is most likely to be seen in this patient? a)Beat-to-beat variation in pulse amplitude b)Drop in pulse amplitude during inspiration c)Pulse with 2 distinct peaks d)Rapidly rising pulse with high amplitude e)Slow-rising low-amplitude pulse

Answer: Choice B (Drop in pulse amplitude during inspiration) -This patient has a pericardial effusion with associated cardiac tamponade. Classic signs of cardiac tamponade include hypotension, elevated jugular venous pressure, and muffled heart sounds (Beck triad). Echocardiography will show pericardial fluid accumulation with late diastolic collapse of the right atrium (due to displacement of pericardial fluid with ventricular expansion). Pulsus paradoxus is an important physical finding in cardiac tamponade and refers to an exaggerated drop in systolic BP (>10mmHg) during inspiration. Fluid accumulation in the pericardium causes limitation of right ventricular expansion that is exacerbated during inspiration by the increase in venous return. This leads to bowing of the interventricular septum toward the left ventricle, decreasing LV end-diastolic volume and stroke volume. The result is decreased systolic pulse pressure during inspiration Choice A= Pulsus alternans refers to the beat-to-beat variation in pulse amplitude due to a change in systolic BP; it is most commonly seen in patient with severe LV dysfunction. Electrical alternans can be seen in very large pericardial effusion or cardiac tamponade and refers to a beat-to-beat variation in the QRS complex amplitude on ECG due to the swinging motion of the heart in the pericardial fluid Choice C= A dicrotic pulse is a pulse with 2-distinct peaks (one during systole and the other during diastole) due to accentuated diastolic dicrotic wave after the dicrotic notch. This finding typically occurs in patients with severe systolic dysfunction and high-systemic arterial resistance Choice D= A hyperkinetic pulse is a rapidly rising pulse with high amplitude due to the rapid ejection of a large stroke volume against a decreased afterload. This can occur with aortic regurgitation and high-output conditions (e.g., thyrotoxicosis, arterio-venous fistula) Choice E= Fixed LV outflow tract obstruction (e.g., valvular aortic stenosis) can cause pulsus parvus et tardus, which is palpable as a slow-rising amplitude pulse due to diminished stroke volume (pulsus parvus) and prolonged LV ejection time (pulsus tardus)

-Age: 32 years woman -Recruited for a research study. She is evaluated in the hospital as part of the study protocol. Her medical history: uncomplicated appendectomy, and medications include oral contraceptive pills and a multivitamin. -During the evaluation, increases in heart rate and cardiac output are observed, and an arterial blood gas study taken at the same time shows normal partial pressures of arterial oxygen (PaO2) and carbon dioxide (PaCO2). These changes are most likely part of an integrated response to which of the following? a)Carotid sinus massage b)Exercise c)High altitude exposure d)Panic attack e)Pulmonary artery occlusion

Answer: Choice B (Exercise) -This patient with increased HR, cardiac output, and unchanged partial pressures of PaO2 and PaCO2 is likely undergoing an exercise test. HR and cardiac output increase to meet increased tissue oxygen demands, and the respiratory rate increases to eliminate the additional carbon dioxide produced. Changes in the partial pressures of oxygen and CO2 during exercise occur predominantly in the venous blood as muscle tissue extracts a higher portion of oxygen and produces greater amounts of carbon dioxide. -The PaO2 and PaCO2 remain relatively close to resting values at most levels of exercise due to a number of adaptations in the lungs and pulmonary circulation, such as hyperventilation and improve ventilation-perfusion (V/Q) matching Choice A= Carotid sinus massage causes reflex parasympathetic stimulation of the SA node, atrial myocytes, and AV node, resulting in a transient decrease in HR and cardiac output Choice C= High altitude stimulation is associated with lower ambient oxygen pressures, consequently, heart rate and cardiac output increase in order to improve oxygen delivery to tissues. The lower oxygen pressures cause hypoxemia and result in hypoxic stimulation of respiration, causing hyperventilation and decreased PaCO2 Choice D= A panic attack can increase HR and cardiac output via sympathetic stimulation of the heart; however, there is usually associated hyperventilation without a significant increase in muscle activity and therefore a fall in the PaCO2 Choice E= Pulmonary artery occlusion as seen in pulmonary embolism results in increased pulmonary vascular resistance, right ventricular strain, and frequently decreased LV stroke volume. Reflex tachycardia can result to maintain (not increase) cardiac output. Additionally, PE often causes hypoxemia (low PaO2) due to acute V/Q mismatching in the affected lung

-Age: 34 years woman -Evaluated in the ED due to 1-day of chest pain and left shoulder pain. The patient describes the pain as a constant and worse in certain positions. A month ago, she was treated for a skin rash and joint pain -History: unremarkable; lifetime nonsmoker; her father has hypertension and had a stroke at age 64. Temp. (37.5C), BP (122/70), pulse (97, regular). -A triphasic scratchy sound is heard over the left sternal border with the patient sitting up. The lungs are clear on auscultation. ECG shows sinus rhythm with ST-segment elevation in all leads except aVR and V1. Diagnosis? a)Eosinophilic myocarditis b)Fibrinous pericarditis c)Hemorrhagic pericarditis d)Nonbacterial endocarditis e)Purulent pericarditis

Answer: Choice B (Fibrinous pericarditis) -This patient with pleuritic-type chest pain radiating to the shoulder and a recent history of skin rash and joint pain likely has acute fibrinous pericarditis due to SLE. Fibrinous pericarditis is the most common type of pericarditis and consists of pericardial inflammation with serous fluid and fibrin-containing exudate in the pericardial space -Patients typically have pleuritic (sharp, worse with breathing or movement) anterior chest pain that can radiate to the left shoulder or posteriorly to the bilateral scapulae. Fibrin deposition causes roughening of the visceral and parietal pericardium, often heard as a triphasic friction rub (occurring during atrial systole, ventricular systole, and early ventricular diastole) on cardiac auscultation; however, the rub can be absent if significant pericardial effusion is present. ECG typically shows diffuse ST elevation due to inflammation of the ventricular myocardium -Other causes of fibrinous pericarditis include viral infection, MI, uremia, and other rheumatologic disease (e.g., RA). If acute fibrinous pericarditis goes without treatment, chronic constrictive pericarditis can develop in some patients Choice A= Eosinophilic myocarditis can occur with malignancy, parasitic infection, or drug hypersensitivity and often presents with symptoms of heart failure. A friction rub on cardiac auscultation is not expected Choices C and E= Hemorrhagic and purulent pericarditis are both less common than fibrinous pericarditis. Hemorrhagic pericarditis consists of blood mixed with fibrinous exudate; it most commonly occurs in the setting of malignancy but may also occur due to TB, following cardiac surgery, or in patients with underlying coagulopathy. Purulent pericarditis results from active bacterial infection (e.g., Staphylococcus aureus, Streptococcus spp.) in the pericardial space, which may occur due to hematogenous spread or direct extension of pneumonia or endocarditis Choice D= Nonbacterial endocarditis can occur in SLE and involves deposition of sterile, platelet-rich thrombi on the mitral or aortic valve. It would not explain a friction rub or diffuse ST elevation on ECG

-Age: 65 years man -Reports multiple episodes of lightheadedness while buttoning a tight shirt collar. During 2 episodes, he passed out briefly but sustained no injuries. His BP (was 70/40), pulse (was 45) during one of the episodes. Past medical history: hypertension, diet-controlled diabetes -The patient is a lifetime nonsmoker and drinks alcohol on social occasions. On physical examination, his BP (125/72), pulse (76) without orthostatic changes -Stimulation of afferent sensory fibers in which of the following nerves is most likely responsible for this patient's symptoms? a)Accessory b)Glossopharyngeal c)Hypoglossal d)Trigeminal e)Vagus

Answer: Choice B (Glossopharyngeal) -The patient's history is suggestive of carotid sinus hypersensitivity, triggered by pressure on the carotid sinus by a tight shirt collar. The carotid sinus baroreceptors are important in BP control and use arterial wall stretch as an indicator of systemic BP. -The carotid sinus is a dilation of the internal carotid artery located just above the bifurcation of the common carotid artery. The carotid sinus reflex has an afferent limb that arises from the baroreceptors in the carotid sinus and travels to the medullary centers via the Hering nerve, a branch of the glossopharyngeal nerve (CN IX). The efferent limb of the carotid sinus carries parasympathetic impulses via the vagus nerve (CN X). Carotid sinus pressure or massage stimulates the baroreceptors and increases the firing rate from the carotid sinus, leading to an increase in parasympathetic output and withdrawal of sympathetic output to the heart and peripheral vasculature. The result is decreased BP (via peripheral vasodilation) and decreased cardiac output (decreased contractility/stroke volume and HR). In sensitive individuals, this response can cause severe bradycardia, hypotension, and sometimes syncope Choices A, C and D= The accessory (CN XI), hypoglossal (CN XII), and trigeminal (CN V) nerves do not transmit baroreceptor signals. The accessory nerve controls the sternocleidomastoid and trapezius muscles, the hypoglossal nerve primarily controls the muscles of the tongue, and the trigeminal nerve mediates facial sensation and also controls the muscles of mastication (chewing) Choice E= The vagus nerve (CN X) acts as the afferent limb for the nerve fibers originating from the aortic arch baroreceptors and as an efferent limb for the carotid sinus reflex. However, a tight collar would stimulate carotid sinus (not aortic arch) baroreceptor afferents carried by the glossopharyngeal nerve

-Age: 25 years woman -Comes to the office with a 3-month history of fatigue, myalgias, arthralgias, and a 5-kg weight loss. She has also recently experienced leg pain with activity, which resolves with rest -The patient has no other medical conditions and takes no medications -Temp. (37.6C), BP (160/90 in the right arm, and 120/180 in the left). A bruit is heard over the left subclavian artery. Left radial pulse and dorsalis pedis pulses in both legs are weak. -The remainder of the exam shows no abnormalities. Labs: Hb (9.8g/dl), ESR (110mm/hr). Cause of patient's condition? a)Exaggerated vascular smooth muscle contraction b)Granulomatous inflammation of large arteries c)Lipid-rich deposits in large- and medium-sized arteries d)Necrotizing inflammation of medium-sized arteries e)Segmental inflammation of medium-sized arteries and veins

Answer: Choice B (Granulomatous inflammation of large arteries) -This patient with lower extremity claudication (i.e., exertional pain due to limited blood flow reserve) and constitutional symptoms has typical features of Takayasu arteritis (TA). TA is a chronic, large-artery vasculitis that primarily involve the aorta and its branches. TA predominantly affects women of reproductive age and is more common in Asians. -Arterial narrowing in TA can manifest as audible bruits, BP discrepancies, pulse deficits, and distal ulcerations. Characteristic findings on histopathology include: mononuclear infiltrates and granulomatous inflammation of the vascular media, leading to arterial wall thickening and occlusion. The pathologic abnormalities in TA closely resemble giant cell (temporal) arteritis; however, TA primarily affects younger individuals, whereas giant cell arteritis occurs almost exclusively in patients age >50 Choice A= Exaggerated vascular smooth muscle contraction (e.g., in response to cold or vibration) causes Raynaud phenomenon. Symptoms are typically acute and episodic and manifest as triphasic color change (pallor, cyanosis, erythema) in the distal extremities. This patient's progressive symptoms and proximal arterial findings are more consistent with TA Choice C= Atherosclerosis is characterized by lipid-rich deposits in large- and medium-sized arteries. It is more common in older patients with cardiovascular risk factors (e.g., smoking, hypertension, hyperlipidemia) but is rare in individuals as young as this patient. In addition, in contrast to this patient who has both upper and lower extremity abnormalities, upper extremity atherosclerosis is usually a late finding. Choice D= Necrotizing inflammation of medium-sized arteries is seen in polyarteritis nodosa, which is characterized by multisystem involvement (e.g., GN, skin lesions, neuropathy, bowel ischemia, myositis). This patient's bruits, BP discrepancies, and pulse deficits indicate a large artery disorder Choice E= Thromboangiitis obliterans (Buerger disease) is a segmental, thrombosing vasculitis of medium- and small-sized arteries, principally the tibial and radial arteries. Manifestations are most noticeable in the distal extremities (e.g., ulcerations), and the condition is almost always seen in association with heavy cigarette smoking

-Age: 82 years woman -Sent to the hospital from a nursing home after 2 days of fever, confusion, and lethargy. The patient has a history of mild dementia, hypertension, and osteoarthritis. -Temp. (38.8C), BP (90/60), pulse (116). She has mild suprapubic and right costovertebral angle tenderness. Her leukocyte count is 18,000/mm3, and urinalysis is consistent with acute cystitis. A femoral venous access is planned for administration of IV fluids and antibiotics. The femoral artery pulsation is palpable immediately below the inguinal ligament. Which of the following describes the optimal cannulation site for the femoral vein? a)Immediately lateral to the femoral artery b)Immediately medial to the femoral artery c)Midway between the iliac crest and the femoral artery d)Superior and medial to the pubic tubercle e)Superior to the inguinal ligament

Answer: Choice B (Immediately medial to the femoral artery) -Femoral vein catheter placement is usually done in patients with unavailable preferred access sites (e.g., subclavian, jugular vein) or those at increased risk of complications (e.g., emergency access, uncooperative patient). The femoral vein is the major deep vein of the lower extremity and passes through the femoral triangle beneath the inguinal ligament before entering the pelvis as the external iliac vein. The femoral triangle is a subfascial space in the upper thigh that is bordered by the inguinal ligament (superiorly), adductor longus muscle (medially), and sartorius muscle (laterally) -Within the femoral triangle lies (lateral to medial) the femoral nerve, femoral artery, femoral vein and deep inguinal nodes/lymphatic vessels. The femoral artery is located at the midinguinal point (midway between the pubic symphysis and anterior superior iliac spine), and the femoral vein lies immediately medial to the artery. Cannulation of the femoral vein should occur approximately 1cm below the inguinal ligament and 0.5-1cm medial to the femoral artery pulsation (Choices C, D and E) Choice A= The femoral nerve is approximately 1cm lateral to the femoral artery pulsation and could be damaged by needle insertion at this site

-Age: 72 years man -Comes to the office due to progressive shortness of breath. Over the past year, the patient has been unable to perform many of his usual outdoor activities, including afternoon walks and working in the yard. -During the last several weeks, he has frequently become easily fatigued and short of breath, and at night he needs several pillows to sleep comfortably. He has also had ankle swelling. The patient has had no chest pain, syncope, abdominal pain, or cough -He is a lifetime nonsmoker. Bibasilar crackles are heard on physical examination. Echocardiography reveals no valvular disease and a nondilated left ventricle with normal wall motion and left ventricular ejection fraction of 55%. Which of the following is most strongly associated with this patient's current condition? a)Glomerular hyperfiltration b)Increased left ventricular afterload c)Increased left ventricular compliance d)Left ventricular thrombus formation e)Right-to-left intracardiac shunting

Answer: Choice B (Increased LV afterload) -The patient has clinical signs and symptoms of decompensated heart failure (e.g., dyspnea, fatigue, orthopnea, lower extremity edema, bibasilar crackles); however, echocardiography shows a preserved LV ejection fraction (>50%) without chamber dilation. -Heart failure with preserved ejection fraction (HFpEF) develops due to diastolic dysfunction, which frequently occurs in the setting of prolonged systemic hypertension (i.e., increased afterload) -Overtime, increased pressure load causes the LV to undergo concentric hypertrophy with the addition of myocardial contractile fibers in parallel to increase contractile strength. The thickened LV walls become stiff and less compliant, leading to impaired diastolic relaxation and increased LV end-diastolic pressure (point 1). Gradually, the elevated pressure transmits back to the pulmonary veins and capillaries, resulting in pulmonary edema. There is also an eventual decline in LV end-diastolic volume (also indicated at point 1) leading to reduced cardiac output and further decompensation (stroke volume decreases despite preserved ejection fraction) Choice A= The decreased cardiac output in heart failure leads to reduced renal perfusion pressure and glomerular hypofiltration. To compensate, RAAS is activated, and angiotensin II constricts glomerular efferent arterioles in an effort to increase GFR back to normal. Choice C= Diastolic dysfunction involves a decrease in LV compliance. LV compliance may increase with eccentric hypertrophy, as occurs in response to LV volume overload (e.g., dilated cardiomyopathy) Choice D= LV thrombus typically results from localized stagnation of blood as can occur with an LV aneurysm or in the setting of a severely dilated cavity with systolic dysfunction (pic on next flashcard). It is not typically associated with diastolic dysfunction and HFpEF Choice E= Atrial and ventricular septal defects are often asymptomatic but can eventually lead to right-to-left intracardiac shunting (Eisenmenger syndrome), typically characterized by cyanosis. This patient's orthopnea, lower extremity edema, and bibasilar crackles suggesting pulmonary edema are more consistent with heart failure.

-Age: 17 years boy -Brought to the ED after passing out. The patient was standing behind the dugout watching his friends play baseball when he suddenly felt warm and light-headed. Then he remembers awakening of the ground. Bystanders reported that the patient was unresponsive for 2 mins. -He feels fatigued but does not have chest pain or shortness of breath. The patient has no significant medical history and does not use alcohol or illicit drugs. Temp. (36.7C), supine BP (105/60), standing BP (110/65), pulse (84), RR (16). Pulse oximetry is 100% on room air. -The patient is awake and alert. Pupils are equal and reactive. Heart and lung auscultation are normal. The abdomen is soft and nontender. Neurologic examination is normal. ECG is unremarkable. Which of the following is the most likely cause of this patient's syncope? a)Increased automaticity of the Purkinje fibers b)Increased parasympathetic output to the heart c)Increased sympathetic output to the small vessels d)Reentry arrhythmia using the atrioventricular node e)Systolic anterior motion of the mitral valve

Answer: Choice B (Increased parasympathetic output to the heart) This patient has experienced vasovagal syncope, a type of reflex syncope. Patients with vasovagal syncope typically have a trigger (e.g., prolonged standing) and experience a prodrome (e.g., warmth, light-headedness) that immediately precedes loss of consciousness. The episodes usually have a short duration (e.g., 1-2 mins) and typically involve a rapid recovery. When vasovagal syncope is repeatedly triggered by a certain activity (e.g., coughing, micturition), it is referred to as situational syncope Reflex syncope is the result of inappropriate autonomic output from the brainstem in response to signals relayed by various baroreceptors or mechanoreceptors. Vasovagal syncope is often triggered by cortical emotions that increase sensitivity to afferent signals, but it can also be triggered situationally by increased afferent transmission from pulmonary, GI, or genitourinary mechanoreceptors. Once these afferent signals are received by the brainstem cardioregulatory center, some combination of the following effector responses occurs: -Cardioinhibitory response: increased parasympathetic stimulation to the SA and AV nodes slows the HR. In addition, decreased sympathetic output to the myocardium decreases myocardial contractility -Vasodepressor response: Decreased sympathetic output to vascular smooth muscle causes vasodilation of venous capacitance veins (i.e., decreased cardiac preload) and systemic arterioles (i.e., decreased perfusion pressure) (Choice C) The result of these responses is a brief but profound decrease in cardiac output that leads to hypotension, decreased cerebral perfusion and syncope Choices A and D= Cardiac syncope can occur due to the marked reduction in cardiac output that results from arrhythmia (e.g., ventricular tachycardia, AV nodal reentrant tachycardia). Increased automaticity of the Purkinje fibers can trigger ventricular tachycardia, but this usually affects patients with ischemic or structural heart disease. AV nodal reentrant tachycardia usually affects young patients, but rarely reduces cardiac output enough to cause syncope. In addition, cardiac syncope of any etiology would not be associated with a vagal prodrome Choice E= Severe LV outflow tract obstruction (e.g., aortic stenosis, hypertrophic cardiomyopathy (HCM)) can cause cardiac syncope. Systolic anterior motion of the mitral valve can worsen outflow obstruction in patients with HCM, but syncope resulting from LV outflow obstruction is typically exertional, and this patient has no systolic murmur to suggest HCM

-Age: 62 years man -Long history of hypertension comes to the clinic for a routine physical examination. BP (150/90), HR (74, regular). Cardiac auscultation reveals a low-frequency, presystolic sound that immediately precedes the S1 heart sound and is best heard during expiration when the patient is lying on his left side. -Chest x-ray reveals extensive calcification around the mitral and aortic valves. Which of the following is the most likely explanation for the additional heart sound? a)Increased flow velocity through the aortic valve b)Increased stiffness of the left ventricular wall c)Rapid passive filling of the ventricles d)Restricted motion of the aortic valve cusps e)Systolic anterior motion of the mitral valve

Answer: Choice B (Increased stiffness of the left ventricular wall) -The patient has a fourth heart sound (S4) secondary to extensive left ventricular hypertrophy from long-standing hypertension (hypertensive heart disease). The calcifications revealed on chest x-ray likely represent degenerative mitral and aortic valve calcification, which is associated with systemic hypertension -An S4 is a sign of diastolic dysfunction. It occurs due to a sudden rise in end-diastolic ventricular pressure caused by atrial contraction against a ventricle that has reached its elastic limit. An S4 may be present in any condition that causes reduced ventricular compliance (e.g., hypertensive heart disease, aortic stenosis, hypertrophic cardiomyopathy). It is heard as a low-frequency, late diastolic sound that occurs just prior to the first heart sound (S1) after the onset of the P wave on ECG (coinciding with the active phase of ventricular filling) -A left-sided S4 is best heard with the bell of the stethoscope over the cardiac apex with the patient in the left lateral decubitus position; it will intensify during expiration due to increased blood flow from the lungs to the left atrium Choice A= Increased flow velocity through the aortic valve would produce a crescendo-decrescendo ejection murmur during systole (after S1 and before S2) similar to the murmur of aortic stenosis Choice C= Sudden cessation of passive ventricular filling causes a diastolic third heart sound (S3) that is heard shortly after S2. It is often heard in patients with systolic heart failure (e.g., dilated cardiomyopathy), ventricular volume overload (e.g., mitral regurgitation), and in high-output cardiac states (e.g., pregnancy, thyrotoxicosis) Choice D= Restricted motion of the aortic valve cusps (due to scarring/thickening) can lead to aortic stenosis and/or aortic regurgitation and may be associated with a high-frequency aortic ejection click. The ejection click is heard after S1 just before the systole murmur of aortic stenosis; the diastolic murmur of aortic regurgitation begins immediately after S2. Choice E= Systolic anterior motion (SAM) of the mitral valve occurs in patients with hypertrophic cardiomyopathy can cause eccentric mitral regurgitation and exacerbation of LV outflow tract obstruction. However, these would result in systolic murmurs (after S1 and before S2)

-Age: 68 years man -History of permanent atrial fibrillation comes to the office for follow-up. He has been having symptoms due to ineffective ventricular rate control despite aggressive medical therapy. On examination, the patient's HR (125, irregular). ECG shows atrial fibrillation with rapid ventricular response. A catheter-based radiofrequency ablation of the atrioventricular node with placement of a permanent ventricular pacemaker is planned. The ablation tip should be positioned at which of the following locations? a)Between the tricuspid valve and the inferior vena cava opening b)Interatrial septum near the opening of the coronary sinus c)Left atrium near the opening of the pulmonary veins d)The posterior border of the fossa ovalis e)Upper part of crista terminalis near the superior vena cava opening

Answer: Choice B (Interatrial septum near the opening of the coronary sinus) -The AV node controls the rate at which atrial impulses are conducted to the ventricles. In patients with atrial fibrillation, continuous stimulation of the AV node can sometimes lead to a rapid ventricular rate, which can cause hemodynamic instability and (if persistent) tachycardia-induced cardiomyopathy. In order to prevent these complications, patients with a rapid ventricular response sometimes undergo radiofrequency ablation of the AV node. The AV node is located on the endocardial surface of the right atrium, near the insertion of the septal leaflet of the tricuspid valve and the orifice of the coronary sinus Choices A and D= The isthmus between the IVC and the tricuspid annulus is the site of radiofrequency ablation for atrial flutter. The fossa ovalis is the name given to the foramen ovale (fetal opening between the right and left atria) once it is closed. However, neither option refers to the location of the AV node Choice C= The pulmonary vein ostia are often the origination site of the aberrant electrical activity that triggers atrial fibrillation, and catheter ablation of these trigger sites (pulmonary vein isolation) can be used to prevent recurrent atrial fibrillation. However, this patient has already developed permanent atrial fibrillation and is undergoing an ablation procedure involving the AV node Choice E= The SA node is located in the upper anterior right atrium at the opening of the SVC

-Age: 45 years man -Comes to clinic due to frequent episodes of palpitations accompanied by dizziness, fatigue, and shortness of breath. Prolonged ECG monitoring identifies episodes of atrial fibrillation associated with a rapid ventricular response rate. -A radiofrequency ablation procedure is planned. The access site is the right femoral vein. The ablation catheter is advanced to the left atrium where radiofrequency energy is used to eliminate an ectopic focus of abnormal electrical activity -During the procedure, the catheter most likely passes through which of the following structures? a)Aortic valve b)Interatrial septum c)Interventricular septum d)Pulmonic valve e)Tricuspid valve

Answer: Choice B (Interatrial septum) -A venous catheter traveling from the femoral vein to the heart passes through the iliac vein and inferior vena cava to reach the right atrium. Once in the right atrium, structures within the right side of the heart and the pulmonary arteries are readily accessible. However, because the pulmonary capillaries are far too small to pass through, the left side of the heart must be accessed by traversing the interatrial septum. The interatrial septum is traversed at the site of the foramen ovale, which in adults is typically covered by a thin membrane of fibrous tissue that can be easily punctured -Entry into the left atrium allows for direct measurement of left atrial pressure (rather than an estimate via PCWP) and for access to arrhythmogenic foci that may be located on the left atrial myocardium or the pulmonary veins. Following the procedure, the small atrial septal defect created by the catheter typically closes spontaneously. Choice A= Arterial catheterization of the left side of the heart typically starts in the femoral or radial artery and proceeds to the ascending aorta where pressure can be measured, or dye can be placed into the coronary arteries to visualize atherosclerotic obstruction (i.e., coronary angiography). In addition, the aortic valve may be crossed to measure left ventricular pressure. However, retrograde crossing of the structurally complex mitral valve is highly difficult, and the LA is not accessed via this route Choice C= The interventricular septum is not traversed during heart catheterization because the myocardium is thick and difficult to puncture, and high LV pressure would likely prevent spontaneous closure of the VSD that is created Choices D and E= The tricuspid valve is crossed during venous catheterization to access the RV, and the pulmonic valve is subsequently crossed to access the pulmonary arteries. However, because the catheter cannot pass through the pulmonary capillaries, the LA can't be accessed via this route

-In experiments, transient myocardial ischemia causes myocardial cells to increase in size. This effect is due in part to which of the following? a)Intracellular K+ accumulation b)Intracellular Ca2+ accumulation c)High cellular HCO3- content d)Cascade protein phosphorylation e)Net cellular solute loss

Answer: Choice B (Intracellular Ca2+ accumulation) -When blood flow cannot meet myocardial demands, cardiac myocytes transition from aerobic to anaerobic metabolism. However, anaerobic metabolism cannot maintain proper intracellular ATP levels, and ADP, AMP, and adenosine accumulate -Without ATP, the membrane Na+/K+ ATPases and the sarcoplasmic reticulum Ca2+-ATPases fail, leading to increase intracellular Na+ and Ca2+ and increased intramitochondrial Ca2+ concentrations. These increased concentrations attract free water, causing cellular and mitochondrial swelling. Failure of the sarcoplasmic reticulum to resequester Ca2+ leads to cessation of contraction within ischemic zones of myocardium Choice A= The cell membrane Na+/K+ ATPase pump normally brings K+ into the cell in exchange for Na+. Thus, in ischemic tissue, intracellular K+ is decreased Choice C= Cellular HCO3- is not elevated in cardiac ischemia. Anaerobic metabolism leads to lactic acid production and a decrease in pH. Tissue CO2, the conjugate acid of HCO3-, is thus elevated in ischemic myocardium Choice D= Protein phosphorylation cascades play a role in a variety of processes including 2nd messenger signaling and regulation of apoptosis. The cell's response to ischemia does involve initiating certain metabolic processes through protein phosphorylation, but this does not cause cell swelling Choice E= Ischemia causes a net solute gain within cardiac myocytes. Elevated concentrations of Na+ and Ca2+ draw free water into the cells, causing edema

-Age: 64 years man -Presents to the office with chest discomfort and mild shortness of breath that started one hour ago after an argument with his wide. Long history of hypertension and diabetes -BP (180/100mmHg), HR (100). An EKG taken in the ED shows no signs of acute ischemia. Cardiac auscultation in the left decubitus position on full expiration reveals a presystolic sound. Which of the following best explains this physical examination finding? a)Papillary muscle dysfunction b)Left ventricular hypertrophy c)Increased left ventricular preload d)Aortic regurgitation e)Mitral leaflet fusion and fibrosis

Answer: Choice B (LV hypertrophy) -An S4 is an isolated presystolic (late diastolic) sound heard just prior to S1. An S4 is caused by left atrial contraction that forces atrial blood into a poorly compliant LV that is nearly filled to capacity. When blood is forced into the ventricle by the atrium under these conditions it generates a large, rapid increase in LV pressure thereby producing a sound possibly resulting from rapid deceleration of the forced blood or from impact of the filling ventricle with the chest wall. LV hypertrophy and other causes of diastolic heart failure therefore favor generation of an S4 sound. This patient's LV hypertrophy is likely secondary to chronic hypertension Choice A= Papillary muscle dysfunction may cause incomplete closure of the AV valves resulting in a murmur audible during systole Choice C= An increase in LV preload is typically accommodated by stretching in a normally compliant ventricle and does not result in characteristic heart sounds Choice D= Aortic regurgitation causes a decrescendo early and mid-diastolic murmur Choice E= Mitral leaflet fusion and fibrosis may cause mitral stenosis resulting a mid-diastolic murmur with possible late diastolic (pre-systolic) accentuation due to atrial contraction

-Age: 56 years woman -Evaluated for fatigue and dyspnea on exertion. Past medical history: hypertension, systemic sclerosis. BP (135/80), pulse (68). Cardiac examination reveals loud second heart sounds with no murmurs. Lungs are clear to auscultation. Further evaluation with a catheterization procedure is performed, during which a balloon-tipped catheter is advanced into the pulmonary artery. A branch of the pulmonary artery is occluded by the balloon, and the presence beyond the point of occlusion is measured. The pressure reading from the procedure most likely corresponds to which of the following pressures? a)Intrapleural pressure b)Left atrial pressure c)Mean airway pressure d)Mean arterial pressure e)Pulse pressure f)Right atrial pressure g)Right ventricular systolic pressure

Answer: Choice B (Left atrial pressure) -Pulmonary artery catheters (PACs; also called Swan-Ganz or right heart catheters) are used to diagnose pulmonary hypertension and occasionally for management of critically ill patients. During pulmonary artery catheterization, the balloon at the distal tip of the catheter is inflated, and the catheter is advanced forward through the right atrium, right ventricle and pulmonary artery and finally into a branch of the pulmonary artery. Once lodged in a pulmonary artery branch, the inflated balloon obstructs forward blood flow, creating a continuous static column of blood between catheter tip and left atrium. -The pressure measured at the catheter tip at this time is called the pulmonary artery occlusion pressure (PAOP, or pulmonary capillary wedge pressure (PCWP)) and closely reflects left atrial and left ventricular end-diastolic pressures Choices A and C= Direct measurement of intrapleural pressure (pleural manometry) can be obtained by placing a catheter in the pleural space during thoracocentesis, whereas main airway pressure is measured during mechanical ventilation. PACs do not measure intrapleural or mean airway pressure Choice D= Mean arterial pressure is measured by placing an arterial catheter directly in central arterial circulation (common femoral or radial artery) and reflects the average aortic pressure (perfusion pressure) Choice E= Pulse pressure refers to the difference between systemic arterial systolic and diastolic blood pressure Choices F and G= Right atrial and right ventricular systolic pressures can be measured directly by PACs by placing the catheter in the respective chambers

-Age: 51 years man -Brought to the ED due to chest tightness that started 30 mins prior to arrival. His chest discomfort is associated with shortness of breath and nausea. -The patient was shoveling snow off his driveway when his symptoms began. He has a history of hypertension and type 2 diabetes. Initial ECG shows ST elevation in leads I and aVL. Cardiac enzymes are elevated. Emergent cardiac catheterization in this patient will most likely show occlusion of which of the following arteries? a)Distal left anterior descending artery b)Left circumflex artery c)Left main coronary artery d)Proximal left anterior descending artery e)Right coronary artery

Answer: Choice B (Left circumflex artery) -This patient with ST elevations in leads I and aVL most likely has an acute lateral MI due to occlusion of the left circumflex artery. ECGs are useful in the diagnosis of MI and may help localize the area of infarction. ST elevations typically represent acute MI, whereas prominent Q waves are suggestive of old MI -Leads I and aVL correspond to the lateral limb leads on ECG; therefore, ST elevation or Q waves in these leads are indicative of infarction involving the lateral aspect of the LV. Because the chest leads V5-V6 are also laterally placed, they may also show ST elevation during a lateral infarction. The lateral aspect of the LV is supplied by the left circumflex artery, which originates from the left main coronary artery Choices A and D= The left anterior descending artery (LAD) primarily supplies the anterior aspect of the LV and interventricular septum (septal branches), which corresponds to the anterior chest leads (V1-V4). Proximal occlusion of the LAD may involve all of these leads; however, distal LAD occlusion typically spares the septal leads (V1-V2) Choice C= The left main coronary artery gives rise to the LAD and left circumflex arteries. Therefore, left main coronary artery occlusion typically results in anterolateral infarction, which corresponds to ST elevation in the anterior (e.g., V1-V4), and lateral (V5-V6, aVL, and I) leads. Choice E= The right coronary artery typically supplies the RV and inferior aspect of the LV. Occlusion of this vessel therefore results in inferior MI, which corresponds to ST elevation in the inferior leads (II, III and aVF)

-Age: 66 years man -Comes to the hospital due to sudden-onset chest pain and dyspnea. The patient has a history of asthma and GERD but says his current symptoms feel different than what he usually experiences. An ECG is consistent with ST-elevation MI, and an emergent cardiac catheterization is performed. Evaluation of the left and right coronary arteries reveals a left-dominant circulation. A stenotic region is identified in one of the coronary vessels just before the origin of the artery supplying the atrioventricular node. Which of the following arteries is most likely affected? a)Anterior interventricular artery b)Left circumflex artery c)Left diagonal artery d)Right coronary artery e)Right marginal artery

Answer: Choice B (Left circumflex artery) Coronary dominance is determined by the coronary artery that supplies blood to the posterior descending artery (PDA (or posterior interventricular artery)). The PDA originates from one of the following: -Right coronary artery in approximately 70-80% of the population (right dominant) -Left circumflex artery in approximately 5-10% of the population (left dominant) -Both RCA and LCx in 10-20% (codominant) The AV nodal artery most often arises from the dominant coronary artery. This patient has left-dominant coronary circulation; therefore, his atherosclerotic lesion is most likely in the left circumflex artery. Involvement of the AV nodal artery during MI can cause varying degrees of AV block Choice A= The left anterior descending artery travels in the anterior interventricular groove and supplies the anterior part of the septum and the anterior wall of the LV. It does not supply the AV node Choice C= The diagonal arteries arise from the left anterior descending artery and supply the anterolateral wall of the LV Choice D= The RCA gives rise to the AV nodal artery in patients with right-dominant or codominant circulation. This patient has left-dominant circulation, so the AV node is most likely supplied by the left circumflex artery Choice E= The right marginal arteries arise from the right coronary artery and supply blood to the free wall of the RV

-Age: 56 years man -Admitted to a tertiary care center after being involved in a high-speed motor vehicle accident. The patient was thrown from his vehicle and suffered severe head trauma along with multiple fractures involving his ribs and extremities -Despite aggressive treatment, he dies the following day from massive cerebral edema and brainstem herniation. Autopsy shows the following heart findings: -Left atrium: enlarged -Left ventricular myocardial mass: increased -Left ventricular wall thickness: increased -The structural changes observed in this patient's heart are most likely associated with? a)Atrial septal defect b)Long-standing hypertension c)Mitral insufficiency d)Mitral stenosis e)Normal aging f)Recent myocardial infarction g)Wolff-Parkinson White syndrome

Answer: Choice B (Long-standing hypertension) -The autopsy findings of this patient's LV are consistent with concentric hypertrophy most likely due to chronic hypertension -LV hypertrophy is generally defined as an increase in left ventricular mass due to increased wall thickness (concentric) or cavity size (eccentric). Concentric hypertrophy is characterized by uniform thickening of the ventricular wall with the outer dimensions of the ventricle remaining almost unchanged, resulting in a narrowed ventricular cavity size. It is due to chronic elevation of ventricular pressures during systole, which is usually caused by long-standing hypertension or aortic stenosis (increased LV afterload). Patients may develop diastolic dysfunction with left atrial enlargement and congestive heart failure due to impaired ventricular compliance and filling Choice A= An atrial septal defect causes a shunting of blood flow from the left atrium to the right atrium. This typically results in volume overload of the RV, producing RA enlargement and eccentric (dilated) RV hypertrophy Choice C= Mitral insufficiency causes LV volume overload as regurgitant flow through the mitral valve during systole increases the amount of blood returning to the LV during diastole. The net structural consequence of mitral insufficiency is enlargement of the LV cavity and eccentric hypertrophy Choice D= mitral stenosis leads to left atrial enlargement due to elevated pressure in the LA. The LV cavity is typically small/normal in size with normal wall thickness due to restriction of blood flow across the mitral valve Choice E= Aging is associated with a decrease in LV chamber size, predominantly through shortening of the apex-to-base dimension accompanied by a leftward bowing of the lower interventricular (sigmoid) septum. The aortic valve may also become calcified, resulting in aortic stenosis; however, structural cardiac changes due to aging are generally not prominent before age 65 Choice F= The ventricular remodeling that occurs after an acute MI involves expansion, thinning, and fibrous healing of the infarcted zone of the myocardium. Regional dysfunction of the infarcted myocardium causes volume overload for the remaining viable myocardium. The net result is usually eccentric hypertrophy, with enlargement of the LV cavity Choice G= Patients with WPW syndrome have an accessory pathway that bypasses the AV node and directly connects the atria and ventricles. This results in ventricular preexcitation and the development of tachyarrhythmias. The LV usually appears grossly normal

-Age: 44 years man -With progressive dyspnea is diagnosed with dilated cardiomyopathy. Despite optimal medical therapy, he continues to have symptoms and disease progression is noted. He undergoes cardiac transplantation after a suitable donor becomes available -Permission is obtained from the patient to study his diseased heart for intracellular calcium regulation. Microelectrodes placed into cardiac muscle cells detect a rapid decrease in cytoplasmic calcium level immediately preceding relaxation. -Which of the following proteins is most likely responsible for the observed change in electrolyte levels? a)Calmodulin b)Na+/Ca2+ exchanger c)Ryanodine receptors d)Troponin C e)Voltage-dependent calcium channels

Answer: Choice B (Na+/Ca2+ exchanger) -Intracellular calcium regulation plays an important role in excitation-contraction coupling. Voltage-dependent calcium channels (L-type) are activated during phase 2 of cardiac action potential (depolarization) and permit influx of calcium into the cardiac myocytes (Choice E). This initial calcium influx is sensed by the ryanodine receptors in the sarcoplasmic reticulum, which triggers further release of Ca2+ (calcium-induced calcium release) into the cytoplasm, thereby increasing intracellular calcium concentration 100-fold (Choice C). The calcium released from the sarcoplasmic reticulum diffuses through the myofilament network and binds to troponin C (Choice D). Tropomyosin is then moved out of the way so that actin and myosin can interact, leading to muscle contraction -The final stage of excitation-contraction coupling is myocyte relaxation, which occurs subsequent to calcium efflux from the cytoplasm. Intracellular calcium is removed primarily via an Na+/Ca2+ exchange pump (NCX) and sarcoplasmic reticulum Ca2+ ATPase pump (SERCA). NCX uses the large extracellular Na+ concentration gradient to help pump Ca2+ out of the cell and, in the process, removes one intracellular Ca2+ in exchange for 3 extracellular Na+ ions. In contrast, SERCA is a Ca2+-ATPase pump that actively transfers Ca2+ from the cytosol to the lumen of the sarcoplasmic reticulum at the expense of ATP hydrolysis Choice A= Calmodulin (calcium modulated protein) is a calcium-binding messenger protein that is present in all cells and mediates many of the regulatory effects of Ca2+. Calmodulin is important for excitation-contraction coupling in smooth muscle cells, which lack troponin, unlike cardiac and skeletal muscles. In cardiac muscle, calmodulin is not directly involved in excitation-contraction coupling; rather, it helps regulate intracellular calcium activity and transcription factor signaling

-Age: 71 years man -Comes to the hospital due to 3 hours of persistent retrosternal chest pain and dyspnea. After prompt evaluation, he is diagnosed with acute MI and undergoes cardiac catheterization. During cannulation of the right common femoral artery, the arterial wall is penetrated superior to the right inguinal ligament. -Percutaneous coronary intervention is then performed, after which firm pressure is applied to achieve hemostasis. Soon after the procedure, the patient becomes cold, clammy, and hypotensive. Physical examination shows a 5cm region of ecchymosis surrounding the femoral puncture site. Internal bleeding is suspected. Which of the following is the most likely location of the blood collection? a)Pelvic cavity b)Retroperitoneal space c)Right paracolic gutter d)Subcutaneous tissue e)Thigh muscles

Answer: Choice B (Retroperitoneal space) -Vascular access during cardiac catheterization is typically obtained through either the common femoral artery or radial artery. The common femoral artery is the continuation of the external iliac artery as it crosses the inguinal ligament. Arterial puncture above the inguinal ligament increases the risk of retroperitoneal hemorrhage, as this portion of the vessel lies directly inferior to the peritoneum. Accidental puncture of the posterior wall can cause blood to track along the loose connective tissue surrounding the vessel and accumulate within the interfascial planes of the retroperitoneum -Bleeding in the retroperitoneal space cannot be controlled with external compression and can lead to life-threatening hemorrhage. Affected patients typically develop hemodynamic instability with significant hypotension, drop in hemoglobin, and ipsilateral flank pain Choice A= Bleeding into the pelvic cavity is frequently due to trauma (e.g., pelvic fracture) or gynecologic hemorrhage (e.g., ruptured ectopic pregnancy). Injury to the common femoral artery is less likely to cause intraperitoneal bleeding as the external iliac artery courses underneath the peritoneum Choice C= The right paracolic gutter is a peritoneal recess found between the ascending colon and abdominal wall. Fluid (bile, pus, or blood) typically accumulates there from pathology involving the GI organs, particularly the appendix and gallbladder Choice D= Subcutaneous tissue hemorrhage from an arterial source typically presents with a large area of ecchymosis and a palpable hematoma. Development of significant hypotension is less likely with subcutaneous bleeding, as the potential space for blood accumulation is smaller than with peritoneal or retroperitoneal bleeding Choice E= Hemorrhage into the thigh muscles would present with significant pain and edema in the thigh and possible lead to compartment syndrome (severe pain, absent distal pulses)

-Age: 72 years man -Long-standing dyspnea was seen in the clinic after experiencing an episode of syncope. Physical examination showed weak and slowly rising arterial pulses. Cardiac auscultation showed a harsh midsystolic murmur best heard at the 2nd right intercostal space with decreased intensity of the 2nd heart sound. -Echocardiogram and electrocardiogram confirmed the diagnosis of severe aortic stenosis. 2 months later, the patient comes to the ED with palpitations and increased shortness of breath. BP (90/60), HR (130, irregularly irregular rhythm). Electrocardiogram shows new-onset atrial fibrillation without significant ST-segment or T-wave changes. Chest x-ray shows bilateral pulmonary edema. Which of the following hemodynamic changes is most likely associated with this patient's current presentation? a)Insidious right ventricular failure b)Sudden decrease in left ventricular preload c)Sudden decrease in left ventricular systolic function d)Sudden increase in left ventricular afterload e)Sudden increase in left ventricular filling

Answer: Choice B (Sudden decrease in left ventricular preload) -This patient's sudden-onset heart failure was most likely precipitated by acute atrial fibrillation, a condition that occurs in up to 10% of patients with severe aortic stenosis (AS). Patients with severe AS already have reduced cardiac output due to significant valvular obstruction, which can be exacerbated by the sudden loss of normal atrial contraction that contributes to ventricular filling. Atrial contraction is especially important for these patients as many have concentric LV hypertrophy and therefore reduced LV compliance. As a result, they become dependent on atrial contraction to maintain adequate LV filling. Without atrial contraction, LV preload can decrease to the point of producing severe hypotension. In addition, loss of the atrial kick can result in significantly increased mean pulmonary venous pressure due to buildup of blood in the LA and pulmonary veins, leading to acute pulmonary edema. As a result, cardioversion is indicated for acute atrial fibrillation in patients with severe AS Choice A= Cardiogenic pulmonary edema is characteristic of left, not right, heart failure. When the forward pumping function of the left heart is impaired, pressure can build up in the left atrium. The rise in pressure is transmitted to the pulmonary veins and capillaries, resulting in increased fluid transudation into the alveoli and pulmonary interstitium Choices C and E= This patient has no evidence of myocardial ischemia on electrocardiogram, meaning that LV systolic function is most likely unchanged. Under these circumstances, an increase in LV filling (preload) would increase cardiac output and mean arterial pressure due to Frank-Starling mechanism. This patient's decreased cardiac output is most likely due to decreased LV preload Choice D= In a patient with AS, LV afterload is determined by the mean systolic BP and the degree of transvalvular obstruction. In degenerative calcific AS and most other forms of adult AS, the transvalvular obstruction gradually increases over years to decades, making an acute increase in LV afterload unlikely. In addition, this patient's acute hypotension would actually cause a reduction in afterload

-Physiologists conducting research on the electrical properties of the heart measure action potential conduction velocity at 4 different points within normal cardiac tissue. The results, expressed in terms of speed of conduction (meters per second), are as follows: -Points 1- 0.05m/sec -Point 2- 0.3m/sec -Point 3- 1.1m/sec -Point 4- 2.2m/sec -From the following list of locations, which most likely corresponds to the order of points 1-2-3-4 (AV=atrioventricular)? a)Atrial muscle, ventricular muscle, Purkinje system, AV node b)AV node, Purkinje system, ventricular muscle, atrial muscle c)AV node, ventricular muscle, atrial muscle, Purkinje system d)Purkinje system, AV node, ventricular muscle, atrial muscle e)Ventricular muscle, AV node, Purkinje system, atrial muscle

Answer: Choice C (AV node, ventricular muscle, atrial muscle, Purkinje system) -Cardiac impulses normally originate in the SA node. SA node depolarization delivers an electrical impulse to the surrounding atrial myocardium, which carries the action potential to the AV node at a rate of 1.1m/sec. -Speed of conduction in the AV node is the slowest (0.05/msec), and his delay allows the ventricles to completely fill with blood during diastole. From the AV node, the action potential enters the His-Purkinje system. Impulses travel the fastest through the Purkinje fibers (2.2m/sec), which ensures that the ventricles contract in a bottom-up fashion (necessary for efficient propulsion of blood into the pulmonary artery and aorta). From the Purkinje fibers, the action potential is transmitted to the ventricular myocardium, where it travels at a rate of 0.3m/sec. -Points 1-2-3-4 are arranged in order of increasing conduction speed (not conduction time), as follows: -Point 1= AV node (0.05m/sec) -Point 2= ventricular muscle (0.3m/sec) -Point 3= atrial muscle (1.1m/sec) -Point 4= Purkinje system (2.2m/sec)

-Age: 63 years man -Comes to the office due to exertional dyspnea that has been progressively worsening over the last 3 weeks. He sleep in a recliner with his head elevated because he gets short of breath when lying flat in bed. History: gout, dyslipidemia, hypertension -BP (154/89), pulse (85). Physical examination shows distended jugular veins in the semi-recumbent position. An apical heave is present on cardiac examination -There is pitting edema in the bilateral lower extremities. Levels of which of the following substances will be higher in the pulmonary vein compared to the pulmonary artery in this patient? a)Aldosterone b)Angiotensin I c)Angiotensin II d)Angiotensinase e)Angiotensinogen f)Arginine vasopressin g)Prostaglandin E1 h)Renin

Answer: Choice C (Angiotensin II) -This patient's exertional dyspnea, orthopnea (dyspnea when lying flat), jugular venous distention, cardiac heave, and peripheral edema are consistent with decompensated heart failure. The decreased cardiac output in heart failure reduces renal perfusion, which stimulates renin secretion by the juxtaglomerular cells of the kidney (Choice H) in a maladaptive effort to maintain effective blood volume and organ perfusion -Renin acts to convert angiotensinogen (produced by the liver) into angiotensin I in the systemic circulation (Choices B and E). Angiotensin I is then converted to angiotensin II, primarily within the small pulmonary vessels, by endothelial-bound angiotensin-converting enzyme (ACE). Because of the pulmonary location of angiotensin conversion, levels of angiotensin II are higher in the pulmonary vein (on the way out of the lungs) than in the pulmonary artery (on the way into the lungs). -Angiotensin II is a potent vasoconstrictor of the systemic circulation and causes a rise in BP (increased afterload) that worsens cardiac output. It also acts on the adrenal glands to stimulate aldosterone release, causing sodium retention and further worsening of volume overload (Choice A) Choice D= The angiotensinases are a group of enzymes found in multiple tissues and are responsible for the breakdown of angiotensin II and other structurally similar enzymes Choice F= Arginine vasopressin (antidiuretic hormone) is released from the posterior pituitary in response to high serum osmolality or decreased arterial blood volume. Because reduced cardiac output causes the body to perceive hypovolemia, antidiuretic hormone secretion is typically increased in heart failure Choice G= Prostaglandins are produced in multiple cell types, including endothelial cells, mast cells, and macrophages. Prostaglandin E1 is a potent vasodilator that maintains patency of the ductus arteriosus and causes afferent arteriolar vasodilation in the kidneys

-Age: 43 years woman -Comes to the ED due to nausea and vomiting with left-sided chest pain. She states that the pain began 30 mins ago while playing with her children -She has a history of dyslipidemia and hypertension. The patient's mother died from an MI at age 51. Physical examination reveals mild tenderness to palpation over the left chest. An ECG and cardiac biomarkers are normal -She is scheduled for an exercise stress test. During the test, which of thee following parameters is likely to be the most similar between the systemic and pulmonary circulation? a)Arterial oxygen content b)Arterial resistance c)Blood flow per minute d)Diastolic arterial pressure e)Driving pressure for blood flow f)Mean arterial pressure

Answer: Choice C (Blood flow per minute) -In order to maintain blood flow through the body, the blood flow (ml/min) in the pulmonary circulation must closely match the blood flow in the systemic circulation. This is true for conditions of both exercise and rest as the circulatory system is a continuous circuit. If the flow of blood through the pulmonary circulation is less than the flow of blood through the systemic circulation, the LV would soon empty completely. Alternatively, if the flow of blood is significantly greater in the pulmonary circulation than it is in the systemic circulation, the LV would soon be overloaded -The major exception to this is the bronchial circuit, which supplies oxygen and nutrients to the pulmonary parenchyma from the systemic circulation but drains mostly to the LA as opposed to the RA (creating a right to left shunt that acts as a partially independent circuit). However, this typically accounts for <5% of the systemic cardiac output Choice A= The arterial oxygen contents of the pulmonary and systemic circulations are dramatically different due to deoxygenated blood in the pulmonary arterial circulation and oxygenated blood in the systemic arterial circulation Choice B= Although the resistance of the pulmonary vasculature may be increased in some circumstances (e.g., high altitudes, idiopathic pulmonary hypertension), the arterial resistance in the systemic circulation is considerably higher in all scenarios Choices D, E and F= The mean arterial pressure, diastolic arterial pressure, and driving pressure for blood flow (difference between mean arterial and venous pressures) are considerably different in the pulmonary and systemic circulations both at rest and during exercise. The normal mean systemic arterial pressures ranges between 70-100mmhHg, and the normal mean pulmonary arterial pressure is approximately 14mmHg, whereas venous pressures in both circuits approach 0mmHg. Due to the low resistance of the pulmonary circuit, only a small difference between arterial and venous pressures is needed to maintain blood flow compared to the relatively large difference needed between systemic arterial and venous pressures

-Age: 34 years man -Comes to the ED with fatigue and lightheadedness. The patient had an upper respiratory infection last week, and since then his energy level has been low with shortness of breath on mild exertion. Otherwise, his medical history is insignificant. He is a lifetime nonsmoker -Temp. (37C), BP (80/60), pulse (120, regular). His pulse becomes undetectable to palpation during inspiration. The lungs are clear to auscultation, but the jugular veins are distended. Which of the following is the most likely diagnosis? a)Acute myocardial infection b)Acute viral myocarditis c)Cardiac tamponade d)Constrictive pericarditis e)Septic shock f)Tension pneumothorax

Answer: Choice C (Cardiac tamponade) -This presentation of hypotension, tachycardia, jugular venous distention with clear lungs, and pulsus paradoxus (manifesting as loss of palpable pulse during inspiration) is consistent with cardiac tamponade. This is most likely due to the patient's recent viral illness causing viral pericarditis with significant pericardial fluid accumulation. Normally there is a <10mmHg decrease in SBP during inspiration. Pulsus paradoxus refers to an abnormal exaggerated decrease in SBP >10mmHg on inspiration, a common finding in patients with large pericardial effusions causing tamponade -Inspiration causes an increase in systemic venous return, resulting in increased right heart volumes. Under normal conditions, this results in expansion of the RV into the pericardial space with little impact on the left side of the heart. However, in conditions that impair expansion into the pericardial space (e.g., cardiac tamponade), the increased RV volume that occurs with inspiration leads to bowing of the interventricular septum towards the LV. This leads to a decrease in LV end-diastolic volume and forward stroke volume, with a resultant decrease in SBP during inspiration Choice A= Acute MI typically presents with sudden onset of substernal discomfort, diaphoresis, and/or dyspnea, and can progress to cardiogenic shock (hypotension, tachycardia, jugular venous distention, and pulmonary edema). Patients can have weak or undetectable peripheral pulses; however, pulsus paradoxus is typically not seen Choice B= Acute viral myocarditis can lead to severe LV systolic dysfunction and present with acute congestive heart failure and cardiogenic shock. Patients with severe LV failure can pulsus alternans- a variation in pulse amplitude with alternate beats. Pulsus paradoxus is typically not seen in such patients Choice D= Pulsus paradoxus can also occur in patients with asthma, chronic obstructive pulmonary disease, and (less frequently) constrictive pericarditis. Constrictive pericarditis is caused by scarring and loss of elasticity of the normal pericardial sac; it takes several weeks to months to develop and only rarely occurs after recurrent episodes of acute pericarditis Choice E= The initial stage of septic shock is a hyperdynamic circulatory state with lowered systemic vascular resistance and increased cardiac output (warm shock). Weak pulses and pulsus paradoxus would not be expected in such patients. Choice F= In tension pneumothorax, breath sounds are typically absent on the affected side with hyperresonance to percussion. Other features of tension pneumothorax include tachypnea, tachycardia, hypotension, distended neck veins, and tracheal deviation to the contralateral side

-Age: 43 years man -Comes to the ED after experiencing 4 episodes of coffee ground emesis that started earlier this morning. He also describes epigastric pain over the last 3-4 months that was relieved by over-the-counter antacids. -He has no other past medical history. The patient has smoked a pack of cigarettes daily for the last 20 years but does not use alcohol or illicit drugs. -BP (70/40), pulse (130, regular). Extremities are cool to the touch with loss of skin turgor. IV access is obtained via 2 large-bore peripheral catheters, and rapid infusion of 2 liters of normal saline is initiated. This intervention is most likely to increase which of the following hemodynamic parameters? a)Aortic wall elasticity b)Diastolic ventricular compliance c)End-diastolic sarcomere length d)Heart rate e)Plasma renin activity f)Total peripheral resistance g)Ventricular muscle contraction velocity

Answer: Choice C (End-diastolic sarcomere length) This patient's hypotension, tachycardia, and cool extremities are indicative of hypovolemic shock most likely due to upper gastrointestinal bleeding from peptic ulcer disease (coffee ground emesis due to oxidation of heme iron by exposure to gastric acid). Decreased intravascular volume from significant blood loss activates the sympathetic nervous system and RAAS, which help to maintain systemic BP via the following compensatory mechanisms: -1) Stimulation of the heart results in increased contractility and heart rate, helping to maintain cardiac output -2) Constriction of the arteriolar beds serves to increase total peripheral resistance and maintain end-organ perfusion. This also shunts blood away from the extremities and towards the vital organs -3) Constriction of the systemic veins increases venous return to the heart, thereby assisting in maintaining preload -4) Increased renal sodium and water retention help to prevent further volume loss In the treatment of hypovolemic shock, the most important intervention other than identifying and eliminating the source of bleeding is rapid infusion of blood products and crystalloid solutions such as normal saline. By infusing IV fluids, intravascular volume and ventricular preload can be increased rapidly. The increase in preload stretches the myocardium and increases the end-diastolic sarcomere length, leading to an increase in stroke volume and cardiac output by the Frank-Starling mechanism. Choice A= Aortic wall elasticity is based on the intrinsic structure of the aortic media and is not affected by fluid infusions Choice B= The diastolic ventricular compliance is determined by intrinsic properties of the myocardium and is unaffected by crystalloid infusions. Conditions that decrease ventricular compliance include left ventricular hypertrophy (aortic stenosis, hypertension), hypertrophic cardiomyopathy, and infiltrative disorders (amyloidosis, sarcoidosis) Choice D= Adequate fluid resuscitation in a hypovolemic patient decreases the sympathetic drive and endogenous catecholamine release, resulting in decreased heart rate Choice E= Plasma renin activity increases in patients with hypovolemic shock due to activation of RAAS and would be expected to decrease after IV resuscitation Choices F and G= In a patient with hypovolemic shock, total peripheral resistance and myocardial contraction velocity are high due to sympathetic activation. Administration of fluids will reduce sympathetic activation and decrease both of these parameters.

-Age: 52 years woman -Comes to the ED due to increasing dyspnea and chest tightness over the past 2 weeks. She has a history of stage 4 chronic kidney disease due to chronic glomerulonephritis -Temp. (36.8C), BP (106/82), pulse (98). Systolic BP decreases by 12mmHg during inspiration. Physical examination shows distant heart sounds and clear lungs. Echocardiography in this patient is most likely to reveal which of the following? a)Apical wall motion abnormality b)Dilation of all cardiac chambers c)Engorged inferior vena cava d)Severe tricuspid regurgitation e)Systolic anterior motion of the mitral valve

Answer: Choice C (Engorged inferior vena cava) -This patient with progressive dyspnea and chest tightness over a period of 2 weeks and a drop in systolic BP >10mmHg with inspiration (i.e., pulsus paradoxus) likely has subacute cardiac tamponade; uremia in the setting of advanced renal failure can lead to pericarditis with associated pericardial effusion and sometimes tamponade. Tamponade classically presents with Beck's triad of hypotension (often less prominent with subacute compared to acute tamponade), distant heart sounds, and jugular venous distention -Normally, a pressure gradient exists between the vena cava and the right side of the heart, allowing blood to fill the right atrium during diastole. With inspiration, a reduction in intrathoracic pressure pulls additional blood into the right side of the heart and the pulmonary circulation, causing normal inspiratory collapse of the inferior vena cava (IVC) -With cardiac tamponade, increasing pericardial pressure begins to compress the relatively low-pressure, right-sided heart chambers and restrict diastolic filling. The increased right-sided pressures are transmitted backward to the vena cava, causing engorgement of the IVC with decreased inspiratory collapse that can be seen on echocardiography. In addition, because the RV is compressed and unable to expand during diastole, the increase in right-sided blood flow during inspiration causes the ventricular septum to bow into the LV; this reduces LV stroke volume during inspiration and is responsible for the observed >10mmHg drop in systolic BP -As tamponade progresses, impaired right-sided filling causes the left ventricle to receive less blood, resulting in decreased cardiac output and obstructive shock. Because the obstruction to blood flow is primarily right sided, there is no pulmonary edema, and the lungs remain clear to auscultation Choices A, B, D and E= A segmental (e.g, apical) wall motion abnormality suggests myocardial ischemia or infarction, dilation of all 4 cardiac chambers is seen with dilated cardiomyopathy, severe tricuspid regurgitation can occur with carcinoid syndrome, and systolic anterior motion of the mitral valve is seen with hypertrophic cardiomyopathy (and often exacerbates obstruction of the LV outflow tract). None of these conditions are expected to cause pulsus paradoxus or distant heart sounds

-Age: 56 years woman -Comes to the ED due to 3 days of fever, chills, and retrosternal chest pain. She has end-stage kidney disease related to previous uncontrolled hypertension and receives intermittent hemodialysis through a tunneled catheter. -Temp. (39C), BP (108/64), pulse (120). The patient is ill-appearing. The lungs are clear to auscultation, but a pericardial friction rub is present. Echocardiography reveals a moderate-sized pericardial effusion. Pericardiocentesis yields a turbid fluid with a large number of neutrophils. -Microscopic analysis of this patient's pericardial fluid is most likely to reveal which of the following pathogens? a)Bacilli with acid-fast cell walls b)Budding yeasts forming germ tubes c)Gram-positive cocci growing in clusters d)Lactose-fermenting gram-negative bacilli e)Silver stain-positive motile spirochetes f)Single-stranded RNA virus

Answer: Choice C (Gram-positive cocci growing in clusters) This patient with fever and chest pain has a turbid pericardial effusion with a high number of neutrophils, indicating purulent pericarditis. Most cases are caused by bacteria or fungi and develop due to: -Hematologic spread from a distant infection -Direct extension from an adjacent pneumonia or cardiac infection (e.g., endocarditis, perivalvular disease) -Complication of penetrating chest injury or cardiothoracic surgery Staphylococcus aureus, a gram-positive cocci that grows in clusters, is the most frequently isolated organism. It is particularly likely in patients who have portals from the skin to the bloodstream (e.g., tunneled dialysis catheter) or from the skin directly to the pericardium (e.g., chest injury, recent cardiothoracic surgery). Streptococcus pneumoniae is the most common organism in patients with adjacent pneumonia Although Candida albicans, a budding yeast that forms germ tubes, is a leading cause of fungal pericarditis, it is seen primarily in persons with significant risk factors for candidemia such as parenteral feeding, prolonged corticosteroid use, or immunosuppression due to malignancy; S. aureus is far more common in patients with end-stage renal disease who have vascular catheters (Choice B) Choice A= Mycobacterium tuberculosis, an acid-fast bacilli, can occasionally cause purulent pericarditis due to direct spread from the lung or mediastinal/hilar lymph node, but S. aureus is far more likely in someone with a hemodialysis catheter Choice D= Lactose-fermenting gram-negative bacilli (e.g., E. coli, Klebsiella pneumoniae, Enterobacter) are uncommon causes of purulent pericarditis. Although Klebsiella pericarditis can occasionally occur due to adjacent anaerobic lung abscesses, this patient with an indwelling catheter is far more likely to have S. aureus infection Choice E= Borrelia burgdorferi is a motile spirochete seen with silver stain. It can cause Lyme myopericarditis as a consequence of early disseminated disease, but most cases are mild and asymptomatic. In addition, an effusion caused by B. burgdorferi infection would be lymphocyte (not neutrophil) predominant Choice F= Although single-stranded RNA viruses such as Coxsackievirus can cause pericarditis, the pericardial fluid is usually lymphocyte (not neutrophil) predominant

-Age: 5 weeks boy -Receiving care in the neonatal intensive care unit. The patient was born at 26 weeks gestation with a birth weight of 0.79kg. Echocardiography reveals a patent ductus arteriosus and a dilated left atrium. -The patient has moderate pulmonary edema. During surgical ligation of the ductus arteriosus, the surgeons perform a left-sided thoracotomy, visualize the ductus, and place a small metal clamp across the vessel. Which of the following changes is most likely to take place immediately after ligation? a)Decreased right ventricular output b)Decreased right ventricular preload c)Increased left ventricular afterload d)Increased left ventricular output e)Increased right ventricular afterload

Answer: Choice C (Increased left ventricular afterload) -A patent ductus arteriosus (PDA) allows blood to flow from the aorta into the relatively low-pressure pulmonary artery. When a PDA persists longer than the initial postnatal period (e.g., >48 hours), the left-to-right shunt causes overcirculation of the lungs and the left side of the heart, which can ultimately lead to left-sided heart failure and pulmonary edema. Closure of a persistent PDA can be achieved with a medication that inhibits prostaglandin synthesis (e.g., indomethacin, ibuprofen) or by surgical ligation -When the PDA shunt is ablated during surgery, there is an immediate reduction in pulmonary venous return to the left atrium (i.e., decreased LV preload) and increased diastolic blood pressure in the aorta (i.e., increased LV afterload). The combination of these two immediate changes can result in acute decrease in LV stroke volume and cardiac output (Choice D) Choices A, B and E= In patients with a left-to-right shunt through a PDA, the increased volume load is handled entirely by the left ventricle. The RV receives deoxygenated blood from the body via the superior and inferior vena cava and pumps that blood to the pulmonary artery. This does not change when the PDA is closed, so there are no postligation changes to preload, afterload or cardiac output of the RV.

-Age: 54 years man -Comes to the physician for evaluation of exertional chest pain. He first noticed the pain a couple of months ago when he was shoveling snow off his driveway and has since experienced several episodes while doing other strenuous tasks -He describes the pain as heavy, crushing sensation and says it is relieved with rest. The patient undergoes coronary angiography and is found to have severe stenosis of the right coronary artery and left anterior descending and circumflex arteries. He is referred to a surgeon for coronary artery bypass grafting. During the procedure, a portion of his great saphenous vein is removed and grafted to one of the diseased coronary arteries to bypass its atherosclerotic narrowing. The vein used as a graft during this patient's procedure can be accessed at which of the following sites? a)At the midline of the popliteal fossa b)Just inferior to the anterior superior iliac spine c)Just inferolateral to the pubic tubercle d)Just superior to the inguinal ligament e)Over the lateral aspect of the foot

Answer: Choice C (Just inferolateral to the pubic tubercle) -When the left anterior descending artery (LAD) alone is occluded by an atherosclerotic plaque, the left internal mammary (thoracic) artery is the preferred vessel for bypass grafting due to superior patency rates. However, when multiple coronary arteries or vessels other than the LAD require revascularization, great saphenous vein grafts are routinely used -The great saphenous vein is located superficially in the leg and is the longest vein in the body. It courses superiorly from the medial foot, anterior to the medial malleolus, and up the medial aspect of the leg and thigh. In the proximal anterior thigh, 3-4cm inferolateral to the pubic tubercle, the great saphenous vein dives deep through the cribriform fascia of the saphenous opening to join the femoral vein. Surgeons access the great saphenous vein in the medial leg or, less commonly, near its point of termination in the femoral triangle of the upper thigh. The femoral triangle is bordered by the inguinal ligament superiorly, sartorius muscle laterally, and adductor longus muscle medially Choice A= The popliteal artery and vein course centrally through the popliteal fossa together with the tibial nerve. Common medical problems that occur in the popliteal fossa include popliteal artery aneurysms, which account for the majority of peripheral artery aneurysms, and synovial (Baker's) cysts, which are commonly associated with arthritis Choice B= No major vessels are located immediately inferior to the anterior iliac spine (ASIS). The ASIS serves as the superior attachment of the inguinal ligament, and a penetrating injury to the region below the ASIS could damage the lateral cutaneous nerve of the thigh Choice D= The deep circumflex iliac vessels course parallel and just superior to the inguinal ligament. The superficial and inferior epigastric veins course above the midportion of the inguinal ligament Choice E= The small saphenous vein can be found at the lateral aspect of the foot. This vein courses posteriorly to drain into the popliteal vein

-Age: 34 years Asian female -Hospitalized with progressive exertional dyspnea, lower extremity edema and cough. She also describes frequent nocturnal episodes of breathlessness and recent hoarseness. She doesn't smoke, drink or use drugs -Auscultation reveals loud first and second heart sounds and a mid-diastolic rumble at the cardiac apex. This patient's hoarseness is most likely caused by? a)Laryngeal edema b)Impaired arterial supply c)Nerve impingement d)Epithelial sloughing e)Vocal cord polyps

Answer: Choice C (Nerve impingement) -In instances, the left recurrent laryngeal nerve may be compressed to the point of neurapraxia (failure of nerve conduction due to blunt injury) by enlargement of the left atrium and/or other structures in the vicinity of its course as it loops behind the ligamentum arteriosum, underneath and around the aortic arch, and back up alongside the trachea to the larynx. Mitral stenosis can cause left atrial dilation sufficient to impinge on the left recurrent laryngeal nerve (Ortner syndrome) -The recurrent laryngeal nerves innervate all of the intrinsic muscles of the larynx except the cricothyroid muscle. Paresis of vocal cord muscles innervated by left recurrent laryngeal nerve can cause hoarseness Choice A= Laryngeal edema can cause hoarseness but is commonly due to direct laryngeal inflammation (e.g., by an upper respiratory infection) Choice B= Vascular disease or injury resulting in ischemia of the vocal cords, the recurrent laryngeal nerves, and/or the vagal motor nuclei in the brainstem could result in laryngeal neuromuscular dysfunction, producing hoarseness. However, the patient's history and presentation make MS a more likely culprit Choice D= Hoarseness is predominantly due to vocal cord disease (including mucosal inflammation or edema) or vocal cord malfunction. Epithelial sloughing could be associated with laryngeal mucosal disease and might be seen in asthma or airway exposure to noxious external gases or gastric acid. However, the other components of the patient's history and physical make severe MS more likely Choice E= Vocal cord epithelial or mucosal polyps can also cause hoarseness, but are less likely etiologies here, given the patient's history

-Age: 64 years man -Brought to the ED due to severe chest pain, diaphoresis, and shortness of breath. Symptoms began suddenly on awakening and have worsened over the past 2 hours. BP (150/90), pulse (102). Physical exam shows an S4. ECG shows ST-segment depression and T-wave inversion in the lateral leads. -Coronary angiography reveals a ruptured atherosclerotic plaque, with a thrombus in the left circumflex artery that causes near-total occlusion. Normal endothelial cells surrounding the lesion have released large amounts of a chemical substance that helps decrease thrombus propagation by inhibiting platelet aggregation -Which of the following substances was most likely secreted by these endothelial cells? a)Hageman factor b)Kallikrein c)Prostacyclin d)Protein C e)Serotonin f)Thromboxane A2

Answer: Choice C (Prostacyclin) The formation of a platelet plug (primary hemostasis) is essential for preventing bleeding after damage to vascular endothelium; it occurs in 3 steps: -Platelet adhesion takes place via von Willebrand factor acting as a connector that binds platelets to underlying collagen -Platelets become activated and secrete multiple substances, including adenosine diphosphate, ionized calcium, and fibrinogen, from their alpha and delta (dense) granules -Thromboxane A2 (Choice F) is released and acts as a vasoconstrictor and potent stimulator of platelet aggregation. ADP also stimulates platelet aggregation Balance is required as excessive platelet plug formation can lead to a pathologic thrombus that restricts blood flow (e.g., MI). To oppose the functions of thromboxane A2, the endothelium secretes prostacyclin (prostaglandin I2), which is derived from arachidonic acid and synthesized from prostaglandin H2 by prostacyclin synthase. Once secreted, prostacyclin acts locally to inhibit platelet aggregation and adhesion to the vascular endothelium and to cause vasodilation. Nitric oxide aids in these functions as well. Atherosclerosis can impair the ability of endothelial cells to synthesize prostacyclin and nitric oxide, creating localized predisposition to excessive platelet thrombus formation A synthetic prostacyclin, epoprostenol, is used in the treatment of pulmonary hypertension, peripheral vascular disease, and Raynaud disease Choice A= Hageman factor (factor XII) is synthesized by the liver and is activated by exposed collagen following damage to endothelial cells. It precipitates in secondary hemostasis by activating the intrinsic clotting pathway, it also activates fibrinolysis Choice B= Kallikrein converts kininogen into bradykinin. It also may play a role in triggering the fibrinolytic pathway, but it doesn't inhibit platelet aggregation Choice D= Protein C is a vitamin K-dependent factor synthesized by the liver. It regulates the coagulation cascade by inactivating factors Va and VIIIa Choice E= Serotonin has a wide range of function, including mediation of mood (in the CNS) and GI motility. It is also released from the delta granules of activated platelets and facilitates localized vasoconstriction

-Age: 4 months boy -Brought to the cardiology clinic by his parents for continued follow-up of tetralogy of Fallot. The diagnosis was made during routine antenatal sonography, and the pregnancy and delivery were otherwise uncomplicated. -The infant has been seen frequently in the clinic and has not had any cyanosis, respiratory distress, or difficulty feeding. The parents become concerned when their son's surgical pain is discussed because he does not have the clinical signs that other children with tetralogy of Fallot demonstrate. Which of the following is the major determinant of symptom severity in this condition? a)Aortic valve insufficiency b)Associated pulmonary malformations c)Right ventricular outflow tract obstruction d)Tricuspid valve insufficiency e)Ventricular septal defect

Answer: Choice C (Right ventricular outflow tract obstruction) -Tetralogy of Fallot (TOF) is characterized by ventricular septal defect (VSD), overriding aorta, right ventricular outflow tract (RVOT) obstruction, and right ventricular hypertrophy. The VSD generally is large, which allows for equal pressure in the right and left ventricles. Therefore, it is the amount of RVOT obstruction that determines how much deoxygenated blood is delivered to the systemic circulation. Infants with significant RVOT obstruction shunt more deoxygenated blood across the VSD to the aorta and are more cyanotic. Infants with no or minimal RVOT obstruction, such as in this patient, deliver more deoxygenated blood to the lungs and appear acyanotic. The degree of RVOT obstruction is dynamic and can increase suddenly, leading to profound cyanosis ("tet spells"). These can be caused by dehydration or hyperventilation but are usually idiopathic Choices A and D= Although the pulmonary valve is usually small and stenotic in TOF (contributes to RVOT obstruction), involvement of the aortic or tricuspid valves is unusual Choice B= less commonly than 25% of patients with TOF have extracardiac malformations. The most common associated syndromes include Down, Alagille, and DiGeorge. Pulmonary malformations are extremely rare Choice E= A VSD is universally present in TOF. The VSD invariably is large and does not determine the degree of cyanosis.

-Age: 29 years man -Brought to the ED by his friend due to retrosternal chest pain for the past hour. The patient is agitated and restless. Temp. (38.1C), BP (180/106 in the right arm and 182/104 in the left arm), pulse (110), RR (20). -Physical exam: bilaterally dilated pupils, normal heart and lung sounds, and a nontender abdomen. ECG= sinus tachycardia with ST-segment depression in the precordial leads. -Chest x-ray= no parenchymal opacities or pleural effusion and a normal mediastinum and cardiac silhouette. Cardiac enzymes are normal -Sublingual nitroglycerin and benzodiazepine therapy relieve the symptoms. Which of the following is the most likely cause of this patient's chest pain? a)Acute pericarditis b)Aortic dissection c)Coronary artery thrombosis d)Coronary artery vasoconstriction e)Pulmonary infarction

Answer: Choice D (Coronary artery vasoconstriction) -This patient is agitated and restless and has dilated pupils, tachycardia, and hypertension, suggesting increased sympathetic activity from cocaine intoxication. Cocaine use commonly leads to adverse cardiovascular effects that result from inhibition of the presynaptic reuptake of NE. -Overstimulation of adrenergic receptors (e.g., alpha-1, beta-1) causes an increase in heart rate, blood pressure, and myocardial contractility, leading to an increase in myocardial oxygen demand. Arterial vasoconstriction, including coronary vasoconstriction, is also enhanced (alpha-1 effect), leading to a decrease in myocardial oxygen supply. This myocardial oxygen supply-demand mismatch often causes myocardial ischemia, evidenced in this patient by retrosternal chest pain and ST-segment depression on ECG. Peripheral vasoconstriction may also impair heat loss, causing low-grade fever. -Patients with suspected cocaine-induced chest pain are frequently treated with nitroglycerin (reduces cardiac preload) and benzodiazepines (reduces sympathetic outflow to alleviate tachycardia/hypertension) in order to improve myocardial ischemia. Benzodiazepines also calm cocaine-induced psychomotor agitation, which helps further decrease myocardial oxygen demand Choice A= Acute pericarditis causes sharp, pleuritic chest pain and sometimes fever and may lead to widespread ST-segment elevation (but not depression) on ECG. Choice B= Acute aortic dissection commonly presents with abrupt-onset, tearing chest pain in the setting of hypertension, and cocaine use increases the risk. However, a widened mediastinum is expected on chest x-ray, and BP asymmetry is often present in the upper extremities. In addition, the chest pain is unlikely to resolve without pain medications Choice C= Coronary artery thrombosis causes MI associated with atherosclerotic plaque rupture, which is unlikely in this relatively young patient. Cocaine use rarely causes coronary artery thrombosis; associated chest pain is usually from myocardial ischemia due to myocardial oxygen supply-demand mismatch Choice E= Acute pulmonary embolism may cause pulmonary infarction, leading to pleuritic chest pain. An associated pleural effusion is usually expected on chest x-ray, and the pain is unlikely to resolve without pain medication

-Age: 43 years man -Comes to the office due to occasional chest discomfort over the last 6 weeks. He thinks he is most likely experiencing musculoskeletal pain but is concerned due to family history of heart disease. The patient has no medical conditions and does not use tobacco -He leads an active lifestyle and exercises everyday. He undergoes treadmill exercise testing. Baseline BP (122/75), pulse (54). After 10 mins of exercise, his BP is 155/80 and pulse is 150 -He has no chest pain and ECG shows no abnormalities. Compared to pretest condition, which of the following is the single most important limiting factor for left ventricular myocardial blood supply during the test? a)Contraction force b)Coronary vasoconstriction c)Diastolic aortic pressure d)Duration of diastole e)Intraventricular pressure f)Systolic ventricular wall stress

Answer: Choice D (Duration of diastole) -Myocardial perfusion is provided by the right and left coronary arteries arising directly from the aortic root. In contrast to most other vascular beds of the body, LV myocardial perfusion occurs mainly during diastole. During ventricular systole, intraventricular pressure and wall stress in the LV exceed the aortic perfusion pressure (e.g., 120mmHg); preventing effective coronary perfusion. Wall tension is highest in the subendocardial region, making this area of the myocardium particularly susceptible to ischemia. Relaxation of the LV during diastole decreases intraventricular pressure to about 10mmHg (which is much lower than aortic diastolic pressure (80mmHg), providing a driving force that allows for adequate perfusion) -Increased heart rate shortens the time of ventricular relaxation (duration of diastole); therefore, the time available for maximal coronary blood flow decreases and consequently becomes the major limiting factor for coronary blood supply to the myocardium Choice B= During exercise, mechanisms such as flow-mediated dilation and release of vasodilatory such as adenosine and nitric oxide ensure adequate coronary blood supply Choices A, E and F= LV contraction force, intraventricular pressure, and ventricular wall stress all increase during exercise to provide adequate blood flow to actively contracting skeletal muscle. However, because there is already minimal LV perfusion occurring during systole, the increase in these forces does not cause further limitation in LV myocardial perfusion Choice C= Diastolic aortic pressure is the driving force for coronary blood flow during diastole, and coronary blood flow is reduced in patients with low diastolic pressures. This patient's diastolic pressures are similar while at rest and during exercise and would not contribute to decreased coronary blood supply

-Age: full-term newborn -Evaluated in the delivery room due to mild tachypnea and cyanosis immediately after birth. Fetal ultrasonography performed during pregnancy revealed transposition of the great arteries. Heart rate and BP are normal. Oxygen saturation is 72% -Cyanosis is apparent in the hands, feet, lips, and tongue. Peripheral pulses are strong. Bedside echocardiography confirms the prenatal diagnosis; there is also a small patent foramen ovale and a small patent ductus arteriosus. Definitive surgical correction of the main congenital abnormality is planned. Pending definitive repair, which of the following temporary measures would be most likely to improve this patient's condition? a)Ablating the sinoatrial node b)Administering 100% oxygen c)Administering digoxin d)Enlarging the patent foramen ovale e)Ligating the patent ductus arteriosus

Answer: Choice D (Enlarging the patent foramen ovale) -Transposition of the great arteries (TGA) is a cyanotic congenital heart defect caused by abnormal spiraling of the conotruncal septum, which divides the truncus arteriosus into the aorta and the pulmonary artery. The result is abnormal connections of the right ventricle to the aorta and the left ventricle to the pulmonary artery. As a result, oxygenated blood cycles through the left side of the heart to the pulmonary circulation and back while deoxygenated blood cycles through the right side of the heart to the systemic circulation and back; this results in neonatal cyanosis -TGA is incompatible with life without a left-to-right mixing lesion to provide systemic circulation with partially deoxygenated blood; a greater degree of intercirculatory mixing typically results in a lesser degree of hypoxemia. Examples of mixing lesions include patent ductus arteriosus, patent foramen ovale, and/or ventricular septal defect -Initial management of TGA therefore must ensure mixing of oxygenated and deoxygenated blood. For example, administration of prostaglandin E1 maintains patency of the ductus arteriosus. Balloon atrial septostomy, which opens or widens the foramen ovale, may also be performed to allow for mixing at the atrial level and typically improves oxygen saturation immediately. Choice A= Sinoatrial node ablation is sometimes used for treating tachyarrhythmias involving the SA node. However, the procedure would not improve this patient's hypoxemia Choice B= Because deoxygenated blood bypasses the pulmonary circulation and is delivered directly to the systemic circulation in TGA, supplemental oxygen administration does not significantly improve hypoxemia or cyanosis Choice C= Digoxin is a positive inotropic agent that can treat heart failure due to left ventricular systolic dysfunction (a finding not present on this patient's echocardiogram). Increased myocyte contractility would not improve this patient's oxygenation Choice E= Ligating the PDA in a patient with TGA would decrease left-to-right shunting of blood and worsen hypoxemia and cyanosis

-Age: 2 years boy -Previously healthy, brought to the ED due to a 4-day history of poor feeding, diarrhea, and abdominal pain. The patient has had 5-7 bowel movements every day for the past 4 days. Temp. (36.8C), BP (65/42). On physical examination, the patient is lethargic. Mucous membranes are dry, and the eyes are sunken. -The lungs are clear to auscultation. No murmurs are heard. Abdominal examination shows distention and tenderness, bowel sounds are hyperactive throughout. Which of the following changes is most likely present in this patient? a)Extravasation of fluid from systemic capillaries b)Increased arteriolar capacitance c)Increased central venous pressure d)Increased ventilatory rate e)Relaxation of venous smooth muscle

Answer: Choice D (Increased ventilatory rate) -This patient has had several days of poor oral intake and diarrhea and now has signs of hypovolemia (e.g., dry mucous membranes, sunken eyes) and shock (e.g., lethargy, severe hypotension). With shock, there is poor organ and tissue perfusion that leads to tissue hypoxia with increased anaerobic metabolism and consequent lactic acidosis. In response to the reduction in pH caused by metabolic acidosis there is increased ventilation to increase CO2 exhalation (i.e., compensatory respiratory alkalosis) -Severe diarrhea causes loss of bicarbonate in the stool, further contributing to metabolic acidosis and a compensatory increase in ventilation in this patient Choices A, B and E= The reduced cardiac output in hypovolemic shock stimulates arteriolar vasoconstriction to increase systemic vascular resistance and help increase BP. This reduces arteriolar capacitance. There is also contraction of venous smooth muscle to cause venular constriction and help increase venous return to the heart. The constricted vasculature reduces extravasation of intravascular fluid. In contrast, septic shock involves peripheral vasodilation with relaxation of vascular smooth muscle, increased vascular capacitance, and extravasation of intravascular fluid Choice C= Blood volume is markedly decreased in hypovolemic shock, leading to decreased central venous pressure. Central venous pressure is increased in cardiogenic shock and obstructive shock.

-Age: 32 years man -Previously healthy, brought to the ED after a stab injury. His friends report that they were "walking down the street minding our own business when a guy jumped out in front of us from behind a dumpster and stabbed him in the chest". They were able to disarm the man after his single attack, and the man then ran off into the alley -The patient is conscious but in distress. Physical examination shows a laterally directed anterior chest wall stab wound at the 5th intercostal space along the left midclavicular line. Which of the following structures is most likely to have been injured in this patient? a)Azygos vein b)Inferior vena cava c)Left atrium d)Left lung e)Right ventricle

Answer: Choice D (Left lung) -A penetrating, laterally directed stab wound (possibly from a left-handed assaulter) that involves the 5th intercostal space (ICS) at the left anterior midclavicular line (MCL) would most likely injure the left lung. The lungs occupy most of the volume of the thoracic cavity. The apex of each lung extends into the neck (3-4cm above the first rib). The lung bases are in direct contact with the diaphragm, which separates the right lung from the right lobe of the liver and left lung from the stomach, spleen and (occasionally) left lobe of the liver. The mediastinal surface of each lung has a cardiac impression that accommodates the heart -A deeper (and more medially directed) wound could have injured the left (not the right) ventricle. The cardiac apex is formed by the LV. Depending on heart size, the point of maximal apical impulse can be felt at the 5th left ICS (between the 5th and 6th ribs) at the left MCL. All other heart chambers, including the RV, lie medial to the left MCL and are unlikely to have been affected by the injury. The RV forms the sternocostal (anterior) surface of the heart (Choice E). The diaphragmatic (inferior) surface is formed by the LV and RV and is in contact with the central tendon of the diaphragm. The posterior surface of the heart is formed mainly by the left atrium (Choice C) Choice A= The azygos vein lies in the posterior mediastinum immediately to the right of the midline. It drains blood from the posterior intercostal veins into the SVC Choice B= The IVC is located in the mediastinum, lies to the right of midline, and drains into the right atrium

-Age: 70 years man -Comes to the ED due to severe midback pain that started several hours ago. He describes the pain as excruciating and wants immediate relief. The patient also has nausea, diaphoresis, and lightheadedness. He has a history of hypertension and chronic kidney disease. His medication compliance has been poor. He has a 40-pack-year smoking history (active smoker) -BP in right arm (220/105). ECG shows sinus tachycardia and voltage criteria for left ventricular hypertrophy with secondary ST-segment and T wave changes. After initial evaluation, a transesophageal echocardiogram shows a dissection flap in the descending aorta but no evidence of dissection in the ascending aorta. The dissection flap in this patient most likely originates near which of the following points? a)Brachiocephalic trunk b)Celiac trunk c)Intercostal arteries d)Left subclavian artery e)Renal arteries

Answer: Choice D (Left subclavian artery) This patient's clinical presentation with severe back pain, hypertension, ECG abnormalities, and transesophageal echocardiogram finding of a dissection flap in the descending aorta is consistent with Stanford type B aortic dissection. The Stanford system uses the anatomic localization of the dissection plane for the classification of aortic dissections: -Stanford type A refers to aortic dissections that involve any part of the ascending aorta -Stanford type B refers to all other dissections involving the descending aorta Type B dissections involving the descending aorta typically originate close to the origin of the left subclavian artery. Type A aortic dissections usually originate in the sinotubular junction. The areas are thought to be predominantly affected due to increases in the rate of rise of pressures (dP/dT) and in the shearing forces at these sites in patients with severe hypertension Choices A, B, C and E= Distal propagation of a type A dissection or proximal propagation of a type B dissection can affect the aortic arch (e.g., brachiocephalic artery, left common carotid artery, left subclavian artery). Either type of dissection can propagate distally into the thoracoabdominal aorta, involving the origins of the celiac trunk, intercostal arteries, and renal arteries

-Age: 64 years man -With type 2 diabetes is evaluated in the clinic due to occasional dizziness. His symptoms usually occur while playing table tennis. The patient has not had any falls or loss of consciousness. He has no known history of coronary artery disease or stroke -The patient has smoked a pack of cigarettes daily for the past 40 years and occasionally drinks alcohol. Orthostatic vital signs are normal. ECG shows normal sinus rhythm. Doppler ultrasound evaluation of the left vertebral artery reveals retrograde (caudal) flow instead of normal anterograde flow. Which of the following arteries is most likely to be occluded based on these ultrasound findings? a)Innominate artery b)Left internal carotid c)Left internal mammary d)Left subclavian e)Right vertebral

Answer: Choice D (Left subclavian) -This patient's presentation is consistent with subclavian steal syndrome, which typically occurs due to hemodynamically significant stenosis of the subclavian artery proximal to the origin of the vertebral artery. Subclavian stenosis is typically caused by atherosclerosis, although less common etiologies include Takayasu arteritis and complications from heart surgery (e.g., aortic coarctation repair). The lowered distal subclavian arterial pressure leads to reversal in blood flow ("steal") from the contralateral vertebral artery to the ipsilateral vertebral artery, away from the brainstem -Most patients with subclavian artery stenosis are asymptomatic. When symptoms occur, they are typically related to arm ischemia in the affected extremity (e.g., exercise-induced fatigue, pain, paresthesias) or vertebrobasilar insufficiency (e.g., dizziness, vertigo, drop attacks). Physical examination can show a significant difference (>15mmHg) in brachial systolic BP between the affected arm and normal arm. Doppler ultrasound of the cerebrovascular and upper extremity arterial circulation establishes the diagnosis in most patients Choice A= Although innominate (brachiocephalic) artery stenosis or occlusion can cause subclavian steal syndrome, patients with significant occlusion would develop retrograde flow through the right vertebral artery rather than the left Choice B= internal carotid artery occlusion may occur due to thrombosis or embolism, leading to a transient ischemic attack (TIA) or ischemic stroke. Patients typically have profound neurologic deficits (e.g., contralateral homonymous hemianopsia, hemiparesis, hemisensory loss), including cortical signs (e.g., aphasia, agnosia, neglect). Blood flow reversal in the vertebral artery is not characteristic Choice C= Coronary-subclavian steal phenomenon occurs in patients with prior coronary artery bypass surgery using the internal mammary artery (IMA). Similar to that of subclavian steal syndrome, blood flow through the IMA can reverse and steal flow from the coronary circulation during increased demand (e.g., upper extremity exercise). However, symptoms are typically related to coronary ischemia (e.g., angina pectoris) Choice E= Right vertebral artery occlusion usually leads to ischemic stroke or TIA and can cause similar symptoms as seen in this patient. However, occlusion of the right vertebral artery would cause retrograde flow on the right versus the left, and symptoms are typically precipitated by arm exertion

-Age: 65 years man -Dies while hospitalized for severe breathing difficulty. The patient had several prior episodes of dyspnea and cough requiring hospitalization. History: hypertension, chronic kidney disease -Smoked daily for 38 years; immigrated to the US 20 years ago. Autopsy (lungs)= alveolar macrophages containing aggregates of golden-brown cytoplasmic granules that turn dark blue with Prussian blue staining. Which of the following conditions is most likely associated with this patient's microscopic findings? a)Airway hyperreactivity b)Coal particle inhalation c)Granulomatous microbial infection d)Left ventricular dysfunction e)Pulmonary arterial hypertension f)Pulmonary thromboembolism

Answer: Choice D (Left ventricular dysfunction) -This patient most likely had heart failure due to left ventricular dysfunction. Impaired forward pumping by the left ventricle causes increased pulmonary capillary pressure that can lead to pulmonary edema and dyspnea. -The rise in hydrostatic pressure also disrupts the integrity of the pulmonary capillaries, leading to extravasation of red blood cells and alveolar hemorrhage. The red blood cells are eventually phagocytosed by macrophages, and the iron from hemoglobin is converted to hemosiderin -Prussian blue stain detects ferric iron stores (e.g., ferritin and hemosiderin). In the Prussian blue reaction, colorless potassium ferrocyanide is converted by iron to blue-black ferric ferrocyanide. Macrophages containing golden-brown cytoplasmic granules that turn blue with Prussian blue staining represent hemosiderin-laden macrophages (siderophages). These cells may be found in any tissue where macrophages encounter extravasated RBCs; in the alveolar parenchyma, they are often called "heart failure cells" Choice A= Airway hyperreactivity occurs in asthma or chronic obstructive pulmonary disease, neither of which if associated with hemosiderin-laden macrophages Choice B= Coal worker's pneumoconiosis results from chronic inhalation of coal dust and is associated with alveolar macrophages containing black intracytoplasmic aggregates, so-called carbon-laden macrophages. These carbon aggregates do not stain blue with Prussian blue staining Choice C= A number of infectious organisms (e.g., Mycobacterium TB, histoplasma capsulatum) can survive within macrophages and produce a granulomatous response in the lungs; however, these organisms are typically visualized using other special staining methods (e.g., acid-fast staining) and would not stain blue with Prussian blue Choice E and F= Pulmonary arterial hypertension (primary pulmonary hypertension) and pulmonary thromboembolism both involve an elevation in pulmonary arterial pressure; however, pulmonary edema with extravasation of RBCs does not occur as pulmonary capillary and pulmonary venous pressures are not elevated.

-Age: 53 years man -Comes to the office due to difficulty breathing and increasing fatigue. He has shortness of breath at night and has been sleeping in a recliner to help relieve his dyspnea. Medical history is significant for hypertension and hyperlipidemia. Two months ago, the patient suffered a myocardial infarction that was not revascularized due to a delay in seeking treatment. -He quit smoking afterward but has previously smoked a pack of cigarettes daily (25 years). On cardiac auscultation, a low-frequency diastolic heart sound is heard shortly after the 2nd heart sound when the patient lies in the left lateral decubitus position. Cardiac imaging shows hypokinesis of the left ventricular wall. Which of the following is most likely to accentuate this patient's abnormal auscultation finding? a)Amyl nitrite inhalation b)Furosemide injection c)Having the patient stand up d)Listening at end expiration e)Phase 2 of the Valsalva maneuver

Answer: Choice D (Listening at end expiration) -This patient has decompensated heart failure (fatigue, dyspnea) with an audible third heart sound (S3), a low-frequency sound that occurs shortly after S2 during diastole. S3 is caused by a sudden limitation of ventricular movement during rapid passive ventricular filling. It can be a normal finding in healthy children and young adults. However, the presence of S3 in patients age >40 is considered abnormal and suggests ventricular volume overload or enlargement. It is often heard with aortic/mitral regurgitation or systolic heart failure (e.g., dilated or ischemic cardiomyopathy). In this case, imaging is consistent with LV systolic dysfunction secondary to this patient's recent MI. -LV gallops (S3 or S4) are best heard with the bell of the stethoscope over the cardiac apex while the patient is in the left lateral decubitus position. Listening at end expiration makes the sound even more audible by decreasing lung volume and bringing the heart closer to the chest wall. Choice A= Amyl nitrite inhalation causes vasodilation, resulting in a transient decrease in venous return and BP. It would lessen S3 intensity due to the reduction of end-systolic ventricular volume Choice B= Furosemide injection would lead to brisk diuresis, resulting in a decrease in end-systolic ventricular volume that would likely lessen S3 intensity Choices C and E= The straining phase (phase 2) of the Valsalva maneuver (bearing down against a closed glottis) as well as abrupt standing decreases venous return to the heart and ventricular volumes, reducing S3 intensity

-Age: 1-hour old neonate -Prenatally diagnosed tetralogy of Fallot is admitted to the neonatal intensive care unit with cyanosis. The mother is a 28-year-old gravida 3 para 2 who received routine prenatal care. The patient was born via spontaneous vaginal delivery at 35 weeks gestation -Initial Apgar scores are 5 and 7 at 1 and 5 minutes after birth, respectively. The neonate's lips, fingers, and toes appear cyanotic -This patient should be immediately treated with a medication producing which of the following effects? a)Decreased left ventricular end diastolic pressure b)Increased pulmonary surfactant levels c)Increased pulmonary vascular resistance d)Maintenance of ductus arteriosus patency e)Stimulation of fetal hemoglobin production

Answer: Choice D (Maintenance of ductus arteriosus patency) -Tetralogy of Fallot is a cyanotic, congenital heart defect characterized by pulmonary stenosis, an overriding aorta, right ventricular hypertrophy, and a ventricular septal defect. When pulmonary stenosis is severe, TOF is a ductal-dependent lesion that relies on the patency of the ductus arteriosus (DA) for adequate oxygenation -The DA is a fetal vascular structure that connects the main pulmonary artery and aorta. In utero, blood (which is oxygenated by the placenta) bypasses the lungs by flowing from the pulmonary artery through the DA to the aorta. After birth, the lungs fill with oxygen and pulmonary vascular resistance decreases, causing the direction of flow through the DA to reverse (i.e., from aorta to pulmonary artery). DA patency is maintained by circulating prostaglandin released by the placenta; after birth, prostaglandin production stops, and the DA starts closing -In severe TOF, critical pulmonary stenosis or atresia limits blood flow from the right side of the heart to the pulmonary circulation for oxygenation. Instead, blood reaches the pulmonary vascular bed via retrograde flow from the aorta through the DA. Without flow through the DA, affected neonates are unable to oxygenate blood. These patients are cyanotic after birth, as in this neonate, and require immediate administration of prostaglandin E1 to maintain patency of DA Choice A= Heart failure medications that reduce preload (e.g., diuretics) decrease LV end diastolic pressure. This decrease would not improve RV outflow tract obstruction or increase retrograde blood flow through the DA in patients with TOF Choice B= Administration of pulmonary surfactant can improve lung compliance in preterm neonates with respiratory distress syndrome. Because the primary pathology in TOF is cardiac rather than pulmonary, surfactant will not improve oxygenation Choice C= Increased pulmonary vascular resistance would worsen the pathophysiology of TOF by decreasing blood flow from the aorta through the DA into the lungs Choice E= Hydroxyurea increases fetal hemoglobin production, thereby improving the oxygen carrying capacity of blood in sickle cell anemia. In TOF, oxygenation is impaired due to inadequate pulmonary blood flow from right ventricular outflow tract obstruction

-Age: 18 years man -Suddenly collapses during a high school soccer game. There was no preceding collision or trauma. Immediate resuscitation is initiated but he dies before the emergency medial service arrives. -The patient had recently experienced occasional exertion-related chest discomfort and dyspnea. Otherwise, he had no significant medical history. He took no medications and did not use tobacco, alcohol or illicit drugs. Which of the following most likely to be seen on autopsy? a)Atherosclerotic narrowing of the left anterior descending artery b)Dilation of all cardiac chambers with thin ventricular walls c)Extracellular deposits and stiff ventricular walls d)Myocardial hypertrophy with patchy interstitial fibrosis e)Thrombus at the bifurcation of the main pulmonary artery

Answer: Choice D (Myocardial hypertrophy with patchy interstitial fibrosis) -Sudden cardiac death (SCD) is most often precipitated by cardiac arrhythmia (e.g., ventricular tachycardia/fibrillation) that leads to markedly decreased cardiac output and impaired coronary artery perfusion -In older adults (e.g., age >35 years), myocardial ischemia due to atherosclerotic coronary artery disease (Choice A) is the most common cause of SCD. However, young adults who experience SCD usually have underlying structural heart disease; hypertrophic cardiomyopathy (HCM) is one of the most common precipitating diseases. Most patients with HCM are asymptomatic, but some may experience exertional dyspnea, fatigue, chest pain or syncope. -HCM is an autosomal dominant genetic disorder caused by mutation of one of several genes coding for cardiac sarcomere proteins. It is characterized by cardiomyocyte hypertrophy (predominantly affecting the septum) and myofiber disarray consisting of an irregular arrangement of abnormally shaped cardiomyocytes and increased interstitial fibrosis. This structural disarray leads to conduction abnormalities that predisposes to ventricular arrhythmia and SCD Choice B= Dilated cardiac chambers and thin ventricular walls are consistent with dilated cardiomyopathy (DCM), a less common cause of SCD than HCM in young adults. A patient with DCM would likely have symptoms of heart failure (e.g., lower extremity swelling, dyspnea, orthopnea) and would not be capable of participating in strenuous activities such as soccer Choice C= Extracellular deposits and stiff ventricular walls suggest restrictive cardiomyopathy due to amyloidosis or hemochromatosis. Although restrictive cardiomyopathy can cause SCD, there is usually a preceding course of progressive and symptomatic heart failure Choice E= Thrombus at the bifurcation of the main pulmonary artery (i.e., saddle pulmonary embolism) is a potential cause of SCD; however, a preceding history of exertion-related dyspnea and chest discomfort would not be expected

-Age: 67 years man -Admitted to the hospital due to acute right sided weakness. History: hypertension, hyperlipidemia, coronary artery disease, for which the patient underwent 2 stents in the left anterior descending artery. -He also has intermittent claudication that limits exercise capacity. The patient's antihypertensive regimen includes chlorthalidone, amlodipine, and lisinopril, which he takes as prescribed. He is an ex-smoker (35-pack-year history) -BP (180/110), pulse (80). Serum creatinine (1.6mg/dl). CT scan of the head reveals hemorrhage of the left basal ganglia, and renal ultrasound shows that the right kidney is 7cm long and the left kidney is 11cm long. Which of the following mechanisms explains the renal morphology in this patient? a)Congenital hypoplasia b)Denervation c)Lack of trophic hormones d)Oxygen and nutrient deprivation e)Senile changes

Answer: Choice D (Oxygen and nutrient deprivation) -This patient has an intraparenchymal hemorrhage in the setting of severe, refractory hypertension. He has coronary artery disease and peripheral vascular disease (claudication) with multiple risk factors (e.g., smoking, hyperlipidemia) for diffuse atherosclerosis. The elevated serum creatinine and the renal ultrasound demonstrating a small (atrophic) right kidney suggest that the patient's refractory hypertension is most likely due to unilateral renal artery stenosis. -The most common cause of renal artery stenosis is obstruction by atheromatous plaque at the origin of the renal artery, leading to unilateral renal ischemia. The ischemic kidney secretes high levels of renin, which activates RAAS and results in systemic hypertension that is often difficult to control despite aggressive antihypertensive therapy. Eventually, the stenosis may also cause renal atrophy due to oxygen and nutrient deprivation. On histologic examination, the atrophic kidney shows crowded glomeruli, tubulointerstitial atrophy and fibrosis, and often, focal inflammatory infiltrates. Choice A= Congenital renal hypoplasia is a failure of the kidney to develop to normal size. Involvement is unilateral in most cases. However, this condition does not cause secondary hypertension Choice B= Unilateral denervation of a kidney, either during or after development, is not known to have significant trophic effects (affecting kidney growth and morphology) Choice C= The role of trophic hormones in renal development remains unclear. Still, a deficiency of such hormones would be expected to produce bilateral renal dysplasia Choice E= Aging is associated with loss of nephron mass as a result of variable degrees of arteriolosclerosis, focal glomerulosclerosis, and tubulointerstitial fibrosis. These processes produce symmetric, not unilateral, decrements in kidney size and functional capacity

Question 1 of 2 -Age: 67 years man -Comes to the ED due to progressive shortness of breath and chest tightness. He has had no lightheadedness or syncope. The patient takes lisinopril for hypertension and metformin for type 2 diabetes. Smoker (daily pack for last 40 years). BP cuff is inflated to 140mmHg and the pressure is released very slowly. At 120mmHg, intermittent Korotkoff sounds are heard only during expiration. At 100mmHg, Korotkoff sounds are heard throughout the respiratory cycle. -The physical exam finding can be seen in which of the following conditions? a)Aortic valve disease b)Mitral valve disease c)Myocardial ischemia d)Pericardial disease e)Peripheral vascular disease

Answer: Choice D (Pericardial disease) -Systemic arterial pressure normally falls by <10mmHg during normal inspiration. Pulsus paradoxus refers to an exaggerated drop (>10mmHg) in systolic blood pressure during inspiration. It is detected by inflating a BP cuff above systolic pressure and gradually deflating it. The difference between the systolic pressure at which Korotkoff sounds first become audible during expiration and the pressure at which they are heard throughout all phases of respiration quantifies pulsus paradoxus (20mmHg in this patient) -Inspiration causes an increase in systemic venous return, resulting in increased right heart volumes. Under normal conditions, this results in expansion of the right ventricle into the pericardial space with little impact on the left side of the heart. However, in conditions that impair expansion into the pericardial space (e.g., acute cardiac tamponade), the increased right ventricular volume occurring with inspiration leads to bowing of the interventricular septum towards the LV. This leads to a decrease in LV end-diastolic volume and stroke volume, with a resultant decrease in systolic pressure during inspiration Choice A= Fixed LV outflow tract obstruction (e.g, valvular aortic stenosis) can cause pulses parvus et tardus, which is palpable as a slow-rising, low-amplitude pulse due to diminished stroke volume (pulsus parvus) and prolonged LV ejection time (pulses tardus) Choice B= Patients with chronic severe mitral regurgitation have a bounding pulse with a brisk upstroke due to increased LV ejection volumes. Arterial pulses are overall reduced in amplitude in patients with mitral stenosis due to reduced stroke volume Choice C= Myocardial ischemia or infarction can lead to decreased systemic blood pressure or hypotension due to reduced cardiac output, but it would not be expected to cause pulsus paradoxus Choice E= Peripheral vascular disease causes decreased pulse amplitude within the affected extremities. This is most commonly observed in the lower extremities, which will have diminished or absent distal pulses caused by widespread peripheral artery disease

-Age: 23 years man -Comes to the ED with sudden onset of heart palpitations that started while he was at his desk at work. The patient has no known medical problems and does not smoke or illicit drugs. -He drinks alcohol occasionally on the weekends. Initial BP (110/70) and pulse (160, regular). Gentle neck massage just below the angle of the right mandible provides immediate improvement of his condition. His BP is now (120/80) and pulse (75). Which of the following mechanisms is responsible for improvement of this patient's condition? a)Decreased baroreceptor firing rate b)Increased sympathetic output to the sinoatrial node c)Increased systemic vascular resistance d)Prolonged AV refractory period e)Prolonged ventricular myocardium refractory period

Answer: Choice D (Prolonged AV node refractory period) -This patient's presentation with sudden onset of palpitations and rapid regular tachycardia is consistent with paroxysmal supraventricular tachycardia (PSVT), which is most often due to a reentrant impulse traveling circularly between the slowly and rapidly conducting segments of the AV node. Vagal maneuvers such as carotid sinus massage, Valsalva, and cold-water immersion can be used to acutely terminate PSVT -The carotid sinuses are bulges in the internal carotid artery with baroreceptors located immediately distal to the bifurcation of the common carotid artery. The carotid sinus reflex has an afferent limb that arises from the baroreceptors in the sinus and travels to the vagal nucleus and medullary centers via the glossopharyngeal nerve (cranial nerve (CN) IX), whereas the efferent limb carries parasympathetic impulses to the SA and AV nodes via the vagus nerve (CN X). Carotid sinus massage leads to increased afferent firing from the carotid sinus, which in turn, increases vagal parasympathetic tone. This slows conduction through the AV node and prolongs the AV node refractory period, helping to terminate the reentrant tachycardia Choices A, B and C= Carotid sinus massage stimulates the baroreceptors and increases the firing rate from the carotid sinus, leading to an increase in parasympathetic output and withdrawal of sympathetic output to the heart and peripheral vasculature. This leads to slowing of the heart rate and AV conduction, along with a decrease in systemic vascular resistance Choice E= Vagal efferent nerves only sparsely innervate ventricular myocardium, and carotid sinus massage does not cause any significant change in the ventricular refractory period

-Age: 4 weeks boy -Undergoes cardiac catheterization for closure of a patent ductus arteriosus. The patient was born at 26 weeks gestation due to placental abruption. He has been cared for in the neonatal intensive care unit since delivery. -Pulse oximetry shows 99% in the right hand and left foot. Echocardiography shows a moderately sized patent ductus arteriosus. After catheterization but before closure, blood gases are obtained from several locations. Compared with the partial pressure of oxygen (PaO2) of a normal infant, this patient would be expected to have a higher PaO2 in which of the following locations? a)Ascending aorta b)Descending aorta c)Left atrium d)Pulmonary artery e)Right atrium

Answer: Choice D (Pulmonary artery) -A patent ductus arteriosus (PDA) is a left-to-right shunt that allows continuous flow of oxygenated blood from the aorta into the pulmonary artery -In the pulmonary artery, the oxygenated blood mixes with deoxygenated blood coming from the RV. This leads to a higher partial pressure of oxygen (PaO2) in the pulmonary artery than would normally be seen Choices A and B= In patients with a left-to-right shunt (e.g., ASD, VSD, PDA), the PaO2 in the aorta remain the same. In contrast, if the shunt becomes right-to-left due to increased right-sided pressures (i.e., Eisenmenger physiology) the oxygen concentration in the aorta can be lower than expected. However, patients with Eisenmenger syndrome have cyanosis, which is not seen in this patient Choice C= The LA receives oxygenated blood from the pulmonary circulation, specifically from the alveolar capillaries via the pulmonary veins. Although the volume of blood received by the LA is increased with left-to-right PDA shunts, the PaO2 in the LA is unchanged as all blood passes through the lungs Choice E- The RA receives deoxygenated blood from the systemic circulation, primarily the SVC and IVC. The left-to-right shunt of a PDA is "downstream" of the RA (i.e., in the pulmonary artery), so the PaO2 of RA blood is unchanged

-Age: 20 years man -Comes to the office due to occasional palpitations. He describes them as "skipped beats". He has no chest pain or shortness of breath. Medical history is unremarkable. -On physical exam: he has wide, fixed splitting of the 2nd heart sound (S2). No murmurs are heard. Chest x-ray reveals enlargement of the right heart contour. Echocardiogram is consistent with a congenital heart defect. Repair of this patient's congenital disease is aimed at preventing irreversible changes to which of the following? a)Coronary vessels b)Left atrium c)Left ventricle d)Pulmonary vessels e)Right atrium

Answer: Choice D (Pulmonary vessels) -This patient has wide, fixed splitting of the 2nd heart sound (S2), a characteristic auscultatory finding of an atrial septal defect (ASD). ASD is characterized by left-to-right shunting due to higher pressure in the left atrium compared to the right atrium. The resulting increase in blood flow causes right-sided volume overload, leading to delayed closure of the pulmonic valve (wide S2) that does not vary with respirations (fixed S2) -Right-sided volume overload leads to right atrial and ventricular enlargement, which may be seen as an enlarged right heart contour on chest x-ray. Atrial arrhythmia (e.g., atrial fibrillation), which is likely causing this patient's palpitations, is a common complication of atrial enlargement -With longstanding ASD; the increased blood flow through the pulmonary circulation can result in a laminated medial hypertrophy of the pulmonary arteries. This increases pulmonary vascular resistance (PVR) and causes pulmonary hypertension. Eventually, PVR can exceed systemic vascular resistance, reversing the direction of the intracardiac shunt and creating net right-to-left flow through the ASD. This reversal, known as Eisenmenger syndrome, allows for blood to bypass the pulmonary circulation and can cause adult-onset cyanosis, clubbing, and polycythemia -This pulmonary vascular remodeling is irreversible. Once Eisenmenger syndrome is present, closure of the ASD is not possible, because it can increase right-sided pressure even further and lead to acute right-sided heart failure. Therefore, early closure of a hemodynamically insignificant ASD is needed to avoid irreversible changes to the pulmonary vessels Choices A, B and C= ASD does not typically cause volume overload or enlargement of the left-sided heart chambers because excess blood flow is shunted to the right atrium before reaching the LV. The coronary vessels are also not typically affected Choice E= ASD causes right atrial dilation. However, these changes are mostly reversible because, unlike in the pulmonary arteries and often the RV, significant muscular hypertrophy of the right atrium does not take place

-Age: 35 years primigravid woman -Comes to the clinic during the 2nd trimester. The patient reports that she has had some tiredness, which she attributes to pregnancy. -She takes 1 nap during the day. She has no chest pain, cough, or lower extremity swelling. As a child, the patient was told that she has a murmur but does not know any details and says that she has felt well until recently. -On physical exam: 3/6 midsystolic murmur is heard at the left upper sternal border and S2 is widely split. Diagnosis? a)Benign flow murmur b)Bicuspid aortic valve c)Hypertrophic cardiomyopathy d)Pulmonic stenosis e)Ventricular septal defect

Answer: Choice D (Pulmonic stenosis) -This patient likely has pulmonic valve stenosis (PS), which most commonly occurs as an isolated congenital defect and rarely occurs as an acquired defect (e.g., rheumatic fever, carcinoid syndrome). Severe PS is typically diagnosed early in life due to development of right-sided heart failure, but patients with relatively mild PS often do not present with symptoms (e.g., dyspnea, fatigue) until adulthood. The physiologic changes of pregnancy (e.g., increased plasma volume) can unmask underlying PS -Cardiac auscultation in PS reveals a pulmonic ejection click followed by a harsh crescendo-decrescendo systolic murmur best heard at the left upper sternal border. The stenosis causes the pulmonic valve to close later than usual as it takes longer for the RV to push blood through the narrowed valve. This results in widened splitting of the aortic and pulmonic components of S2. Inspiration draws additional blood into the right side of the heart, which increases the intensity of the murmur and further widens the splitting of S2. -Pulmonary hypertension is another cause of widened splitting of S2 because more time is required for the RV to overcome pulmonary arterial hypertension (i.e., the pulmonic valve opens later and closes later) Choice A= A benign flow murmur can occur in patients with high cardiac output states (e.g., pregnancy) due to increased flow across the aortic and pulmonic valves, however, the murmur is typically low grade (2/6 or less) and abnormal splitting of S2 is not present. Choice B= A bicuspid aortic valve is the most common congenital heart defect seen in adults and can lead to aortic stenosis in relatively young patients (e.g., those in their 50s). However, aortic stenosis causes delayed closure of the aortic valve, leading to narrowed or paradoxical splitting of S2. Choice C= Hypertrophic cardiomyopathy can cause a systolic murmur that mimics that of aortic stenosis and is best heard at the mid-left sternal border. Aortic valve closure may be delayed due to prolonged ejection of blood through the LV outflow tract Choice E= A small- or moderate-sized ventricular septal defect may present in adulthood with symptoms of heart failure. There is typically widened splitting of S2 due to early closure of the aortic valve (less blood is ejected into the aorta because some blood is ejected into the RV). However, a holosystolic murmur best heard at the left lower sternal border is expected

-Age: 35 years man -Previously healthy, evaluated for several episodes of syncope in the past 6 weeks. Physical examination is unremarkable. Echocardiogram shows no structural heart defect. An electrophysiological study is performed during which catheters are passed into the patient's right and left atrium to record atrial electric potentials -Cardiac monitor currently shows normal sinus rhythm. Which of the following is the most likely earliest site of electric activation? a)Junction of left atrium and atrial appendage b)Left atrium near the opening of pulmonary veins c)Right atrium near the opening of inferior vena cava d)Right atrium near the opening of superior vena cava e)Right atrium near the septal cusp of tricuspid valve

Answer: Choice D (Right atrium near the opening of superior vena cava) -The cardiac conduction system consists of the SA node, AV node, bundle of His, right and left bundle branches, and the Purkinje system (which directly activates the ventricular myocardium). In patients with normal sinus rhythm (as in this patient), the SA node acts as the dominant pacemaker site and is the site of earliest electrical activation. The SA node is a compact subepicardial structure that consists of specialized pacemaker cells located at the junction of the RA and the SVC Choices A and B= The SA node is located in the right, not the left, atrium. The area around the opening of the pulmonary veins in the LA is frequently involved in the pathogenesis of atrial fibrillation Choice C= The junction of the RA and IVC is not a major site of electrical activity Choice E= The AV node is located in the RA near the septal cusp of the tricuspid valve

-Age: 54 years man -Comes to the office due to episodic burning substernal chest pain. His pain increases with activity and improves with rest. The patient has a history of hypertension & hyperlipidemia -Smoker (30 years, daily). BP (140/85), pulse (76). Cardiac auscultation reveals a S4 heart sound. Lung and abdominal examinations are unremarkable. An ECG at rest shows left ventricular hypertrophy. A myocardial perfusion scan reveals inducible ischemia of the inferior surface of the heart. Which of the following coronary arteries is most likely occluded in this patient? a)Left anterior descending coronary artery b)Left circumflex coronary artery c)Left main coronary artery d)Right coronary artery e)Right marginal branch

Answer: Choice D (Right coronary artery) -The right and left main coronary arteries arise directly from the root of the aorta and provide the blood supply to the heart. The left main coronary artery divides into the left anterior descending (LAD) and circumflex coronary arteries, which supply most of the anterior and left lateral surfaces of the heart. -In 85-90% of individuals, the right coronary artery gives rise to the posterior descending artery. Such patients are said to have right dominant coronary circulation. The posterior descending artery runs down the posterior interventricular groove and supplies the posterior one third of the interventricular septum and most of the inferior wall of the LV. -The inferior (diaphragmatic) surface of the heart is formed by the LV (two thirds) and RV (one third), which are separated by the posterior interventricular groove. Because most individuals have right dominant circulation, occlusion of the right coronary artery is most likely to cause inducible ischemia of the inferior surface of the heart Choice A= The LAD artery normally supplies the anterior two thirds of the interventricular septum (septal branches), the anterior wall of the LV (diagonal branches), and part of the anterior papillary muscle Choice B= The left circumflex coronary artery usually supplies the lateral and posterior superior walls of the LV via obtuse marginal branches Choice C= In approximately 10% of individuals, the posterior descending artery derives from the circumflex branch of the left main coronary artery (left dominant circulation); therefore, the left main coronary artery is not the artery most likely occluded in this patient Choice E= the right marginal branch of the right coronary artery supplies the wall of the RV and may provide collateral circulation in patients with LAD occlusion

-Age: 59 years African American male -Presents to the ED with crushing chest pain, sweating and lightheadedness. BP (90/60), pulse (48). ECG shows sinus bradycardia and ST segment elevation in leads II, III, and aVF -Occlusion of which of the following coronary arteries is responsible for symptoms? a)Left main coronary artery b)Left anterior descending artery c)Left circumflex artery d)Right coronary artery e)Intramural arteries

Answer: Choice D (Right coronary artery) -This patient presents with symptoms and ECG findings consistent with transmural ischemic injury to the inferior wall of the heart. In 90% of individuals, the posteroinferior wall of the LV is supplied by the posterior descending branch of the right coronary artery (RCA). -The patient's bradycardia and resultant hypotension suggests that there may have been ischemic injury to the sinus node as well (the sinus node normally receives its arterial blood supply from the RCA) Choices A, B and C= The left main coronary artery gives rise to the left anterior descending (LAD) and the left circumflex (LCX) coronary arteries. The LAD supplies the interventricular septum and the anterior wall of the LV. Transmural ischemia of the septum would produce ST elevation mainly in leads V1 and V2. Infranodal (Mobitz type II) 2nd degree or 3rd degree AV block is a possible result as well, but sinus bradycardia would not occur with this type of injury. Transmural ischemia of the anterior LV wall would produce ST elevations mainly in leads V3-V4. An occlusion of the proximal LAD would cause anteroseptal transmural ischemia with ST elevations in leads V1-V4 The LCX supplies the lateral wall of the LV. Transmural ischemia secondary to LCX occlusion would produce ST elevations mainly in leads V5 and V6, and possibly leads I and aVL. An occlusion of the left main coronary artery would therefore be expected to produce widespread transmural ischemia of the interventricular septum and LV, with ST elevations in all the chest leads, and possible also in leads I and aVL Choice E= Intramural arteries in the substance of the myocardium are unlikely sites of occlusion because, unlike the epicardial coronary arteries, they are rarely affected by atherosclerosis. Moreover, if occlusion of intramural arteries did occur, subendocardial ischemia (with associated ST depression) would be more likely than transmural ischemia (with ST elevation)

-Age: newborn -Evaluated for cyanosis immediately after birth. On examination, oxygen saturation is 70% in all 4 extremities and doesn't change despite 100% oxygen administration. The patient's lungs are clear to auscultation, and there are no heart murmurs or gallops. Peripheral pulses are normal. A chest radiography reveals clear lungs and a normal cardiac silhouette but a narrowed mediastinal shadow. Which of the following is the most likely cause of cyanosis in this patient? a)Impaired alveolar-capillary diffusion b)Impaired left ventricular contractility c)Inability of hemoglobin to bind oxygen d)Right-to-left cardiac shunting e)Ventilation-perfusion mismatch

Answer: Choice D (Right-to-left cardiac shunting) -The cyanotic newborn has severe hypoxemia that does not improve with oxygen administration. These findings suggest a congenital heart defect in which deoxygenated blood bypasses the pulmonary circulation and is delivered directly to the systemic circulation (i.e., right-to-left shunt) -The most likely etiology is transposition of the great arteries (TGA), which is characterized by the abnormal origination of the aorta from the right ventricle and the pulmonary artery from the left ventricle. The abnormal position of the aorta directly on top of the pulmonary artery results in a narrow-appearing mediastinum -The anatomy in TGA results in disconnected and parallel systemic and pulmonary circuits with delivery of deoxygenated blood form the right ventricle to the systemic circulation via the aorta (effectively a right-to-left shunt). Cyanosis occurs immediately after birth, and cardiac examination is often normal. Oxygen administration (i.e., hyperoxia test) does not improve cyanosis or hypoxemia in TGA because the oxygenated blood cannot effectively reach the systemic circulation; instead, it is cycled from the lungs to the left side of the heart back to the lungs. Choice A and B= Pulmonary edema secondary to sepsis or heart failure (i.e., impaired LV contractility) can lead to hypoxemia and cyanosis due to impaired alveolar-capillary diffusion leading to intrapulmonary shunting. However, crackles would be expected on examination, and abnormal cardiopulmonary chest x-ray findings (e.g., cardiomegaly, alveolar infiltrates, pleural effusion) are typical Choice C= Congenital methemoglobinemia, which is characterized by the inability to reduce methemoglobin back to hemoglobin, is a rare cause of neonatal cyanosis. Oxygen saturation fails to improve with oxygen administration; however, methemoglobinemia would not explain a narrow mediastinum on chest x-ray. Moreover, pulse oximetry is inaccurate in methemoglobinemia and reads no lower than 85% even if true oxygenation is lower. Choice E= Primary pulmonary pathologies (e.g., respiratory distress syndrome (RDS), pulmonary hypoplasia) are typical causes of ventilation-perfusion mismatch and often present with cyanosis at birth. However, chest x-ray findings are usually abnormal (e.g., ground-glass opacification in RDS), and oxygen administration would be expected to increase oxygen saturation

-Age: 74 years man -Brought to the ED due to acute-onset lower abdominal and back pain that was followed by an episode of syncope 2 hours ago -The pain is sharp and constant. He has had no changes in bowel or bladder function. Other medical conditions: hypertension, hyperlipidemia. The patient has a 50-pack-year history of smoking. -Temp. (36.9C), BP (90/60), pulse (108, regular), RR (18). BMI (34). The patient appears diaphoretic and pale. Heart and lung sounds are normal. Abdominal exam: moderate tenderness to deep palpation in the lower abdomen, there is no guarding or rebound tenderness. Bowel sounds are normal. Diagnosis? a)Acute mesenteric ischemia b)Colonic perforation c)Renal colic d)Ruptured abdominal aortic aneurysm e)Vertebral compression fracture

Answer: Choice D (Ruptured abdominal aortic aneurysm) -Ruptured abdominal aortic aneurysm (AAA) is a surgical emergency involving a full-thickness compromise of the aortic wall with extravasation of blood into the surrounding tissues and spaces. -The acute onset of severe abdominal and back pain is the most common presenting symptom. In general, anterior rupture into the peritoneal cavity is quickly accompanied by syncope, hypotension, and shock, whereas posterior rupture into the retroperitoneum may be temporarily contained, resulting in a delayed onset of hemodynamic instability. Other signs suggesting AAA rupture include abdominal distention, a pulsatile abdominal mass, and umbilical or flank hematoma (indications of retroperitoneal hemorrhage) -Early detection of AAA can be difficult because patients frequently have few symptoms until the AAA either rupture or markedly expands. Therefore, ruptured AAA should be suspected in patients with consistent symptoms and known risk factors, which include advanced age (>60), smoking, male sex, and a history of atherosclerosis or connective tissue disease. CT imaging can confirm the diagnosis, but hemodynamically unstable patients with suspected AAA should go straight to emergent surgical repair Choice A= Acute mesenteric ischemia has similar risk factors to AAA. Patients typically describe severe abdominal pain but have few findings on physical examination (e.g., minimal tenderness) and hypotension and syncope are usually not seen Choice B= Colonic perforation can be due to diverticulitis, malignancy, or complications from colonoscopy. Patients can develop hypotension due to sepsis, but signs of infection (e.g., fever), and peritonitis (e.g., rebound, guarding) are also expected Choice C= Renal colic is characterized by episodic (e.g., every 30 mins) flank or groin pain due to capsular stretch from nephrolithiasis-associated obstruction. Hypotension and syncope are not typical features Choice E= Vertebral compression fractures can cause acute back pain exacerbated by minimal movement (e.g., coughing, bending). Although the pain can occasionally refer to the abdomen, it is most commonly at the midline over the spinous processes. There should not be associated hypotension

-Age: 72 years man -Comes to the clinic due to episodic lightheadedness. The episodes typically occur in the morning when the patient adjusts his shirt collar or tightens his tie, and he briefly feels like he is going to pass out. The patient had a thyroidectomy 5 years ago for a large benign tumor - He also has hypertension and takes amlodipine. BP (135/78), pulse (78). Physical examination reveals no heart murmurs or carotid bruits. ECG shows normal sinus rhythm. The patient is given an ambulatory ECG monitoring device During symptomatic episodes, the device is most likely to show which of the following? a)Atrial fibrillation b)Mobitz type II second-degree heart block c)Paroxysmal supraventricular tachycardia d)Sinus pause e)Ventricular tachycardia

Answer: Choice D (Sinus pause) -This patient with episodes of presyncope (e.g., lightheadedness, feeling of impending loss of consciousness) that occur when he adjusts his shirt collar or necktie most likely has carotid sinus hypersensitivity (CSH). This condition results from overly sensitive carotid baroreceptors that stimulate an excessive vasovagal response to tactile stimulation. CSH is most commonly seen in elderly men; underlying carotid atherosclerotic disease and prior neck surgery (e.g., thyroidectomy) are risk factors ThebaroreceptorresponseinCSHmimicsthenormalresponsetoincreasedBPbutismoreexaggerated,increasedparasympatheticoutputleadstoaprolongedsinuspause (e.g.,3-5seconds)anddecreasedsympathetic outflow causes marked peripheral vasodilation. The resulting delay in ventricular contraction and decreased systemic vascular resistance leads to a profound reduction in BP and transientlossofcerebralperfusionthatmanifestsaspresyncopeor syncope Choices A, C and E= Intermittent tachyarrhythmias, including atrial fibrillation with rapid ventricular response, paroxysmal supraventricular tachycardia (e.g., AV nodal reentrant tachycardia), and ventricular tachycardia, can cause syncope due to loss of cardiac output and reduced cerebral perfusion (of the 3 tachyarrhythmias, ventricular tachycardia is most concerning). However, CSH is more likely in this patient with symptoms associated with tactile stimulation of the carotid sinus Choice B= Advanced AV block (i.e., complete AV block, Mobitz type II second-degree heart block) can cause presyncope or syncope but it is also less likely than CSH in this patient with symptoms associated with tactile stimulation of the carotid sinus

-Age: 38 years woman -Suddenly collapses at home after experiencing severe chest pain and dies before reaching a hospital. Postmortem examination reveals occluding thrombus overlying a ruptured atherosclerotic plaque on the left anterior descending artery. This patient is also found to have thickened mitral valve leaflets with multiple small vegetations on both valvular surfaces and fibrinoid necrosis of arterioles. Other findings include glomerular capillary basement membrane thickening with wire-loop changes. This patient was most likely suffering from? a)Bacterial endocarditis b)Churg-Strauss syndrome c)Granulomatosis with polyangiitis d)Systemic lupus erythematosus e)Thromboangiitis obliterans

Answer: Choice D (Systemic lupus erythematosus) -This patient's postmortem findings are consistent with SLE, an autoimmune disease that affects women of childbearing age and results in immune complex-mediated tissue injury in various organs. -Cardiovascular manifestations of SLE include accelerated atherosclerosis with increased risk of MI. Small arteries and arterioles can also show evidence of vasculitis with fibrinoid necrosis in any tissue. Other cardiac features include peritonitis and less commonly verrucous (Libman-Sacks) endocarditis, characterized by thickened valve leaflets (typically mitral (red arrows), and/or aortic (black arrows)) with multiple small vegetations on both surfaces. The vegetations are composed of sterile platelet thrombi intertwined with strands of fibrin, immune complexes, and mononuclear cells. These vegetations are easily dislodged and can result in systemic embolization -Renal involvement in SLE most often manifests as diffuse proliferative GN, which is characterized by proliferative and necrotizing lesions with crescent formation during active disease. Light microscopy also classically shows diffuse thickening of the glomerular capillary walls with "wire-loop" structures due to subendothelial immune complex deposition Choice A= Bacterial endocarditis is characterized by large, potentially destructive vegetations on the valve cusps that contain fibrin, inflammatory cells, and bacteria (e.g., staphylococci, streptococci). Renal involvement can occur with septic embolization or development of immune complex-mediated GN with hypercellular infiltrate and immune deposits in the glomerular capillary wall. Choice B= Churg-Strauss syndrome (eosinophilic granulomatosis with polyangiitis) is a small-vessel necrotizing vasculitis characterized by allergic rhinitis, asthma, and peripheral eosinophilia. Although renal involvement is not typically prominent, patients can develop necrotizing crescentic GN. Choice C= Granulomatosis with polyangiitis (Wegener granulomatosis) is a small-vessel necrotizing vasculitis characterized by upper respiratory (e.g., sinusitis, otitis media) and lower respiratory (e.g., dyspnea, hemoptysis) symptoms as well as renal disease. Renal involvement typically manifests as focal necrotizing, crescentic GN Choice E= Thromboangiitis obliterans (Buerger disease) is a small to medium vessel vasculitis that typically occurs with cigarette smoking and causes vascular insufficiency in the extremities (e.g., gangrene, autoamputation of digits). It is characterized histologically by intraluminal thrombus and prominent inflammatory cell infiltrate with relative sparing of the vessel wall.

-Age: 35 years man -Evaluated for progressive fatigue and shortness of breath. Recently, he has noticed bilateral leg swelling and abdominal distention despite overall weight loss. -He doesn't smoke, drink or illicit drugs. Despite treatment, the patient dies several weeks later. Autopsy= endocardial thickening with dense fibrous deposits around the tricuspid and pulmonary valves as well as moderate pulmonary valve stenosis -The left-sided cardiac chambers and valves are normal. Measuring the levels of which of the following substances would have helped in diagnosing this patient? a)Plasma cortisol b)Plasma homocysteine c)Plasma phenylalanine d)Urinary 5-hydroxyindoleacetic acid e)Urinary porphobilinogen f)Urinary vanillylmandelic acid

Answer: Choice D (Urinary 5-hydroxyindoleacetic acid) -The autopsy findings- endocardial thickening and fibrosis of tricuspid and pulmonary valves- are characteristic of carcinoid heart disease. Carcinoids are well-differentiated neuroendocrine tumors found most commonly in the distal small intestine and proximal colon, with a strong propensity for metastasis to the liver. -These tumors secrete several products (including histamine, serotonin, and vasoactive intestinal peptide) that are metabolized in the liver. In patients with liver metastasis, these hormones are released directly into the systemic circulation, leading to carcinoid syndrome -Carcinoid heart disease is caused by excessive secretion of serotonin, which stimulates fibroblast growth and fibrogenesis. Pathognomic plaque-like deposits of fibrous tissue can occur most commonly on the endocardium, leading to tricuspid regurgitation, pulmonic valvulopathy, and right-sided heart failure (e.g., ascites, peripheral edema). Endocardial fibrosis and thickening are generally limited to the right heart as vasoactive products are inactivated distally by pulmonary vascular endothelial monoamine oxidase -5-hydroxyindoleacetic acid (5-HIAA) is an end product of serotonin metabolism, and elevated 24-hour urinary 5-HIAA levels are helpful in diagnosing suspected carcinoid syndrome Choice A= Measurement of plasma cortisol levels is used for diagnosing adrenal insufficiency and Cushing syndrome Choice B= Inherited or acquired elevation of plasma homocysteine levels may contribute to arterial and venous thrombosis and to the development of atherosclerosis Choice C= Plasma phenylalanine may be elevated in patients with phenylalanine hydroxylase deficiency (phenylketonuria), resulting in central nervous system damage/intellectual disability Choice D= Urinary porphobilinogen may be elevated in the porphyrias, which are caused by deficiencies of the heme synthesis enzymes. The porphyrias may produce cutaneous lesions, skin photosensitivity, or attacks of abdominal pain and neurological disturbances (acute intermittent porphyria) Choice F= Vanillylmandelic acid is a byproduct of norepinephrine and epinephrine, and can be used to detect neuroblastoma and other tumors of neural crest origin

-Age: 44 years man -History of IV drug use came to the ED due to fever and chills. Multiple blood culture sets were positive for Staphylococcus aureus, and the patient was diagnosed with infective endocarditis. He was successfully treated with a long course of antibiotics -Six months later, he returns to the clinic for a follow-up evaluation. The patient has no symptoms and reports good exercise tolerance. -He is afebrile, BP (140/62), pulse (82), RR (16). Chest auscultation reveals a decrescendo diastolic murmur over the 3rd intercostal space along the left sternal border. Echocardiogram shows severe aortic regurgitation, likely as a sequela to the prior infection. Which of the following changes is most responsible for maintaining cardiac output in the setting of this valvular abnormality? a)Concentric left ventricular hypertrophy b)Decrease in left ventricular preload c)Increase in aortic elasticity d)Increase in left ventricular afterload e)Increase in left ventricular stroke volume f)Sustained increase in heart rate

Answer: Choice E (Increase in left ventricular stroke volume) -This patient has developed chronic aortic regurgitation (AR) as a consequence of infective endocarditis. Incomplete closure of the aortic valve during diastole allows a portion of LV stroke volume to leak back into the LV; in severe AR this regurgitant flow volume may be up to 50% of total forward stroke volume. LV dilation caused by the increase in LV end-diastolic volume (LVEDV) (volume overload) triggers eccentric hypertrophy, which involves ventricular wall lengthening due to the addition of myocardial contractile fibrils organized in series. -This compensatory response causes an increase in stroke volume that is able to maintain cardiac output, and allows for a relatively long asymptomatic period (e.g., several years) in most patients with chronic AR -However, the compensatory eccentric hypertrophy in chronic AR is overall maladaptive. Over time, progressive left ventricular dilation leads to overwhelming wall stress with decreased stroke volume and eventual left ventricular failure Choice A= Concentric hypertrophy occurs due to conditions that create ventricular pressure overload (e.g., hypertension, aortic stenosis); it involves ventricular wall thickening due to the addition of myocardial contractile fibrils in parallel. Although some concentric hypertrophy may occur in AR, eccentric hypertrophy due to volume overload is the predominant change Choice B= In AR, left ventricular preload or LVEDV is increased, rather than decreased. This helps the ventricle maintain cardiac output via the Frank-Starling mechanism. Decreased preload would cause a reduction in cardiac output Choice C= Increased aortic elasticity (i.e., decreased compliance or increased aortic stiffening) occurs with aging and results in higher systolic blood pressure, widened pulse pressure, and reduced cardiac output Choice D= In chronic AR, there is an effective increase in LV afterload due to the enlarged chamber size (increases wall stress) and high peak systolic pressures. This increases left ventricular work, making it more difficult to maintain cardiac output and eventually leads to heart failure Choice F= Because cardiac output is maintained by an increase in stroke volume, there is typically minimal increase in heart rate in chronic AR. However, in acute AR, eccentric hypertrophy does not have time to develop and the capacity to increase stroke volume is limited. The resulting decrease in cardiac output triggers a reflexive increase in heart rate, but this response is typically insufficient, and most patients develop severe cardiogenic shock with pulmonary edema

-Age: 62 years man -Comes to the office due to chest pain over the last 6 months. He describes pressure-like pain in the substernal area when walking fast or climbing stairs that gradually subsides when he stops. The patient does not smoke or use illicit drugs. -He has no family history of heart disease or sudden death. Physical exam: mid-systolic murmur at the upper sternum and slow-rising carotid pulses. After initial evaluation, cardiac catheterization is performed and reveals 30% stenosis of the mid-left anterior descending artery. A 50mmHg pressure gradient is seen when a catheter is passed across the aortic valve. Which of the following is the most important contributor to this patient's presenting symptoms? a)Decrease in coronary blood supply due to lipid-rich plaque b)Decreased in coronary blood supply due to vasospasm c)Decrease in left ventricular compliance d)Increase in left ventricular stroke volume e)Increase in left ventricular wall stress

Answer: Choice E (Increase in left ventricular wall stress) -This patient's slow-rising carotid pulses, mid-systolic murmur, and large pressure gradient across the aortic valve are consistent with severe aortic stenosis (AS). Patients can remain asymptomatic for long periods but eventually develop classic symptoms of severe AS; dyspnea is typically the first manifestation, followed by angina, presyncope or syncope, and overt heart failure -The symptoms of severe AS usually occur with exertion, as the capacity to increase stroke volume and cardiac output is limited due to the fixed LV outflow obstruction (Choice D). This outflow obstruction also leads to higher chamber pressures and increased wall stress during systole and diastole. The rise in systolic wall stress increases myocardial oxygen demand while the increased diastolic wall stress reduces the coronary perfusion pressure gradient, leading to decreased myocardial perfusion. Increased LV mass due to concentric hypertrophy further increases myocardial oxygen demand and contributes to anginal symptoms. Choice A= Reduced coronary blood supply due to flow-limiting atherosclerotic plaque can cause angina in patients with obstructive coronary artery disease. Stenosis >70% is typically necessary for anginal symptoms to develop; this patient's 30% stenosis is not clinically significant. Angina often occurs in severe AS in the absence of coronary artery disease. Choice B= Decreased coronary blood supply due to vasospasm is seen in patients with vasospastic (Prinzmetal) angina. Coronary vasospasm typically affects relatively young patients (age <50) and is not related to the angina that occurs with severe AS Choice C= Decreased LV compliance occurs over time in AS due to prolonged high-pressure load created by the stenotic valve. This can result in heart failure (e.g., fatigue, dyspnea) due to diastolic dysfunction, but would be less likely to induce angina symptoms

-Age: 63 years man -Comes to the ED due to increasing dyspnea. He was unable to mow his lawn the previous day due to shortness of breath. History: hypertension, 20-pack-year smoking history -He is retired and traveled to Hawaii 2 months ago. BP (160/86), pulse (94), RR (24); oxygen saturation is 94% on room air. Physical examination reveals bilateral lung crackles and occasional wheezes. -Lab testing: elevated brain natriuretic peptide levels. Which of the following is the primary trigger for the release of this substance in this patient? a)Activation of alveolar juxtacapillary receptors b)Decrease in renal perfusion pressure c)Decrease in vena cava blood return d)Focal myocardial necrosis e)Increase in ventricular wall stress f)Stretching of aortic baroreceptors

Answer: Choice E (Increase in ventricular wall stress) -In addition to their contractile and electrical conduction properties, cardiomyocytes possess endocrine capabilities to help regulate blood volume. Volume overload leads to stretching and increased wall stress of the ventricular wall. In response, the ventricular myocardium releases brain natriuretic peptide (BNP); similarly, the atrial myocardial releases atrial natriuretic peptide (ANP) in response to atrial stretching -These natriuretic peptides stimulate both venous and arterial vasodilation to decrease cardiac preload and afterload and reduce strain on the myocardium. In addition, ANP and BNP stimulate salt and water excretion by the kidneys to facilitate diuresis -Clinically, BNP has diagnostic utility in patients with shortness of breath of unknown cause. For patients in whom the serum BNP level is not elevated, shortness of breath is unlikely to be due to heart failure Choice A= Alveolar juxtacapillary receptors increase respiratory rate in response to pathologic alveolar processes (e.g., pulmonary edema, pneumonia) Choice B= Decreased renal perfusion pressure triggers the juxtaglomerular apparatus to release renin, activation of RAAS stimulates vasoconstriction and fluid retention, opposing the effects of natriuretic peptides Choice C= Decrease in vena cava blood return reduces preload and leads to a consequent reduction in cardiac output. The reduced cardiac output is sensed by carotid baroreceptors, and an increase in sympathetic drive is triggered to stimulate vasoconstriction and increased HR and contractility Choice D= Focal myocardial necrosis causes release of troponin into the bloodstream, which acts as a sensitive marker of myocardial damage. BNP levels often increase during MI; however, this increase is likely secondary to ventricular dysfunction leading to volume overload and myocardial stretch Choice F= Elevated BP causes stretching of aortic baroreceptors, which triggers a reduction in sympathetic drive to alleviate vasoconstriction and reduce myocardial contractility

-Age: 63 years man -Comes to the ED due to dyspnea. Over the past several days, the patient has experienced progressively worsening shortness of breath while walking his dog around the block. -In addition, he could not breathe while lying in bed last night and fell asleep only after moving to a recliner. The patient had an acute MI 2 years ago and has been nonadherent with his medications and follow-up appointments. -Temp. (36.7C), BP (122/74), pulse (94), RR (22). Physical examination shows bibasilar lung crackles, jugular venous distention, and bilateral pitting edema in the lower extremities. Chest x-ray reveals cardiomegaly and pulmonary venous congestion. Which of the following factors is most likely contributing to this patient's symptoms? a)Decreased arteriolar resistance b)Decreased plasma renin activity c)Decreased ventricular end-diastolic pressure d)Increased plasma brain natriuretic peptide concentration e)Increased sympathetic nervous system activity

Answer: Choice E (Increased sympathetic nervous system activity) -This patient's progressive dyspnea, orthopnea, and lower extremity swelling are consistent with acute decompensated heart failure. Heart failure occurs when a structural or functional cardiac disorder impairs ventricular filling or ejection filling of blood, causing reduced cardiac output and elevated filling pressure. The reduced cardiac output causes an initial drop in BP, which triggers compensatory neurohumoral activity mainly via stimulation of the sympathetic nervous system and RAAS -These adaptations function together in an effort to improve organ perfusion by increasing cardiac chronotropy and inotropy (improves cardiac output), promoting arterial and venous vasoconstriction (maintains perfusion pressure), and expanding the extracellular fluid compartment (improves circulating blood volume). However, over time, these responses lead to adverse consequences that perpetuate a downward spiral of progressive cardiac deterioration. Chronic hemodynamic stress (e.g., increased preload and afterload) and prolonged exposure to sympathetic drive (catecholamines) and RAAS stimulation (angiotensin II, aldosterone) lead to deleterious remodeling with worsening cardiac function and eventual symptomatic decompensation Choices A and B= Decreased cardiac output in heart failure stimulates increased RAAS activity and sympathetic output, resulting in increased arteriolar resistance Choice C= Ventricular end-systolic and end-diastolic pressures are increased in heart failure due to increased preload and afterload Choice D= Heart failure triggers the myocardium to secrete natriuretic peptides (i.e., ANP, BNP) in response to myocardial stretch from volume overload. These peptides stimulate vasodilation and increase salt and water excretion to improve the symptoms of heart failure; however, these positive effects are overpowered by the negative effects of increased sympathetic drive and RAAS activation

-Age: 53 years mana -Comes to the office with progressive exertional dyspnea. He has smoked 2 packs of cigarettes per day for the last 35 years. Physical examination shows increased anteroposterior diameter of his chest. Auscultation reveals decreased breath sounds and scattered wheezes throughout his lungs. Examination of his extremities is unremarkable. An echocardiogram reveals moderate dilation of the right ventricle and increased central venous pressure. The absence of peripheral edema in this patient is best explained by which of the following compensatory mechanisms? a)Decreased capillary permeability b)Decreased circulating aldosterone levels c)Decreased interstitial fluid pressure d)Increased plasma oncotic pressure e)Increased tissue lymphatic drainage

Answer: Choice E (Increased tissue lymphatic drainage) The patient has dyspnea, signs of lung hyperinflation, and a heavy smoking history, suggesting chronic obstructing pulmonary disease (COPD). Chronic hypoxia in COPD can lead to pulmonary vasoconstriction, increased pulmonary arterial pressure, and right heart failure (i.e., cor pulmonale), which is confirmed by this patient's echocardiogram. In turn, right heart failure leads to increased central venous pressure and excessive capillary hydrostatic pressure, which predisposes him to developing peripheral edema Fluid movement across the capillary wall (filtration) into the interstitium is dependent on hydrostatic pressure, oncotic pressure, and capillary permeability. It is tightly balanced by lymphatic drainage, which returns interstitial fluid to the vasculature. Peripheral edema (accumulation of excess fluid in the interstitial space) develops when transcapillary plasma filtration exceeds the resorptive capacity of the lymphatics. Factors that favor edema include: -Elevated capillary hydrostatic pressure: Higher capillary pressures increase plasma filtration into the interstitium, particularly in dependent tissues. Causes include arteriolar dilation (e.g., dihydropyridine calcium channel blockers) and impaired venous return (e.g., high central venous pressure, venous thrombosis) -Decreased plasma oncotic pressure: Plasma proteins (e.g., albumin) generate oncotic pressure that pulls interstitial fluid back into the capillary bed venules. Edema develops in conditions with decreased albumin levels (e.g., nephrotic syndrome, cirrhosis, malnutrition) -Sodium and water retention: increased intravascular volume raises capillary hydrostatic pressure and decreases plasma oncotic pressure (dilutes plasma proteins). Causes include kidney disease and heart failure -Lymphatic obstruction: decreased lymphatic return impairs removal of excess interstitial fluid. Common causes of lymphatic obstruction include filariasis, invasive malignancies, and iatrogenic etiologies (e.g., surgical lymph node dissection and radiation therapy) Increased capillary fluid transudation raises interstitial hydrostatic pressure, resulting in increased lymphatic drainage. This compensatory response can delay the development of clinically apparent edema. However, progressive heart failure, cirrhosis, or renal failure can eventually overwhelm lymphatic return capacity and produce significant edema Choices A and D= Although increased plasma oncotic pressure and decreased capillary permeability oppose edema formation, neither is associated with COPD. Severe COPD may cause polycythemia due to hypoxia, but erythrocyte concentration does not affect plasma oncotic pressure Choice B= In patients with heart failure, activation of RAAS leads to increased aldosterone levels that cause sodium and water retention (exacerbating edema formation). Volume expansion causes increased atrial/brain natriuretic peptide release, which can partially counteract the effects of RAAS activation; however, aldosterone levels remain above normal in patients with heart failure Choice C= Decreased interstitial fluid pressure favors flow into the interstitium and would contribute to the development of peripheral edema

-Age: 66 years man -Comes to the office for follow-up. The patient has a history of nonischemic cardiomyopathy and has had persistent dyspnea despite taking the maximal dosage of his heart failure medications. He has had no lightheadedness or chest pain. BP (133/72), pulse (76). -Physical examination shows normal lung sounds and no lower extremity edema. His medical treatment is modified with the addition of a combination pill that inhibits neprilysin and blocks angiotensin II receptors. Which of the following is the most likely effect of this medication? a)Decreased renal free-water reabsorption b)Decreased venous compliance c)Increased myocardial contractility d)Increased peripheral arterial resistance e)Increased urinary sodium excretion

Answer: Choice E (Increased urinary sodium excretion) -Medical therapy for heart failure (e.g., beta blockers, ACE inhibitors, angiotensin II receptor blockers) is primarily targeted toward the inhibition of detrimental neurohumoral pathways involving the sympathetic nervous system and RAAS. However, an additional strategy involves augmenting beneficial counter-regulatory mechanisms, such as those involving natriuretic peptides -Atrial natriuretic peptide (ANP) and brain natriuretic peptide (BNP) are released from the myocardium in response to high atrial and ventricular pressures (myocardial stretch). These peptides induce vasodilation and diuresis by antagonizing the actions of RAAS and also protect against the deleterious myocardial remodeling and fibrosis that occur in heart failure. Neprilysin is a metalloprotease that cleaves and inactivates both ANP and BNP. Therefore, medications that inhibit neprilysin (e.g., sacubitril) lead to increased levels of ANP and BNP and promote beneficial effects in heart failure -However, because neprilysin is also responsible for inactivating angiotensin II, inhibition of neprilysin further stimulates deleterious vasoconstriction and fluid retention via increased angiotensin II levels. Therefore, in treating heart failure, neprilysin inhibitors are combined with an angiotensin II receptor blocker (e.g., sacubitril-valsartan) to mitigate these negative effects Choice A= Increased levels of ANP and BNP stimulate the excretion of both salt and water. Decreased reabsorption of free water (water without solute) can be achieved with medications that block vasopressin-2 receptors (e.g., tolvaptan) on the renal collecting tubule Choices B and D= The vasodilatory effects of ANP and BNP increase (rather than decrease) venous compliance and decrease (rather than increase) peripheral arterial resistance, leading to a reduction in both cardiac preload and afterload and a consequent reduction in cardiac work. These effects would be counteracted by angiotensin II-induced vasoconstriction, but angiotensin II activity is blocked by combining the neprilysin inhibitor with an angiotensin II receptor blocker Choice C= Neprilysin inhibition combined with angiotensin II receptor blockade improves cardiac output in heart failure via a reduction in the cardiac work required to attain a certain stroke volume; however, cardiac contractility is not significantly affected.

-Age: 10 years boy -Brought to the clinic by his parents due to chronic fatigue, shortness of breath, and failure to gain weight. The child was recently adopted from an orphanage in Asia. He weighs 20kg. Vital signs are within normal limits. -Physical examination reveals a loud systolic murmur. Echocardiogram shows pulmonary stenosis and subaortic ventricular septal defect with deviation of the origin of the aorta to the right. The mother describes occasional episodes of severe dyspnea and cyanosis, during which the child quickly assumes a squatting position. -Which of the following mechanisms during squatting relieves this child's symptoms? a)Decreasing left ventricular workload b)Decreasing pulmonary blood flow c)Decreasing pulmonary vascular resistance d)Improving lung compliance e)Increasing systemic vascular resistance

Answer: Choice E (Increasing systemic vascular resistance) -This child has Tetralogy of Fallot (TOF) with ventricular septal defect, overriding aorta, pulmonic stenosis and right ventricular hypertrophy. The pulmonic stenosis and overriding aorta cause a relatively low ratio of systemic vascular resistance (SVR) to pulmonary vascular resistance (PVR). This low SVR-PVR ratio allows the deoxygenated right ventricular output to take the low-resistance route to the systemic circulation, leading to acute hypoxemia ("Tet spells"). Tet spells are most common in infancy but can occur at any age in children with unrepaired TOF. The mechanism of Tet spells has not been clearly defined but may be caused by acute changes of the infundibular septum leading to more RV outflow tract obstruction -Children with TOF quickly learn to squat to relieve Tet spells (infants who cannot squat can be placed in the knee-to-chest position by their caregiver). This posture quickly increases SVR without changing PVR, increasing the SVR/PVR ratio. The increased SVR forces a higher proportion of RV output to enter the pulmonary circulation and oxygenated in the pulmonary capillary beds, increasing arterial oxygen concentration and relieving the Tet spells Answer: Choice E (Increasing systemic vascular resistance) -This child has Tetralogy of Fallot (TOF) with ventricular septal defect, overriding aorta, pulmonic stenosis and right ventricular hypertrophy. The pulmonic stenosis and overriding aorta cause a relatively low ratio of systemic vascular resistance (SVR) to pulmonary vascular resistance (PVR). This low SVR-PVR ratio allows the deoxygenated right ventricular output to take the low-resistance route to the systemic circulation, leading to acute hypoxemia ("Tet spells"). Tet spells are most common in infancy but can occur at any age in children with unrepaired TOF. The mechanism of Tet spells has not been clearly defined but may be caused by acute changes of the infundibular septum leading to more RV outflow tract obstruction -Children with TOF quickly learn to squat to relieve Tet spells (infants who cannot squat can be placed in the knee-to-chest position by their caregiver). This posture quickly increases SVR without changing PVR, increasing the SVR/PVR ratio. The increased SVR forces a higher proportion of RV output to enter the pulmonary circulation and oxygenated in the pulmonary capillary beds, increasing arterial oxygen concentration and relieving the Tet spells

-Age: 71 years man -Comes to the ED due to sudden-onset vision loss in his left eye for one hour. He has a history of coronary artery disease, hypertension and type 2 diabetes -BP (145/80), pulse (72). On examination, the patient can see only hand motions through the left eye. Fundoscopic evaluation of the eye shows a cherry-red spot in the macula with surrounding retinal whitening. Cranial nerve examination is otherwise unremarkable. -There is a left-sided neck bruit on cardiovascular examination. Which of the following is the most likely path of the embolus causing this patient's symptoms? a)External carotid artery, facial artery, ophthalmic artery, retinal artery b)External carotid artery, ophthalmic artery, retinal artery c)External carotid artery, temporal artery, retinal artery d)Internal carotid artery, anterior cerebral artery, retinal artery e)Internal carotid artery, ophthalmic artery, retinal artery

Answer: Choice E (Internal carotid artery, ophthalmic artery, retinal artery) -Retinal artery occlusion (RAO) is an important cause of acute, painless, monocular vision loss. Thromboembolic complications of atherosclerosis in the internal carotid are the most common cause of RAO. The retinal artery is one of the first branches of the ophthalmic artery, which receives its supply from the internal carotid (Choice B). The retinal artery then travels within the retinal nerve to supply the inner retina and the surface of the optic nerve. Choices A and C= There are a number of anastomoses between branches of the ophthalmic artery and branches of the external carotid, including the facial artery and temporal artery, but neither branch anastomoses with the retinal artery or supplies the retina Choice D= The anterior cerebral artery is a branch off the circle of Willis that supplies blood to the midline portions of the frontal and superior medial parietal lobes. Its occlusion can lead to loss of strength and sensation in the contralateral lower extremity, incontinence, and transcortical motor aphasia

-On physical examination, jugular venous pressure is elevated, breath sounds are decreased at the right lung base with dullness to percussion, and 2+ bilateral lower extremity pitting edema is present. Chest x-ray shows cephalization of the blood vessels, Kerley B lines and right pleural effusion. Thoracentesis is performed for pleural fluid analysis. In comparison to plasma fluid, which of the following pleural fluid findings is most likely to be observed? a)High amylase content b)High protein content c)High white blood cell count d)Low glucose content e)Low lactate dehydrogenase content

Answer: Choice E (Low lactate dehydrogenase content) -In determining the etiology of a pleural effusion, differentiating whether the effusion is transudative or exudative is an important step. The light criteria allows differentiation of these two types of effusion via analysis of the protein and LDH levels of the fluid -Transudative pleural effusions form due to an imbalance in hydrostatic or oncotic pressure. Heart failure causes increased hydrostatic pressure in the pulmonary circulation due to backup of blood flow from the failing LV. This increased pressure leads to leakage of fluid from the pulmonary capillaries and causes a transudative pleural effusion, which is characterized by low protein and low LDH levels compared to serum values -In contrast, exudative effusions result from inflammatory disruption of vascular permeability (e.g., infection, malignancy) and demonstrate high protein levels (Choice B) due to increased capillary permeability and reduced sieving of proteins as fluid traverses the capillary wall. LDH levels also tend to be high in exudative effusions Choice A= High amylase content is suggestive of a pleural effusion due to pancreatitis, pancreatic pseudocyst, or esophageal rupture Choice C= High leukocyte count is typical of a complicated parapneumonic effusion or some types of malignant pleural effusion. Transudative effusions do not demonstrate high leukocyte count Choice D= Low glucose count is seen in the setting of malignant or infectious pleural effusion due to consumption of glucose by bacteria, neutrophils, and/or malignant cells

-Age: 63 years man -Comes to the ED with 2 hours of chest pain and shortness of breath. The pain started as a lower sternal discomfort that he attributed to indigestion, but then spread all over the anterior chest. The patient has never had similar pain before. -History: hypertension, hyperlipidemia, and diet-controlled type 2 diabetes. The patient is an ex-smoker with a 20-pack-year history. On cardiac auscultation, a systolic murmur is heard. Basilar crackles are present on both sides. -After initial evaluation, the patient is taken to the cardiac catheterization lab and is found to have a coronary artery occlusion, which is successfully revascularized. The next morning, the patient appears comfortable and wants to go home. The lungs are clear on auscultation, and no heart murmur is present. -The murmur heard during initial presentation is most likely explained by a pathologic process involving? a)Aortic root b)Aortic valve leaflets c)Interventricular septum d)Mitral valve chordae e)Papillary muscle

Answer: Choice E (Papillary muscle) -This patient with myocardial infarction developed a new systolic murmur that resolved following revascularization, which is consistent with mitral regurgitation (MR) due to papillary muscle dysfunction. -MI can cause ischemia of the papillary muscle and the adjacent LV wall on which it is mounted. This results in hypokinesis and outward displacement of the papillary muscle, creating increased tension on the attached chordae tendineae and preventing complete closure of the corresponding mitral valve cusp. Timely restoration of adequate blood supply with coronary revascularization restores papillary muscle and LV wall motion, often leading to resolution of the MR -In contrast to papillary muscle dysfunction, papillary muscle rupture is a mechanical complication of MI that occurs acutely or within 3-5 days and leads to acute MR, it does not resolve with coronary revascularization and typically requires surgical repair Choices A and B= Aortic root and aortic valve leaflet function are not directly affected by myocardial ischemia and would not explain a systolic murmur during acute MI Choice C= Rupture of the interventricular septum can occur acutely or within 3-5 days of acute MI. Patients have sudden-onset hypotension, dyspnea, pulmonary edema, and a harsh, holosystolic murmur at the left sternal border with palpable thrill. The condition does not resolve with coronary revascularization and typically requires surgical repair Choice D= Mitral valve chordae tendineae are not directly affected by myocardial ischemia; however, rupture resulting in acute MR can occur in patients with myxomatous mitral valve disease (mitral valve prolapse), rheumatic fever, or endocarditis

-Age: 62 years woman -No significant medical history, brought to the ED due to fatigue and substernal chest pain that began 6 hours ago. On arrival, ECG is consistent with ST-segment elevation myocardial infarction. Coronary angiography shows right-dominant coronary circulation and a thrombotic occlusion of the proximal right coronary artery, which supplies blood to the patient's right ventricle and the inferior and posterior portions of the left ventricle. -A drug-eluting stent is placed, and she is discharged 2 days later without complication. After 6 weeks, the patient follows up in the clinic and undergoes echocardiography that shows left ventricular contractile dysfunction with normal right ventricular function. Which of the following characteristics of the right ventricle compared to the left ventricle best explains these echocardiography findings? a)Higher resting oxygen extraction b)Less dependence on collateral circulation c)Less ischemic preconditioning d)More active fibroblast proliferation e)Perfusion during both systole and diastole

Answer: Choice E (Perfusion during both systole and diastole) MI involving the LV commonly leads to scarring and a sustained reduction in contractile function; however, contractile function of the RV almost always returns to normal following MI. Some of the reasons for protection of the RV from infarction include: -The relatively small muscle mass and afterload of the RV creates less oxygen demand and necessitate lower oxygen extraction at rest. This allows for a large capacity to increase oxygen extraction during periods of ischemia -The relatively low systolic pressure of the RB (e.g., <25mmHg) allows for coronary perfusion throughout the cardiac cycle, during MI, this enables the RV to pull blood flow from collateral vessels during both systole and diastole. In contrast, the high systolic pressure of the LV (e.g., 120mmHg) blocks coronary blood flow of the LV walls during systole, allowing for perfusion during diastole only -These factors likely contribute to the typical clinical course of MI involving the RV, which is acute RV contractile dysfunction often leading to profound hypotension and hemodynamic instability; acute mortality risk is high, but there is minimal long-term RV dysfunction for those who survive. Choice A= The large muscle mass of the LV necessitates higher oxygen extraction at rest, with relatively little capacity to increase oxygen extraction during periods of ischemia Choice B= The most robust collateral flow occurs from the left to right coronary arteries, suggesting the RV has a more developed collateral circulation and may be more dependent on collateral flow than the LV Choice C= Ischemic preconditioning refers to the myocardium being repeatedly exposed to mildly ischemic states (e.g., during periods of exercise), which stimulates adjustments (e.g., collateral circulation development, cellular improvement in oxygen utilization) that allow future ischemic states to be better tolerated. The RV is thought to experience heightened ischemic preconditioning compared to the LV, contributing to protection of the RV from infarction Choice D= Fibroblast proliferation is more prominent in the muscular LV, where repair of a larger mass of tissue is needed.

-Age: 78 years woman -Hospitalized due to acute myocardial infarction. The patient had been having intermittent chest pain for 3 days and came to the hospital when the pain became unremitting. -She has type 2 diabetes; her medical follow-up has been poor. On the third day of hospitalization, the patient has sudden-onset shortness of breath due to pulmonary edema. Echocardiography confirms severe mitral regurgitation. -She is taken immediately to surgery, which reveals rupture of the posteromedial papillary muscle. This patient's finding typically suggests compromised blood flow through which of the following coronary arteries? a)Conus branch b)Diagonal branch c)Left anterior descending d)Obtuse marginal e)Posterior descending

Answer: Choice E (Posterior descending) -The mitral valve apparatus consists of the mitral valve annulus, anterior and posterior mitral leaflets, and chordae tendineae, which are tethered to the LV wall via the anterolateral and posteromedial papillary muscles. Pathologic processes that disrupt any of these structures can lead to improper alignment of the mitral leaflets, resulting in mitral regurgitation and pulmonary edema -The patient's mitral regurgitation is the result of papillary muscle rupture, a life-threatening mechanical complication typically occurring 3-5 days following acute myocardial infarction. The anterolateral papillary muscle has a dual blood supply from the LAD and LCx. In contrast, the posteromedial papillary muscle is supplied solely by the posterior descending artery, a branch of the right coronary artery (right dominant circulation) or LCx artery (left dominant circulation). Because blood flow to the posteromedial papillary muscle depends on a single artery, it is more likely to become ischemic and rupture following MI Choice A= The conus artery is an early branch of the proximal right coronary artery; it supplies the anterior area of the interventricular septum and conus of the pulmonary artery Choice B= Diagonal arteries branch from the LAD artery and supply blood to the anterolateral walls of the LV Choice C= The LAD artery supplies the anterior two thirds of the interventricular septum, anterior wall of the LV, and anterolateral papillary muscle. Abrupt occlusion of the LAD rarely results in ischemic rupture of the anterolateral papillary muscle as this muscle also receives blood supply from the LCx Choice D= The obtuse (left) marginal artery is a branch of the LCx artery and supplies the lateral wall of the LV

-Age: 34 years woman -Being evaluated for fatigue and progressive exertional dyspnea. She goes mountain biking with friends a few times a month and recently noticed that she has to stop and rest more frequently when traveling uphill. The patient has no significant medical history/no medications -She occasionally drinks alcohol, doesn't smoke. Her father died of an MI at age 72. Lung auscultation is normal. An echocardiogram shows an enlarged coronary sinus. Which of the following is the most likely cause of the observed finding in this patient? a)Aortic dissection b)Coronary artery disease c)Hypertrophic cardiomyopathy d)Pericardial effusion e)Pulmonary hypertension

Answer: Choice E (Pulmonary hypertension) -Most of the venous drainage from the myocardium traverse the coronary sinus (CS), which delivers deoxygenated blood to the right atrium. The CS runs transversely in the left atrioventricular groove on the posterior aspect of the heart, and opens into the RA between the inferior vena cava and tricuspid valve at the CS orifice -Because the CS communicates freely with the right atrium, it will become dilated by any factor that causes dilation of the RA. The most common cause of coronary sinus dilation evident on echocardiography is elevated right-sided heart pressure secondary to pulmonary hypertension. CS dilation also occurs in patients with anomalous venous drainage into the CS, including persistent left superior vena cava and total anomalous pulmonary venous return. Choice A= Aortic dissection can occur in patients with severe hypertension. Such patients may have elevated left heart pressures, but elevated right-sided heart pressures with dilation of the CS would be unlikely Choice B= The presence of asymptomatic coronary artery disease usually does not change the size of CS. Long-standing left ventricular systolic dysfunction due to prior MI and ischemic cardiomyopathy can lead to secondary pulmonary hypertension, with the possibility of CS dilation over time Choice C= Hypertrophic cardiomyopathy usually manifests as asymmetric septal hypertrophy with or without LV outflow tract obstruction. CS dilation should not occur unless there is superimposed biventricular heart failure Choice D= Pericardial effusion can cause extrinsic compression of the CS

-Age: 36 years woman -Brought to the ED with sudden-onset right side weakness and speech difficulty. During the last 3 weeks, the patient has experienced progressive fatigue, malaise, and low-grade fevers. Despite the symptoms, she did not seek medical attention and did not take any medications. -She had a dental extraction 5 weeks ago, which was uncomplicated. The patient has never previously been significantly ill or hospitalized. She works as a receptionist at a legal firm and has never traveled outside the US -She doesn't smoke. The patient is admitted to the hospital, but despite adequate resuscitative measures, she dies 2 hours later. Gross autopsy= large, friable irregular masses attached to the atrial surface of a valve; Underlying condition that predisposed this patient to developing the cardiac changes? a)Atrial septal defect b)Coronary artery disease c)Hypertrophic cardiomyopathy d)Mitral annular calcification e)Regurgitant mitral valve prolapse

Answer: Choice E (Regurgitant mitral valve prolapse) -The patient had subacute infective endocarditis (IE) complicated by an embolic stroke. The large, friable irregular masses seen on autopsy are most likely vegetations and are the most probable source of this patient's left middle cerebral artery embolus (e.g., right-sided weakness, speech difficulty) -The mitral valve is the valve most frequently affected by IE. Mitral valve prolapse (MVP) is the most common underlying valvular disease predisposing to the development of IE in developed countries, particularly when it is associated with coexistent mitral regurgitation. Microscopic deposits of platelets and fibrin occur spontaneously in individuals with valvular disease secondary to endocardial injury from turbulent blood flow. These deposits then become colonized by microorganisms (e.g., viridans streptococci) during episodes of bacteremia (e.g., following dental extraction) -In developing countries, the most common acquired heart disease leading to IE development is rheumatic heart disease (RHD). RHD is a late (10-20 years) complication of acute rheumatic fever (e.g., migratory arthritis, carditis/valvulitis, erythema marginatum). RHD was historically considered a frequent predisposing factor for IE, however, MVP, valvular sclerosis, and mechanical valves are now more common causes in developed countries due to the recognition and treatment of group A streptococcal pharyngitis Choice A= Congenital heart lesions that predispose to increased risk of IE include bicuspid aortic valve, ventricular septal defects, patent ductus arteriosus, and unrepaired tetralogy of Fallot. Isolated atrial septal defects are not commonly associated with an increased risk of IE, likely due to the low-pressure differential between the atria and the absence of high-velocity intracardiac flow jets that can damage the endocardium Choice B= Coronary heart disease can cause ischemic papillary muscle dysfunction with resultant mitral regurgitation. However, the clinical presentation and autopsy findings in this patient are most consistent with MVP and IE. Choice C= IE is a rare complication of hypertrophic cardiomyopathy and is almost always seen in patients with outflow obstruction. HCM is characterized by asymmetric ventricular hypertrophy leading to reduced ventricular cavity size. Choice D= Mitral annular calcification refers to degeneration and progressive calcium deposition involving the fibrous annulus of the mitral valve. Mitral annular calcification is usually seen in older adults (age >60), and the annulus is not a common location for IE

-Age: 58 years man -History: hypertension, hyperlipidemia; comes to the ED due to substernal chest pain that began 6 hours ago and has gradually worsened. BP (104/72), pulse (88). ECG shows normal sinus rhythm with ST-segment depression and T-wave inversion, elevated troponin levels -The patient is diagnosed with non-ST-segment elevation myocardial infarction and admitted to the hospital for medical management, with coronary angiography scheduled the following morning. That evening, telemetry monitoring shows a new Mobitz type 1 second-degree atrioventricular block. Which of the following arteries was most likely obstructed during this patient's myocardial infarction? a)Left anterior descending b)Left circumflex c)Left main coronary d)Ramus intermedius e)Right coronary

Answer: Choice E (Right coronary) -Low-degree AV block, including first-degree and Mobitz type 1 2nd-degree, usually involves conduction disruption within the AV node. This patient's development of Mobitz type 1 second-degree AV block in the setting of acute MI suggests obstruction of the right coronary artery (RCA). In approximately 90% of patients, the RCA supplies blood to the AV node while coursing around the right side of the heart to perfuse the RV and inferior portion of the LV. Because the RCA also usually supplies blood to the SA node, sinus bradycardia is also commonly seen with MI involving the RCA -In contrast, the left anterior descending (LAD) artery (Choice A), which courses down the anterior LV, typically supplies blood to the left and right bundle branches and sometimes the bundle of His. MI involving the LAD can lead to high-degree AV block (i.e., Mobitz type 2 second-degree, infranodal third-degree) involving conduction disruption below the AV node. Because the LAD does not supply blood to the AV node, low-degree AV block is not typically seen Choices B and C= The left main coronary artery bifurcate into the LAD artery and the left circumflex artery, which courses around the left side of the heart to supply the lateral left ventricle. The left main coronary artery does not directly supply blood to the conduction system. The left circumflex artery frequently supplies blood to the SA node after coursing around to the back of the heart, but only supplies the AV node in about 10% of patients (i.e., those with left-dominant coronary circulation) Choice D= Present in approximately 20% of the population, a ramus intermedius is a third branch off of the left main coronary artery. It originates at the bifurcation of the LAD and left circumflex arteries and courses between them to supply the anterolateral LV wall. It does not supply blood to the conduction system

-Age: 23 years woman -Brought to the ED with a chest wound. She was in her house during a severe thunderstorm when a large tree branch fell through a window. -The window was shattered, and large fragments of wood and glass struck the patient. She di not lose consciousness, but her family found her bleeding profusely and rushed her to the hospital. BP (80/50), pulse (130). The patient appears to be in severe distress -Physical examination shows a deep penetrating wound in the 4th intercostal space along the left sternal border. Smaller lacerations are noted across the face and forearms. Which of the following structures is most likely injured in this patient? a)Inferior vena cava b)Left atrium c)Left ventricle d)Pulmonary trunk e)Right ventricle

Answer: Choice E (Right ventricle) A penetrating injury at the left sternal border in the 4th intercostal space (level of the nipple) will pass through the following layers (in order) -1) skin and subcutis -2) pectoralis major muscle -3) external intercostal membrane -4) internal intercostal muscle -5) internal thoracic artery and veins -6) transversus thoracis muscle -7) parietal pleura -8) pericardium -9) RV myocardium The RV composes the majority of the anterior surface of the heart and is at risk of injury with penetrating trauma at the mid- and lower-left sternal border. The parietal pleura would be injured as well, but the left lung itself would not be punctured as there is no middle lobe on the left side, and the superior lobe of the left lung is displaced laterally by the cardiac impression Choice A= The inferior vena cava (IVC) passes through the right side of the central tendon of the diaphragm at the level of T8. A penetrating wound to the back at the immediate right of the vertebral body could strike the IVC Choice B= The left atrium is located at the base of the heart (posterior surface, opposite the apex) and makes up most of the heart's posterior surface. Only the auricle of the LA is visible anteriorly (protruding between the pulmonary trunk and LV) Choice C= The LV composes the left lateral aspect of the heart. A stab wound angled slightly medially in the 4th intercostal space at the midclavicular line could strike the LV, but only after passing through the bulk of the left lung Choice D= The pulmonary trunk could be pierced by a penetrating injury to the 2nd intercostal space at he left sternal border

-Age: 46 years man -Comes to the office with chest pain and dyspnea on exertion. He has no known medical problems and leads a sedentary lifestyle. He is a lifetime nonsmoker. Noninvasive cardiac testing is nondiagnostic. Left and right heart catheterization is planned. -During the procedure, the catheter records periodic pressure changes with a maximum of 25mmHg and minimum of 2mmHg. The catheter is advanced further, and then shows periodic pressure changes with a maximum of 25mmHg and a minimum of 10mmHg. Assuming the results of the procedure are normal, the first set of readings was most likely obtained from which of the following locations? a)Left atrium b)Left ventricle c)Pulmonary artery d)Right atrium e)Right ventricle

Answer: Choice E (Right ventricle) Cardiac catheterization is performed to evaluate for cardiac etiologies and obtain direct measurements of intracardiac pressures in patients with unexplained symptoms. In general, right-sided pressures are much lower than those of the left side as the right ventricle is pumping blood into a much lower resistance pulmonary vasculature A balloon-tipped pulmonary artery (PA) catheter (attached distally to a pressure transducer) is inserted into the subclavian or internal jugular vein. When the PA catheter enters the superior vena cava (SVC), a venous waveform appears, and the balloon is inflated with air and the catheter is advanced. The location of the catheter tip is determined by the pressure readings as follows: -1) The venous pressure waveform of the SVC appears as a small-amplitude oscillations. The pressure remains unchanged as the catheter is advanced into the right atrium (RA). Normal pressure in the SVC and RA is 1-6mmHg (Choice D) -2) As the catheter is advanced across the tricuspid valve into the RV, a pulsatile waveform appears. The peak (systolic) pressure is the strength of RV contraction, and the lowest (diastolic) pressure is just below the RA pressure. RV systolic is normally 15-30mmHg -3) As the catheter advances across the pulmonic valve into a main pulmonary artery, the pressure waveform shows a sudden increase in diastolic pressure with no change in systolic pressure. This rise in diastolic pressure is due to pulmonary capillary resistance and backward transmission of left atrial pressure. The PA diastolic pressure is normally 6-12mmHg (Choice C) -4) When the catheter is advanced into the distal PA branches, a venous type waveform appears; this is called the PA occlusion pressure or pulmonary capillary wedge pressure (PCWP). This pressure is reflective of venous pressure in the left heart (left atrial pressure and LV diastolic pressure) (Choice A). PCWP is usually equal to PA diastolic pressure (6-12mmHg) Choice B= Left-sided pressures are much higher than right heart pressures in order to maintain systemic perfusion. The normal left ventricular systolic pressure is approximately 90-140mmHg with a LV end-diastolic pressure of 6-12mmHg

-Age: 80 years man -Brought to the hospital due to increasing chest pain. Over the past 6 weeks, he has had dry cough, dysphagia, and a change in voice quality. -History: long history of hypertension; ex-smoker (35-pack-year history) -On physical exam: patient appears pale, anxious, diaphoretic. BP (80/60), pulse (120), and RR (20). While being evaluated in the ED, he develops cardiac arrest with pulseless electrical activity and can't be resuscitated. Autopsy will show? a)Fat globules in the pulmonary vasculature b)Left ventricular free-wall rupture c)Longitudinal tear in the esophagus d)Ruptured pulmonary bled e)Ruptured thoracic aortic aneurysm

Answer: Choice E (Ruptured thoracic aortic aneurysm) -Thoracic aortic aneurysm (TAA) can involve the ascending or descending aorta above the diaphragm. It is usually asymptomatic until it grows large enough to encroach on nearby structure or cause rupture. This patient likely experienced progressive TAA expansion with symptomatic compression of surrounding structures, followed by acute rupture that led to shock and cardiac arrest -TAA usually results from age-related degenerative changes in the medial layer of the aorta; these changes are likely accelerated by risk factors, including dyslipidemia, hypertension, tobacco use, and family history. Connective tissue diseases such as Marfan or Ehlers-Danlos syndrome also increase the risk -The most common symptomatic presentation of TAA is pain, which is typically localized to the chest and back. If the TAA impinges upon the esophagus, it can also cause dysphagia. Similarly, compression of the left recurrent laryngeal nerve or left vagus nerve results in hoarseness, whereas compression of the phrenic nerve can cause hemidiaphragmatic paralysis. Respiratory manifestations, including wheeze, cough, hemoptysis, and dyspnea may occur due to tracheobronchial obstruction. Other complications include heart failure due to aortic valve regurgitation and superior vena cava syndrome from venous compression and occlusion -Chest x-ray may suggest the diagnosis of TAA, demonstrating a widened mediastinum, enlarged aortic knob, and tracheal deviation Choice A= Pulmonary fat embolism most commonly results in acute dyspnea, tachypnea, and hypoxemia. Patients commonly develop neurologic abnormalities and a petechial rash, both of which this patient lacks Choice B= Ventricular free-wall rupture is a life-threatening complication of STEMI that typically occurs 5 days to 2 weeks after the event. It can also result in shock and cardiac arrest, but it would not explain this patient's subacute cough, voice change, or dyspnea Choice C= Patients with a Mallory-Weiss tear have hematemesis and epigastric or back pain. These esophageal tears are usually preceded by episodes of nonbloody vomiting or retching Choice D= Spontaneous pneumothorax results from the rupture of a pulmonary bleb and can occur with or without a preceding history of lung disease. Most patients describe acute chest pain and dyspnea. Although it can rarely progress to tension pneumothorax with obstructive shock, such a condition would not explain this patient's subacute symptoms

-Age: 6 months boy -Brought to the ED due to lethargy for several hours. His mother says he is usually a "very active baby" and she is afraid that he is "coming down with something". The patient's birth history is unremarkable, and he was born at 38 weeks of gestation -He is afebrile. BP (70/44), pulse (240), RR (52). The patient appears pale and is perspiring. Capillary refill time is less than 2 seconds. The lungs are clear to auscultation. No murmurs are heard. An ECG shows a regular hear rate of 240bpm with narrow QRS complexes and no identifiable P waves. Which of the following is the most likely cause of this patient's hypotension? a)Decreased effective intravascular volume b)Decreased myocardial contractility c)Decreased peripheral vascular resistance d)Left-to-right intracardiac shunting e)Shortening of ventricular diastole

Answer: Choice E (Shortening of ventricular diastole) -This patient is pale, lethargic, and diaphoretic with ECG findings consistent with supraventricular tachycardia (SVT), the most common pediatric arrhythmia -Normally, depolarization impulses originate at the SA node and travel through the atrial myocardium to the AV node, where the signal pauses to allow ventricular filling following atrial contraction. The impulse then continues through the His-Purkinje system to the ventricular myocardium. -SVT occurs when an impulse originates at an abnormal focus above the ventricles, most commonly a reentrant circuit within the AV node (i.e., AV nodal reentry tachycardia). Heart rate is elevated (usually >150/min in adults or >220/min in infants), but rhythm is usually regular. QRS complexes are narrow, and P waves are generally buried within the QRS complex due to retrograde atrial depolarization occurring in conjugation with anterograde ventricular depolarization -With persistent tachycardia, ventricular diastole shortens, allowing less time for ventricular relaxation and filling. Stroke volume and cardiac output decrease, leading to hypotension and poor peripheral perfusion. Infants may become altered (e.g., lethargic, poor feeding) and develop signs of heart failure (e.g., tachypnea, crackles, hepatomegaly) Choice A= Decreased effective intravascular volume is the mechanism for hypertension associated with hypovolemic shock. This patient has normal capillary refill time without a history of volume depletion (e.g., diarrhea, vomiting, blood loss) Choice B= Hypotension due to decreased myocardial contractility is seen with an MI, which is extremely rare in infancy, or a viral-induced cardiomyopathy, which is unlikely in the absence of infectious symptoms (e.g., fever, rhinorrhea) Choice C= Low cardiac output associated with SVT would lead to a compensatory increase, not a decrease, in peripheral vascular resistance Choice D= A large left-to-right intracardiac shunt (e.g., ventricular septal defect) can cause hypotension and tachycardia, but signs of heart failure (e.g., crackles, hepatomegaly) would be expected. In addition, depending on the size of VSD, ECG may show left or biventricular hypertrophy, not narrow QRS complexes with absent P waves

-Age: 4 years boy -Being evaluated for failure to thrive, shortness of breath, and exercise intolerance. The parents first became aware of his symptoms after the patient started preschool a few weeks ago. His teacher mentioned that the patient seemed to tire faster than the other children when playing outside. Cardiac examination shows bounding peripheral pulses and a palpable thrill below the clavicle near the left upper sternal border. On auscultation, a continuous murmur is best heard over the same region. After discussion with the parents, a thoracotomy is scheduled to correct this patient's condition. During the procedure, the surgeon should plan on ligating a derivative of which of the following embryologic structures? a)Bulbus cordis b)Fourth aortic arch c)Primitive atria d)Sinus venosus e)Sixth aortic arch

Answer: Choice E (Sixth aortic arch) -This patient most likely has a patent ductus arteriosus (PDA). The ductus arteriosus is an embryonic derivative of the sixth aortic arch that allows fetal blood to pass directly from the pulmonary artery to the proximal descending aorta (bypassing the pulmonary circulation). This vessel usually closes shortly after birth, secondary to decreased prostaglandin E2 (PGE2) levels and increased oxygen concentration. Patency of the ductus after birth results in a left-to-right shunt that can cause LV volume overload and symptoms of heart failure (e.g., failure to thrive, respiratory distress). Physical examination will show a continuous "machinery-like" murmur and palpable thrill over the left infraclavicular region due to turbulent blood flow through the PDA. -Pharmacologic closure of a PDA can be achieved by PGE2 synthesis inhibitors (e.g., indomethacin) in premature infants. However, older patients usually require surgical ligation or percutaneous PDA occlusion Choice A= The bulbus cordis forms the beginning of the ventricular outflow tract in the embryonic heart. This structure forms the smooth portions of the LV and RV adjacent to the aorta and pulmonary artery, respectively Choice B= The 4th aortic arch on the left forms the arch of the aorta between the left carotid artery and ductus arteriosus. Abnormalities during development can result in coarctation (narrowing) of the aorta, which can present with continuous murmurs over the back due to collateral formation Choice C= The primitive atrium receives blood from the sinus venosus in the embryonic heart and transmits it to the primitive ventricle. The primitive atrium forms the rough portions of the left and right atria Choice D= The sinus venosus is an embryologic structure within the heart that receives blood from the vena cava. In adults, this structure forms the smooth portion of the right atrium, known as the sinus venarum

-Age: 64 years man -Comes to the ED due to chest discomfort and shortness of breath. History: hypertension, type 2 diabetes, and severe osteoarthritis of the left knee. The patient is an active smoker with a 20-pack-year smoking history. He undergoes dobutamine infusion with simultaneous cardiac imaging. The following is seen: - Before infusion: normal left ventricular contractility, ejection fraction= 60% - During infusion: decreased apical contractility, ejection fraction= 45% - 5 mins after infusion: normal left ventricular contractility, ejection fraction= 60% -Which of the following best explains the observed findings? a)Coronary artery occlusion due to plaque thrombosis b)Coronary artery vasospasm c)Focal myocardial fibrosis d)Normal myocardial contractile response e)Supply-demand mismatch due to fixed coronary stenosis

Answer: Choice E (Supply-demand mismatch due to fixed coronary stenosis) -Stable angina results from fixed coronary artery stenosis in the setting of atherosclerotic coronary artery disease (CAD). Such a stenosis limits blood supply to the downstream myocardium, which, during periods of increased myocardial oxygen demand (e.g., exercise), can cause a mismatch of oxygen supply and demand that manifests as anginal symptoms (e.g., chest pain, shortness of breath) -Pharmacological stress testing can be useful in diagnosing atherosclerotic CAD. Dobutamine, a beta-agonist, increases heart rate and contractility to mimic the increase in myocardial oxygen demand that occurs with exercise. Myocardium that is unable to obtain sufficient blood flow to meet the increased oxygen demand typically demonstrates a transient decrease in contractility (i.e., wall motion defect), often leading to reduced ejection fraction. This patient with a transient wall motion defect localized to the cardiac apex most likely has a fixed atherosclerotic lesion that limits blood flow to that area of the myocardium Choice A= Coronary artery occlusion due to plaque rupture and thrombosis leads to acute MI. Because of total coronary artery occlusion and a completely blocked myocardial oxygen supply, a persistent (before, during, and after dobutamine infusion), rather than transient, wall motion defect would be expected. Choice B= Vasospastic angina is due to coronary artery vasospasm and can mimic stable angina that results from atherosclerotic CAD; however, vasospastic angina typically occurs in relatively young patients (age <50). In addition, dobutamine has only a small agonistic effect on alpha-1 receptors and is unlikely to trigger coronary vasospasm Choice C= Focal myocardial fibrosis typically results from previous MI. Contractility is impaired in the fibrosed myocardium, leading to a persistent wall motion defect similar to that expected with acute MI Choice D= In the normal state, dobutamine boosts myocardial contractility, and the rise in myocardial oxygen demand is met by increased myocardial oxygen supply via increased blood flow. Therefore, the normal response to dobutamine infusion is a transient increase in ejection fraction

-Age: 52 years woman -Comes to the office due to fatigue for the last 2 weeks. The patient was previously healthy, but over the past 3 months, she has had significant weight loss without changing her diet. -She also reports increased sweating, hand tremor, and decreased sleep. Examination shows diffuse, nontender enlargement of the thyroid gland -The eyeballs are protuberant. The remainder of the examination is notable for 1+ pitting edema in the ankles. Which of the following parameters is most likely to be decreased in this patient? a)Cardiac contractility b)Myocardial oxygen consumption c)Pulmonary artery pressure d)Pulse pressure e)Systemic vascular resistance f)Systolic blood pressure

Answer: Choice E (Systemic vascular resistance) The patient with an enlarged thyroid gland and multiple consistent symptoms (e.g., fatigue, unintentional weight loss, sweating, tremor, insomnia) most likely has hyperthyroidism. Protuberance of the eyeballs (i.e., exophthalmos) suggests Graves disease, specifically In adults, the major effect of thyroid hormone is control of cellular metabolism, including the rate of oxygen consumption and use of proteins, carbohydrates, and lipids. As a result, hyperthyroidism causes characteristic cardiovascular effects -Increased metabolic demand in the peripheral tissues, combined with a direct effect of thyroid hormone on vascular smooth muscle, causes peripheral vasodilation, which leads to decreased systemic vascular resistance (SVR) and reduced diastolic BP -Thyroid hormone has a direct sympathetic-like effect on the myocardium, which stimulates increased HR and contractility (Choice A). A reflexive response to decreased SVR also contributes to these changes. With increased contractility, stroke volume is increased, resulting in increased pulse pressure and systolic BP (Choices D and F). Ejection fraction and cardiac output are also increased Over time, some patients develop high-output heart failure due to the hyperdynamic circulatory state. Restoration of a euthyroid state normalizes cardiovascular hemodynamics and typically resolves any heart failure that has developed Choice B= Myocardial oxygen consumption increases in hyperthyroidism, caused by a direct effect of thyroid hormone and by hemodynamic changes that increase myocardial oxygen demand (i.e., increased HR, contractility, and afterload (SBP)) Choice C= Both diastolic and systolic pulmonary artery pressures increase in hyperthyroidism. This is likely because cardiac output increases, but the pulmonary circulation does not undergo the same vasodilation and reduction in resistance that occurs in the systemic circulation (pressure= Cardiac output x resistance)

-Age: 48 years man -With diet-controlled type 2 diabetes is being evaluated for occasional retrosternal chest pain. He has no history of hypertension, and his BP measurements during office visits are always within normal limits. There is no family history of cardiovascular disease -The patient undergoes exercise treadmill stress testing. He walks for 7 minutes on the treadmill and stops due to fatigue but does not experience chest pain. His ECG does not show any abnormal changes. Heart rate ranges from 70/min at rest to 132/min at peak and mean blood pressure ranges from 95mmHg at rest to 112mmHg at peak. -Which of the following parameters was most likely decreased during peak stress compared to the resting state in this patient? a)Cardiac output b)Left ventricular end-diastolic pressure c)Pulmonary artery systolic pressure d)Systemic systolic blood pressure e)Total systemic vascular resistance

Answer: Choice E (Total systemic vascular resistance) -Exercise is associated with several hemodynamic and vasoregulatory adjustments to provide maximal blood flow to the active muscles. Stimulation of mechano- and chemoreceptors in the exercising muscle causes sympathetic activation and withdrawal of vagal tone, resulting in increased HR, stroke volume, and overall cardiac output (Choice A). At higher levels of exercise, HR is predominantly responsible for most of the increase in CO. -Sympathetic discharge causes contraction of the arterioles in all tissues except the actively working muscles, effectively shunting blood toward the exercising muscle. This also results in an increase in systolic BP, which helps improve perfusion of actively contracting skeletal muscle (Choice D). However, the rise in the mean arterial pressure is much lower due to an overall decrease in systemic vascular resistance. This occurs due to the substantial arteriolar vasodilation in active skeletal muscles, which is mediated by local release of adenosine, potassium ions, ATP, CO2, and lactate. Choice B= LV end-diastolic volume and pressure both rise during exercise due to an increase in LV filling (increased venous return) by peripheral vasoconstriction and the pumping actions of actively contracting muscles Choice C= Pulmonary artery systolic pressure typically rises during exercise; however, similar to systemic circulation, the rise in the pulmonary pressure is much smaller than the increase in blood flow across the pulmonary circulation due to a fall in pulmonary vascular resistance

-Age: 82 years woman -Comes to the office due to fatigue, abdominal discomfort, and lower extremity swelling. She also occasionally feels an uncomfortable pulsation in her neck. 6 weeks ago, the patient underwent pacemaker implantation due to sick sinus syndrome -Episodes of lightheadedness that she had experienced before the procedure have now resolved. The patient is afebrile, and other vital signs are normal. Physical examination reveals bilateral, lower extremity edema and a tender, pulsatile liver. The lungs are clear on auscultation. The pacemaker implantation site has healed normally. Complete blood count is normal. Diagnosis? a)Aortic valve regurgitation b)Constrictive pericarditis c)Infective endocarditis d)Pulmonic valve regurgitation e)Tricuspid valve regurgitation

Answer: Choice E (Tricuspid valve regurgitation) -This patient likely has tricuspid valve regurgitation (TR) due to an adverse effect of her permanent pacemaker. The RV lead of an implantable pacemaker or cardioverter-defibrillator passes through the superior vena cava into the right atrium and then through the tricuspid valve orifice to terminate on the endocardium of the RV. Damage to the tricuspid valve leaflet or inadequate leaflet coaptation can sometimes occur, leading to severe TR in some patients. -Severe TR typically presents with right-sided heart failure. Patients can have distended jugular veins, pulsatile and tender hepatomegaly, abdominal distention with ascites, and lower extremity edema. The lungs are clear on auscultation in the absence of concomitant left-sided heart disease. Cardiac examination typically reveals a holosystolic murmur best heard at the left lower sternal border; the murmur intensifies with maneuvers that increase RV preload (e.g., deep inspiration, leg raise) Choice A= Causes of aortic regurgitation include rheumatic heart disease and aortic root dilation. Severe disease can lead to left-sided heart failure; however, pulmonary edema with crackles on lung auscultation would be expected Choice B= Constrictive pericarditis can result from chronic pericardial inflammation in the setting of infection or rheumatologic disease. It typically manifests with signs and symptoms of right-sided heart failure; however, the relationship between this patient's heart failure symptoms and recent pacemaker placement makes TR more likely Choice C= Infective endocarditis may affect the tricuspid valve and lead to TR; however, tricuspid involvement typically develops in patients with a history of injection drug use. In addition, this patient's lack of fever makes infective endocarditis unlikely Choice D= Severe pulmonic valve regurgitation can lead to right-sided heart failure. Causes of pulmonic valve regurgitation include rheumatic heart disease and endocarditis, but it is not associated with pacemaker placement as pacemaker leads do not cross the pulmonic valve.

-Age: 32 years woman -Comes to the ED due to sudden-onset left-sided facial droop and arm weakness. The patient also has had right leg swelling after a recent road trip. She has no chronic medical conditions, and her only medication is an oral contraceptive -CT scan of the head shows a right middle cerebral artery territory acute infarction. Lower extremity ultrasound examination reveals a right femoral vein thrombus. No obvious cardiac structural abnormalities are detected on an echocardiogram. -Agitated normal saline is injected into a peripheral vein to evaluate for abnormal right-to-left shunting. No bubbles are seen in the left side of the heart after several minutes. -Which of the following is most appropriate to increase right-sided heart pressure to reveal an obscure intracardiac shunt? a)Abrupt standing from a supine position b)Amyl nitrate injection c)Breath-holding after exhalation d)Sustained hand grip e)Valsalva maneuver release phase

Answer: Choice E (Valsalva maneuver release phase) -This patient has a deep venous thrombosis and has experienced a large stroke, suggesting paradoxical embolization of thrombus (i.e., passage from the venous circulation to the arterial circulation). Because thrombi are too large to pass through the pulmonary capillaries, paradoxical embolization typically occurs via an intracardiac right-to-left shunt. In this case, echocardiography did not reveal overt septal abnormalities (i.e., atrial septal defect or ventricular septal defect), raising strong suspicion for a patent foramen ovale (PFO) -PFO is common, affecting up to 30% of adults. It results from failure of the septum primum and septum secundum to fuse during early childhood, creating a one-way tissue valve. This valve is forced closed when left atrial pressures exceeds right atrial pressure (which is most of the time), but it opens when right atrial pressure exceeds left atrial pressure, allowing for transient right-to-left shunting -To test for the presence of a PFO, normal saline bubbles are injected IV, and echocardiography is performed to observe whether the bubbles pass from the right atrium into the left atrium. The release phase of a Valsalva maneuver increases venous return to the right atrium and consequently increases right atrial pressure to facilitate right-to-left shunting of saline bubbles and confirm the diagnosis (the strain phase of Valsalva maneuver decreases venous return to the RA and discourage shunting through a PFO) Choices A, B and C= Medications and physiologic maneuvers that reduce venous return to the right atrium discourage right-to-left shunting through a PFO. Abrupt standing from a supine position causes blood to pool in the lower extremities, amyl nitrate (an inhaled nitrate) causes venous vasodilation, and exhalation causes increased intrathoracic pressure (resulting in partial collapse of the inferior vena cava at end-expiration), all of which reduce venous return to the right atrium Choice D= Sustained hand grip increases systemic vascular resistance, increasing pressure in the LV and LA and discouraging shunting through a PFO

-Age: 18 years man -Comes to the clinic due to hematuria and intermittent left flank pain of several months duration. He has no history of trauma of sexually transmitted diseases and no associated fever or dysuria. Examination reveals a soft abdomen with normal bowel sounds and no localized tenderness. Urinalysis confirms 3+ blood but no white blood cells, crystals, or organisms. Contrast-enhanced CT scan shows no abnormalities in the ureters or kidneys but does reveal compression of the left renal vein between the superior mesenteric artery and the aorta. -Which of the following is most likely to develop due to the vascular abnormality seen in this patient? a)Esophageal varices b)Left-sided ankle swelling c)Periumbilical venous distention d)Rectal varices e)Varicocele

Answer: Choice E (Varicocele) -The right renal vein is a relatively short structure and runs anterior to the right renal artery before joining the IVC. The right gonadal vein also drains directly to the IVC. In contrast, the left renal vein is significantly longer and rungs posterior to the splenic vein before crossing the aorta beneath the superior mesenteric artery. The left gonadal vein joins the left renal vein upstream of where it crosses the aorta and does not enter the IVC directly -The pressure within the left renal vein is often higher than on the right due to compression between the aorta and the superior mesenteric artery ("nutcracker effect"). Pressure in the left renal vein can also be elevated due to compression from a left-sided abdominal or retroperitoneal mass. Persistently elevated pressure in the left renal vein can cause flank or abdominal pain, along with gross or microscopic hematuria (left renal vein entrapment syndrome). Increased pressure in the left gonadal vein results in valve leaflet failure and varices of the testicular pampiniform plexus (varicocele) Choices A, C and D= Esophageal varices, rectal varices, and periumbilical venous distention are signs of portal venous hypertension. The renal veins are not part of the portal system Choice B= Unilateral left-sided ankle swelling can result from many causes, including obstruction of the left common iliac vein, left external iliac vein, left femoral vein, or any of the major veins of the left leg

-Age: 21 years man -Comes to the office due to multiple episodes of syncope. The patient has no chest discomfort or dyspnea. He has no known medical problems and doesn't smoke, drink or illicit drugs -The patient is a computer analyst (sedentary lifestyle). He reports that several family members have died of sudden cardiac death -Genetic analysis reveals an ion channel defect. Due to the defect, cardiac cells show decreased outward potassium flow and resultant prolongation of the action potential -Which of the following is the most likely consequence of this patient's disease? a)Abnormal anatomic communication between cardiac chambers b)Asymmetric hypertrophy of the left ventricle c)Ischemic myocardial necrosis followed by scarring d)Left ventricular dilation and systolic dysfunction e)Ventricular tachycardia and sudden death

Answer: Choice E (Ventricular tachycardia and sudden death) -This patient's recurrent syncope, family history of sudden death, and ion channel defect (decreased outward potassium outflow with prolonged action potential) is suggestive of congenital long QT syndrome (LQTS). There are several genetic mutations described in patients with congenital LQTS. -The 2 most common forms (LQT type 1 and LQT type 2) are due to genetic mutations in K+ channel proteins that contribute to the outward-rectifying potassium current -Phase 3 (repolarization) of the myocardial AP occurs due to time-dependent inactivation of the L-type (long-lasting, large-conductance) Ca2= channels (which reduces inward current) along with activation of the voltage-gated delayed rectifier K+ channels (which increases outward current). This net increase in outward transmembrane current is responsible for ventricular repolarization. -Decreased outward potassium flow due to K+ channel mutations leads to prolongation of action potential duration and QT interval. Prolongation of QT interval predisposes to the development of life-threatening ventricular arrhythmias (e.g., torsades de pointes, bottom pic), which can present as recurrent palpitations, syncope, seizures, or sudden cardiac death Choice A= Abnormal anatomic communication between cardiac chambers presents in patients with atrial septal defects, ventricular septal defects, or patent ductus arteriosus. These disorders are not associated with any genetic defects in ion channel Choice B= Asymmetric hypertrophy of the LV occurs in patients with hypertrophic cardiomyopathy, an autosomal dominant disorder caused by mutations in sarcomere genes. The 2 most common mutations involve the cardiac myosin-binding protein C gene and cardiac beta-myosin heavy chain gene Choice C= Ischemic myocardial necrosis and scarring occur in MI and often lead to LV systolic dysfunction. Patients with scarred myocardium are also at increased risk for developing ventricular arrhythmias; however, these arrhythmias would not be associated with an ion channel defect Choice D= Dilated cardiomyopathy is characterized by LV dilation and systolic dysfunction. Several genetic mutations involving the sarcomere structures and function have been identified in patients with familial dilated cardiomyopathy. However, abnormal transmembrane K+ currents are not known to be directly involved in the pathophysiology

-Age: 46 years man -30-pack-year smoking history develops right leg pain and swelling after returning from an overseas trip. He is diagnosed with a lower extremity deep venous thrombosis and is started on IV heparin. -Shortly after being hospitalized, he develops right-sided weakness and facial droop. CT scan of the head reveals a left middle cerebral artery stroke. Which of the following physical examination findings is most likely to be present in this patient? a)Diastolic decrescendo-type murmur that decreases following amyl nitrite inhalation b)Ejection-type systolic murmur that increases with standing c)Presystolic murmur that disappears with atrial fibrillation d)Midsystolic click that occurs earlier with inspiration e)Wide splitting of S2 that does not change with inspiration

Answer: Choice E (Wide splitting of S2 that does not change with inspiration) -A cerebrovascular event (e.g., TIA, stroke) in the setting of known venous thromboembolic disease is suspicious for paradoxical embolism -Paradoxical emboli originate in the systemic venous circulation (lower or upper extremities) and enter the systemic arterial circulation via an intracardiac or intrapulmonary shunt. They can occur in patients with patent foramen ovale, atrial septal defects, ventricular septal defects, or large pulmonary arteriovenous malformations -ASDs with left-to-right shunting typically cause wide and fixed splitting (no change with respiration) of the 2nd heart sound (S2). Patients may also have a mid-systolic ejection murmur over the left upper sternal border resulting from increased flow across the pulmonic valve and a mid-diastolic rumble due to increased flow across the tricuspid valve. Importantly, a paradoxical embolism can occur even in patients with net left-to-right shunting due to transient reversal of the shunt during periods of elevate right-sided pressure (e.g., early ventricular systole, straining during coughing or defecation) Choice A= An early diastolic decrescendo murmur is characteristic of aortic regurgitation. Inhaled amyl nitrite produces marked vasodilation, which reduces systemic arterial pressure and regurgitant flow (lessening the murmur) Choice B= A systolic ejection murmur that increases in intensity with standing can be heard in patients with hypertrophic cardiomyopathy (due to a decrease in LV outflow tract size). In contrast, the murmur of valvular aortic stenosis decreases in intensity upon standing Choice C= Mitral or tricuspid valve stenosis can cause a diastolic murmur with presystolic accentuation due to atrial contraction. With mild stenosis, the murmur may only be audible during the accentuation phase in late diastole; atrial fibrillation may cause the murmur to disappear completely Choice D= The midsystolic click of mitral valve prolapse occurs earlier with inspiration due to the reduced LV volume caused by decreased venous return to the left heart

-Age: 34 years man -Comes to the office due to heart palpitations that are particularly prominent at night. With moderate exertion, he also experiences head pounding accompanied by involuntary head-bobbing. The patient recently emigrated from Southeast Asia and remembers being diagnosed with a heart murmur years before, but he cannot recall the type of murmur and has never received treatment. -He has no other conditions/no medications. He doesn't smoke. The patient's father has coronary artery disease, and his mother has type 2 diabetes. Based on this patient's history, which of the following findings is most likely to be present on further evaluation? a)Impaired left ventricular contractility b)Left ventricular outflow obstruction c)Restricted left ventricular filling d)Systolic and diastolic hypertension e)Widening of the pulse pressure

Answer: Choice E (Widening of the pulse pressure) -This patient has a number of clinical findings that are characteristic of chronic aortic regurgitation (AR). Failure of the aortic valve to tightly seal off the aorta from the LV at the end of systole leads to rapid loss of aortic pressure and an overall decrease in diastolic blood pressure. In addition, the LV undergoes eccentric hypertrophy to initially compensate for an increase in LV end-diastolic volume, which leads to increased stroke volume and cardiac output. Because pulse pressure (the difference between diastolic and systolic BP) is proportional to stroke volume, the compensatory increased stroke volume on top of reduced diastolic pressure causes the characteristic widened pulse pressure seen with AR -In high-amplitude pulsation with rapid diastolic collapse is responsible for the unusual pulsation findings in AR. Forceful pulsations in the intracranial arteries can cause head bobbing with each heartbeat (de Musset sign), and patient may experience palpitations due to forceful LV contraction. In addition, physical examination can show abrupt distention and collapse of the carotid arteries (Corrigan sign) and peripheral arteries ("water-hammer" pulse), and reveal "pistol-shot" femoral pulses (Traube sign) on auscultation Choice A= Impaired LV contractility occurs later in the course of AR and eventually leads to heart failure. Because head-bobbing and other characteristic findings in AR are due to increase stroke volume and high-amplitude, rapid rise-rapid fall pulsation, these findings are less prominent once impaired LV contractility (and decrease stroke volume) sets in Choices B and C= LV outflow tract obstruction, such as occurs with aortic stenosis, leads to reduced stroke volume and characteristic weak and delayed pulses (pulsus parvus et tardus). Restriction of LV filling, as occurs in mitral stenosis; also reduces stroke volume due to reduced LV end-diastolic volume (reduced preload). Clinical findings associated with high-amplitude pulsation will not be present with reduced stroke volume Choice D= Combined systolic and diastolic hypertension is not necessarily accompanied by widened pulse pressure; the signs of high amplitude, rapid rise-rapid fall pulsation are more characteristic of chronic AR

-Age: 54 years man -Comes to the office due to 2 days of redness and pain in his right arm. He was recently diagnosed with superficial thrombophlebitis involving the left lower and upper extremities -His symptoms then subsided within days of NSAID therapy only. The patient has no other past medical history. He has some abdominal discomfort that he attributes to GERD. Temp. (37.6C), on examination, there is erythema and tenderness extending linearly from the right forearm to just anterior to the right antecubital fossa. No fluid collection is palpable -CBC shows mild leukocytosis. The patient's symptoms may indicate presence of? a)Celiac sprue b)Chlamydial infection c)Hyperthyroidism d)Polycystic kidney e)Visceral cancer

Answer: Choice E (visceral cancer) -Migratory thrombophlebitis should raise suspicion for cancer. Hypercoagulability is a very common paraneoplastic syndrome seen most frequently in visceral adenocarcinoma of the pancreas, colon, and lung. Hypercoagulability develops because adenocarcinomas produce a thromboplastin-like substance capable of causing chronic intravascular coagulations that can disseminate and tend to migrate -Migratory superficial thrombophlebitis, known as Trousseau syndrome, is named after Armand Trousseau, a well-respected physician who first described the associated with cancer. Trousseau later diagnosed his own visceral cancer after developing the syndrome. -Trousseau also described the Trousseau sign (hand/forearm muscle spasms on sphygmomanometeric measurement) associated with hypocalcemia Choice A= Celiac sprue has been associated with hemorrhagic diathesis (rather than abnormal thrombosis). The malabsorption resulting from celiac sprue can cause vitamin K deficiency Choices B and D= Polycystic kidney disease is not among the conditions commonly associated with an acquired prothrombotic state. Disseminated chlamydia infections are classically associated with reactive arthritis (not thrombophlebitis), and infections in general can create a systemic inflammatory state that favors thrombosis. However, malignancy would be a more likely explanation for migratory superficial thrombophlebitis accompanied by vague abdominal pain Choice C= Hyperthyroidism has been associated with a hypercoagulable state and is occasionally reported in cases of cerebral venous thrombosis, but there is no evidence of a significant associated with venous thrombotic events

-A group of researchers are studying population-based screening interventions to reduce mortality from abdominal aortic aneurysm. This disorder develops primarily in elderly patients and is often asymptomatic until an acute rupture event, which is frequently fatal. -Screening of high-risk patients for abdominal aortic aneurysm is found to reduce mortality. Which of the following risk factor combinations would likely define the highest-risk group for screening purposes? a)Diabetes, smoking b)Female, hyperlipidemia c)Female, hypertension d)Hyperlipidemia, hypertension e)Male, diabetes f)Male, smoking

Answer: Choice F (Male, smoking) Focal dilation of the abdominal aorta >50% above normal (or >3cm in diameter) is termed abdominal aortic aneurysm (AAA). AAA formation involves transmural inflammation of the aortic wall with subsequent apoptosis of smooth muscle cells and degradation of matrix proteins (e.g., elastin, collagen). This causes progressive thinning of the aortic wall with secondary expansion of the lumen due to chronic hemodynamic stress. AAA is generally asymptomatic until aneurysm rupture, which typically presents with acute abdominal pain and hypotension, mortality of AA rupture is 75%-90% Major risk factors for development of AAA include: -Age> 65 -Smoking (up to 15-fold risk increase) -Male sex Smoking appears to be associated with increased inflammatory infiltrates and formation of reactive oxygen species (e.g., superoxide anion) in the aortic wall. In light of this, screening for AAA with ultrasonography is recommended for men age 65-75 who have ever smoked. The risk of rupture increases with aneurysm diameter, and surgical or endovascular repair is considered for aneurysms >5.5cm in diameter Choices A and E= Although diabetes mellitus increases the risk for atherosclerotic cardiovascular disease, the risk of AAA is lower in patients with diabetes, possibly due to the effects of glycosylation of matrix proteins in the aortic wall Choices B and C= Female sex is associated with a lower risk of AAA, and the benefit of screening in women (including women who smoke) has not been shown to outweigh the risks of overdiagnosis and potential treatment complications Choice D= Hypertension is associated with an increased risk of aneurysm rupture, but the relation to aneurysm incidence and expansion is less clear. Although there is a broad coincidence of AAA and atherosclerosis, studies on an epidemiologic link between hyperlipidemia and AAA are mixed.

-Age: 78 years man -Demonstrates decreased left ventricular cavity size and a sigmoid-shaped ventricular septum. Light microscopy shows increased collagen content within the ventricular wall. -Some myocardial cells also have brownish perinuclear cytoplasmic inclusions. The changes described are most consistent with which of the following? a)Dilated cardiomyopathy b)Hemolytic anemia c)Hypertrophic cardiomyopathy d)Ischemic heart disease e)Long-standing hypertension f)Normal aging

Answer: Choice F (Normal aging) -The morphologic changes described are consistent with normal aging of the heart. Aging is associated with decreased left ventricular chamber size, particularly in the apex-to-base dimension. This decrease in chamber length causes the ventricular septum to acquire a sigmoid shape, with the basilar portion bulging into the LV outflow tract. Atrophy of the myocardium results in increased interstitial connective tissue, often with concomitant extracellular amyloid deposition. Within the cardiomyocytes, there is a progressive accumulation of cytoplasmic granules containing brownish lipofuscin pigment (the result of indigestible byproducts of subcellular membrane lipid oxidation) Choice A= Dilated cardiomyopathy causes an increase in the LV chamber size. There is an increased risk of mural thrombi with this condition due to contractile dysfunction Choice B= chronic hemolytic anemia may result in the formation of hemosiderin-granules containing excess iron inside cardiac myocytes (hemosiderosis), which can be visualized with Prussian-blue stain. Excessive deposition of iron within the myocytes usually leads to dilated or restrictive cardiomyopathy Choice C= Hypertrophic cardiomyopathy (idiopathic hypertrophic subaortic stenosis) causes asymmetric, septal hypertrophy with disproportionate thickening of the ventricular septum compared to the LV free wall. Microscopic examination shows extreme myocyte hypertrophy, disorganized myocyte bundles, and myofiber disarray Choice D= Chronic ischemic heart disease tends to result in progressive congestive heart failure and ventricular dilation. Histologically, the major findings are diffuse subendocardial vacuolization and fibrosis Choice E= Long-standing hypertension results in hypertensive heart disease. Although left ventricular chamber size may be reduced in this condition, the major morphologic change is concentric left ventricular hypertrophy

-Age: 62 years woman -Comes to the office due to vague feeling of heaviness in her legs, especially when standing for long periods. She has no history of trauma to the legs. The patient's other medical problems include osteoarthritis and GERD. She has worked as a cashier at a department store for the last 20 years. -Vital signs are normal. Jugular venous pressure is normal. The heart has regular rate and rhythm with no murmurs or extra sounds. -Lung sounds are clear with normal air movement. The abdomen is soft and without masses. Examination of the lower extremities shows dilated, tortuous veins at the calves and ankles bilaterally, with no erythema or edema. Upper extremity examination is unremarkable. This patient's current condition increases the risk for? a)Intermittent claudication b)Ischemic stroke c)Myocardial infarction d)Phlegmasia alba dolens e)Pulmonary embolism f)Skin ulcerations

Answer: Choice F (Skin ulcerations) -This patient has varicose veins- dilated, tortuous veins typically involving the superficial venous drainage of the leg. Increased intraluminal pressure or loss of vessel wall tensile strength can lead to venous dilation and failure of the venous valves. The resulting backflow of blood exacerbates the venous hypertension, precipitating a vicious cycle that leads to further valvular incompetence and venous congestion -Varicose veins are most common in patients age >50, and the risk is increased by conditions that cause elevated venous pressure (e.g., prolonged standing, pregnancy, obesity). There is also often a familial association, suggesting possible genetic factors involving the venous wall or valves. Common complications include chronic edema, stasis dermatitis, skin ulcerations, poor wound healing and infection Choices A, B and C= Intermittent claudication is achy or crampy pain in the legs caused by inadequate arterial blood flow, usually due atherosclerosis in the branches of the femoral artery. In contrast to venous disease, in which the pain is worsened by prolonged standing or sitting with the legs dependent, claudication due to arterial disease is worse with exertion and relieved with rest. Because peripheral artery atherosclerosis is often associated with atherosclerotic disease elsewhere, patients with intermittent claudication have an increased risk for ischemic stroke or MI Choices D and E= DVT of the lower extremity can lead to pulmonary embolism. In addition, massive iliofemoral thrombosis can cause an acute rise in tissue pressure that impairs arterial inflow, leading to phlegmasia alba dolens (painful white "milk leg"). Although varicose veins can sometimes be complicated by thrombophlebitis, it primarily involves the superficial rather than deep venous system and does not increase the risk for pulmonary embolism or phlegmasia alba dolens

-Age: 41 years man -Comes to the ED due to 3 days of fever, chills, and nonproductive cough. The patient has also felt tired and weak during this period. History: injection drug use and has been hospitalized several times for injection site cellulitis. -Temp. (38.3C), BP (125/72), pulse (112, regular). Cardiopulmonary auscultation reveals scattered rhonchi and a cardiac murmur, which were not previously present. Lab evaluation: leukocytosis; chest x-ray: nodular opacities -Echocardiography of this patient is most likely to reveal? a)Aortic regurgitation b)Aortic stenosis c)Interventricular septal defect d)Mitral regurgitation e)Pulmonic stenosis f)Tricuspid regurgitation

Answer: Choice F (Tricuspid regurgitation) -Injection drug use is associated with concurrent injection of abrasive particulate matter and microorganisms into the bloodstream. Over time, injected particulate material denudes the surface of the heart valves, thereby creating a variegated surface for microorganism attachment. This dramatically increases the risk of infective endocarditis (IE). The right-sided valves are at greatest risk because pulmonary capillaries prevent large particulate matter from reaching the left side of the heart; the tricuspid valve is involved in >90% of cases. -Common manifestation of tricuspid valve IE include right-sided heart failure (e.g., jugular venous distention, ascites, lower extremity edema) and/or septic pulmonary emboli (e.g., multiple pulmonary nodules on chest x-ray). Cardiopulmonary evaluation usually reveals a blowing, holosystolic murmur heard best along the left lower sternal border that is intensified during inspiration and reduced with standing. Crackles and rhonchi are often present (in the setting of septic emboli). Echocardiography will show valvular vegetations, which frequently result in regurgitation due to incomplete valve closure Choices A and D= Aortic and mitral valve IE is less common in injection drug users than tricuspid valve IE because the pulmonary vasculature filters out particulate matter and microorganisms. In addition, septic emboli from the left side of the heart usually lodge in capillary beds (e.g., cutaneous) prior to returning to the right-side of the heart; therefore, septic pulmonary emboli are less common Choice B= Aortic stenosis is common in older adults and usually causes dyspnea, angina, or syncope. Stenosis is not typically seen in IE, and left-sided IE does not usually present with pulmonary septic emboli Choice C= An interventricular septal defect (VSD) can cause a left-to-right cardiac shunt, which can limit systemic blood flow and result in pulmonary hypertension. A VSD is less likely in this patient with IV drug use, signs of infection, and likely septic pulmonary emboli Choice E= Pulmonary valve IE sometimes occurs in injection drug users; however, it generally causes pulmonary valve regurgitation (not stenosis) due to incomplete valvular closure. Patients with pulmonary stenosis often have signs of right-sided heart failure (e.g., edema, ascites, elevated jugular venous distention) or dyspnea

-Age: 19 years man -Transitioning care from his pediatrician; recently changed his glasses for myopia, he otherwise has no symptoms; no medications; vitals are normal -Physical exam: tall with long upper extremities and fingers. Face appears narrow with down-slanted palpebral fissures, flattened malar bones, and a small jaw -Lungs are clear on auscultation. Late-systolic murmur is present at cardiac apex. Abdomen is soft and nontender with no organomegaly. Cause of patient's murmur? a)Aortic root dilation b)Aortic valve cusp fusion c)Endocardial fibrous deposition d)Myxomatous mitral degeneration e)Papillary muscle dysfunction

Answer: Choice D (Myxomatous mitral degeneration) -The patient has clinical features of Marfan syndrome, an autosomal dominant disorder caused by mutations in the FBN1 gene encoding fibrillin-1, a main component of extracellular matrix microfibrils. Fibrilling-1 provides support to elastic fibers and helps maintain connective tissue integrity. It also regulates extracellular matrix remodeling by binding to and sequestering transforming growth factor-beta (TGF-beta) -In Marfan syndrome, defective fibrillin-1 is unable to bind TGF-beta. The resulting overexpression of free, active TGF-beta leads to increased production of matrix metalloproteinases, which cleave elastic fibers and other components of the extracellular matrix, reducing tissue integrity. Within the mitral valve, this process results in fragmentation of elastic fibers and decreased collagen density and pooling of glycosaminoglycans (myxomatous mitral degeneration) -The elongated, floppy mitral valve leaflets prolapse into the atrium which causes a midsystolic click on auscultation and allows for mitral valve regurgitation. This regurgitation flow is characterized by a mid-to-late systolic apical murmur, as seen in this patient; severe regurgitation may produce a holosystolic murmur Choice A= Although aortic root dilation is common with Marfan syndrome, the associated aortic regurgitation causes a decrescendo diastolic murmur Choice B= Aortic valve cusp fusion (i.e., bicuspid aortic valve) is commonly associated with Turner syndrome, not Marfan syndrome. Auscultatory findings are attributed to aortic stenosis (systolic crescendo-decrescendo murmur at the right upper sternal border) and./or aortic regurgitation (diastolic murmur along the left sternal border) Choice C= Endocardial fibrous deposition occurs with carcinoid heart disease. In contrast to this case, presentation includes right-sided heart failure (e.g., edema, hepatomegaly), and common murmurs include tricuspid regurgitation (i.e., holosystolic murmur at the left lower sternal border) and pulmonic regurgitation (i.e., diastolic decrescendo murmur) Choice E= Papillary muscle dysfunction, which is typically the results of myocardial ischemia/infarction, can cause mitral regurgitation (i.e., holosystolic murmur). Marfan syndrome is not associated with papillary muscle dysfunction

-Age: 42 years man -Found dead at home. Medical problems: hypertension, dyslipidemia, but he had been noncompliant with his medications. -The patient had a lengthy smoking history (only quit recently). Autopsy= complete thrombotic occlusion of the left main coronary artery and diffuse atherosclerotic vascular disease characterized by multiple atheromas -Along with a lipid core, these atheromas have a fibrous cap formed from dense deposition of collagen. Which of the following cells are directly responsible for synthesizing this fibrous cap? a)Endothelial cells b)Interstitial fibroblasts c)Macrophages d)Smooth muscle cells e)T lymphocytes

Answer: Choice D (Smooth muscle cells) -Atherosclerosis is a chronic inflammatory and fibroproliferative disease. Endothelial cells, vascular smooth muscle cells (VSMCs), and leukocytes play an important role in the development and progression of this disease. The process is initiated by chronic hemodynamic stress and hyperlipidemia, which cause endothelial cell injury. This leads to increased expression of surface vascular cell-adhesion molecules (VCAMs) that allow adherence and migration of monocytes and T lymphocytes into the intima. -The infiltrating leukocytes and dysfunctional endothelium release cytokines and growth factors (e.g., PDGF, FGF, endothelin-1, IL-1) that promote migration and proliferation of VSMCs within the intima. VSMCs are also stimulated to synthesize extracellular matrix proteins (e.g., collagen, elastin, proteoglycans) that form the fibrous cap typical of mature atheromas. Disruption of the fibrous cap with luminal thrombosis (atherosclerotic plaque rupture) has catastrophic clinical consequences and is responsible for the majority of acute coronary syndrome cases and sudden cardiac deaths. Choice A= Endothelial dysfunction is the earliest pathophysiologic change that precedes the formation of atherosclerotic lesions. Endothelial injury results in increased lipid permeability, leukocyte (monocytes, T cells) and platelet recruitment, and migration and proliferation of VSMC that form media to the intima. However, the VSMCs are directly responsible for synthesis of new collagen Choice B= Fibroblasts are found infrequently in the tunica intima of blood vessels and are not significantly involved in atherosclerosis pathogenesis Choices C and E= Activated macrophages and T lymphocytes secrete growth factors that recruit the VSMCs responsible for forming the fibrous cap, but they are not directly responsible for the dense deposition of fibrillar collagen. Macrophages also produce matrix metalloproteinases and tissue factors that degrade the extracellular matrix, causing the formation of a large, soft lipid-rich core with thinning of the fibrous cap. Such vulnerable plaques have an increased propensity for rupture.

-Age: 21 years Caucasian male -Presents to the ED following an episode of syncope. The syncopal episode was not provoked by any activity or circumstance, nor was it preceded by lightheadedness. -The patient has no significant medical history; no medications -ECG= QT interval prolongation; Assuming it is an inherited condition, the relevant mutation most likely affects which of the following structures? a)Cardiac cell cytoskeleton protein b)Cardiac cell sarcomere proteins c)Mitochondrial enzymes of oxidative phosphorylation d)Calcium-binding sarcoplasmic reticulum protein e)Membrane potassium channel proteins

Answer: Choice E (Membrane potassium channel proteins) -This patient's sudden-onset syncopal episode suggests a sudden cardiac arrhythmia. QT prolongation in an otherwise healthy young individual is usually congenital. The mutation most likely to cause QT-interval prolongation can be determined based on an understanding of cardiac electrophysiology. -On an ECG tracing, the QT interval begins at the start of the QRS complex and ends at the end of the T wave. Thus, the QT-interval reflects the cardiac myocyte action potential duration, which is determined in part by K+ currents through channel proteins. The mutations listed in the other answer choices would be less likely to directly affect the cardiac cell action potential duration. -There are two important congenital syndromes that cause QT prolongation. Jervell and Lange-Nielsen syndrome (autosomal recessive with neurosensory deafness) and the more common Romano-Ward syndrome (autosomal dominant, no deafness). Both may predispose to torsades de pointes (a ventricular tachyarrhythmia) at a young age, causing syncopal episodes and possible sudden cardiac death Choices A and C= Mutations affecting cardiac cell cytoskeletal proteins, or the mitochondrial enzymes of oxidative phosphorylation are thought to cause the genetic form of dilated cardiomyopathy (DCM). DCM usually presents with gradual onset of left-sided heart failure (with symptoms like dyspnea on exertion initially), as opposed to the sudden syncopal episode in a previously asymptomatic patient described above. Choice B= Mutations in cardiac cell sarcomere proteins (e.g., beta-myosin heavy chain) underlie hypertrophic cardiomyopathy (HCM). Although HCM may present as syncope in a previously asymptomatic young person, the syncope of HCM is typically provoked by exertion. Additionally, QT prolongation is not generally found in HCM. Choice D= Mutations of a calcium-binding sarcoplasmic reticulum protein might underlie some cases of arrhythmogenic right ventricular cardiomyopathy (ARVC), a progressive fibrofatty replacement of the right ventricular myocardium of uncertain pathogenesis. QT prolongation is not generally seen with ARVC

-Age: 34 years woman -Comes to ED because of sharp chest pain that radiates to left shoulder. The pain increases with inspiration and is partially relieved by sitting up and leaning forward. -Review of her outpatient medical records shows that she was seen for facial rash 6 months ago. She is also being evaluated for proteinuria that was identified during her last clinical appointment. Which of the following is the most likely cause of this patient's chest pain? a)Aortic dissection b)Cardiac tamponade c)Intimal hyperplasia of pulmonary arteries d)Non-infectious cardiac valve vegetation e)Pericardial inflammation

Answer: Choice E (Pericardial inflammation) -The patient's chest pain is characteristic of acute pericarditis, which along with her facial rash and proteinuria is suggestive of underlying SLE. SLE is a chronic autoimmune disease predominantly affecting women age 20-40 that causes constitutional and multisystemic symptoms. Serosal inflammation is common in SLE and most often manifests as pleuritis or pericarditis -Pericarditis presents with severe and constant middle or left chest pain that may radiate to the neck and shoulders (particularly the trapezius ridge). The pain increases on inspiration (pleuritic) and is relieved by sitting up and leaning forward (postural). Auscultation of the chest reveals a scratchy sound called a pericardial friction rub that is best heart when the patient is leaning forward or lying prone. Additional cardiovascular manifestations in SLE include pericardial effusion, verrucous (Libman-Sacks) endocarditis and increased risk of coronary artery disease Choice A= Aortic dissection occurs in patients with long standing hypertension and those with Marfan and Ehlers-Danlos syndromes (the 2 latter conditions cause cystic medial necrosis of the aortic wall). It presents with abrupt onset, severe, tearing chest pain that radiates to the back Choice B= Cardiac tamponade is caused by accumulation of fluid in the pericardial space that prevents the heart from filling properly in diastole. It presents with dyspnea and tachypnea. Physical exam shows distended neck veins, hypotension, diminished heart sounds, and pulsus paradoxus (drop in systolic BP >10mmHg on inspiration) Choice C= Intimal hyperplasia of the pulmonary arteries is characteristic of both primary and secondary pulmonary hypertension. It presents with dyspnea, malaise, and findings associated with right ventricular failure (e.g., jugular venous distention, pedal edema, hepatomegaly). Severe symptomatic pulmonary hypertension is a rare complication of SLE Choice D= Non-bacterial endocarditis (verrucous or Libman-Sacks endocarditis) refers to small, wart-like fibrinous lesions and generalized thickening that can affect the heart valves of patients with SLE. These are typically asymptomatic but may lead to valvular insufficiency or embolism

-Age: 62 years man -Brought to the ED by a friend due to severe shortness of breath. He is agitated and gasping for air and is unable to provide history. His friend says the patient has a history of chronic alcohol use and attends Alcoholics Anonymous meetings. -He is unaware of any other medical history. Temp. (36.7C), BP (110/70), pulse (104), RR (32). Due to worsening respiratory distress, immediate endotracheal intubation is performed, and mechanical ventilation is begun. The patient is admitted to the intensive care unit and, despite appropriate therapy, dies several hours later. -Autopsy of lungs= engorged pulmonary capillaries and intraalveolar, acellular, pink material that is mor prominent at the bases. Patient had? a)Aspiration pneumonia b)Centriacinar emphysema c)Hypersensitivity pneumonitis d)Pulmonary arterial hypertension e)Pulmonary edema f)Pulmonary infarction

Answer: Choice E (Pulmonary edema) - Engorged alveolar capillaries (top pic) reflect increased pulmonary venous pressure, and pink, acellular material within the alveoli results from transudation of fluid plasma across the alveolar-capillary membrane. These histopathologic findings are most consistent with acute pulmonary edema caused by increased alveolar capillary hydrostatic pressure from left-sided heart failure. This patient likely had dilated cardiomyopathy due to chronic alcohol use and had an acute exacerbation of heart failure, resulting in cardiogenic shock with pulmonary edema and acute respiratory failure Choice A= Aspiration pneumonia may be chemical (e.g., aspiration of gastric contents), or bacterial (e.g., aspiration of oral flora). Histopathology shows neutrophil rich, inflammatory exudates within the bronchioles and alveolar spaces Choice B= Centriacinar emphysema most commonly results from heavy smoking. Enlarged airspaces with macrophage-predominant infiltrates are expected Choice C= Hypersensitivity pneumonitis results from exposure to an environmental irritant (e.g., mold). Histopathology shows lymphocyte-predominant, interstitial inflammation, sometimes accompanied by necrotizing granulomas and loose fibroelastic tissue within the small airways Choice D= Histopathology in pulmonary arterial hypertension typically shows hypertrophy of the pulmonary vascular smooth muscle Choice F= Pulmonary infarction yields hemorrhagic, ischemic necrosis of the lung parenchyma. Histopathology shows a well-demarcated area of alveolar and vascular wall necrosis with cells that lack nuclei, and the alveolar spaces may contain RBCs

-Age: 42 years woman -Brought to the hospital due to right-sided weakness and difficulty speaking. She has a longstanding history of a diastolic murmur, but her medical follow-up has been poor. -She doesn't smoke. CT scan of brain= large ischemic stroke involving the left middle cerebral artery distribution. The patient dies 2 days later due to progressive neurologic deterioration -Autopsy= left atrium shows diffuse fibrous thickening and distortion of the mitral valve leaflets, commissural fusion at the leaflet edges, and narrowing of the mitral orifice. Cause? a)Congenital heart disease b)Degenerative valvular calcinosis c)Infective endocarditis d)Late syphilis e)Rheumatic fever f)Rheumatoid arthritis g)Rupture of the papillary muscle

Answer: Choice E (Rheumatic fever) -This patient's presentation suggests mitral stenosis (MS) due to rheumatic heart disease. Fibrous thickening and fusion of the valve leaflets in chronic rheumatic heart disease following acute rheumatic fever is the most common cause of MS, accounting for up to 99% of cases. There is often a latency period of 10-20 years between the initial episode of rheumatic fever and symptomatic MS, with most patients manifesting during the 4th-5th decade of life -Mitral stenosis can cause atrial enlargement, which may lead to atrial fibrillation and/or atrial mural thromboses. Thrombi dislodged from the wall of the left atrium may later cause an embolic stroke, which likely occurred in this patient. Cardiac auscultation in mitral stenosis often reveals a loud first heart sound (S1), an early diastolic sound (opening snap), followed by a mid-diastolic murmur from turbulent flow across the mitral valve Choice A= Congenital endocardial cushion defects can result in mitral regurgitation. However, congenital mitral valve stenosis is rare and untreated newborns and infants typically have a grim prognosis Choice B= Degenerative calcific deposits most commonly develop in the mitral valve annulus in women over age 60 and generally do not impair valve function. This patient's autopsy did not show any peripherally located nodular calcific deposits Choice C= Embolization of vegetations due to infective endocarditis can cause a stroke or septic infarct resulting in a brain abscess. However, infective endocarditis is usually associated with large friable vegetations on the valve cusps along with destruction (rather than fibrosis) of the valve leaflets Choice D= Tertiary syphilis may result in aortitis, aortic aneurysm, and/or aortic regurgitation. Mitral valve lesions are not commonly observed Choice F= Although cardiac involvement is uncommon in RA, patients can develop pericarditis and myocarditis. Rheumatoid nodules can also develop in cardiac tissue (including valvular structures), but these are rarely symptomatic Choice G= Rupture of papillary muscle (whether due to bacterial endocarditis, a connective tissue disease, or MI) leads to prolapse of the valve leaflets and mitral regurgitation (holosystolic or early systolic murmur)

-Age: 20 years student -Collapses suddenly in class and dies, despite resuscitation efforts. -Review of the medical chart= unremarkable except for a family history of sudden cardiac death. -Autopsy is performed and myocardial histology is shown -Complete genotyping is most likely to reveal a pathologic mutation that affects a protein belonging to which of the following structures? (learn don't answer) a)Cell junction complex b)Ion channel c)Mitochondria d)Nuclear envelope e)Sarcomere

Answer: Choice E (Sarcomere) -Histologic examination of this patient's myocardium is consistent with hypertrophic cardiomyopathy (HCM), which is characterized by cardiomyocyte hypertrophy with haphazard cellular arrangement and prominent interstitial spacing with fibrosis. HCM is caused by genetic mutation affecting structural proteins of the cardiac sarcomere, most commonly beta-myosin heavy chain and myosin-binding protein C. Approximately 50% of cases result from recognized familial mutations with autosomal dominant inheritance and variable expression -HCM is one of the most common causes of sudden cardiac death (SCD) in young adults. Although SCD due to HCM often occurs with exertion, it can occur at rest. It is believed that cardiomyocyte disarray and fibrosis alters the spatial relationship of intercalated discs (the primary mediators of organized cardiac conduction), increasing susceptibility to ventricular arrhythmia (e.g., ventricular tachycardia, Vfib). Chronic myocardial ischemia, which can occur due to insufficient blood supply to the hypertrophied myocardium also contributes to interstitial fibrosis and the risk of arrhythmia. Choice A= Arrhythmogenic right ventricular cardiomyopathy results from mutations affecting proteins that make up desmosomes (e.g., plakoglobin, desmoplakin). There is fibrosis and scarring of the right ventricle that predisposes to ventricular arrhythmias and SCD; however, histology does not demonstrate the cardiomyocyte hypertrophy or marked cellular disarray seen in HCM Choice B= Long QT syndrome and Brugada syndrome result from mutations affecting cardiac ion channels. Both disorders can lead to ventricular arrhythmias and SCD; however, they do not involve structural derangement of the myocardium Choices C and D= Mutations affecting mitochondria typically interfere with energy production via aerobic metabolism and a number of such diseases (e.g., myoclonic epilepsy with ragged red fibers, Barth syndrome) can lead to dilated cardiomyopathy and SCD. Mutations affecting components of the nuclear envelope (e.g., lamin A) are also a potential cause of dilated cardiomyopathy complicated by conduction defects and SCD. However, these cardiomyopathies are far less prevalent and do not involve the same structural derangement seen in HCM

-Age: 39 years man -Comes to the ED with anterior chest pain; he had felt well until the pain developed 4 days ago -Pain is sharp and makes it difficult to take a deep breath. Since yesterday, he has also felt out of breath. Medical history is unremarkable -The patient's father died of MI at age 52, and his mother suffers from rheumatoid arthritis -Temp. (37.4C), BP (112/65), pulse (103, regular). Bedside ultrasound= moderate pericardial effusion. Cause of patient's current condition? a)Autoimmune disease b)Coronary artery disease c)Gram-positive cocci d)Malignancy e)Viral infection

Answer: Choice E (viral infection) -This patient most likely has acute viral pericarditis; viral infection is the most common cause of pericarditis and many viruses (e.g., adenovirus, coxsackievirus, echovirus, influenza virus) have been implicated. Because the viral infection often cannot be confirmed, presumed viral pericarditis is sometimes referred to as idiopathic. Pericarditis typically presents with substernal pleuritic chest pain that may radiate to the bilateral scapulae posteriorly. The pain is typically worse when lying flat and improves with sitting up and leaning forward. Fever is common but often not present. -Cardiac auscultation in acute viral pericarditis classically reveals a triphasic pericardial friction rub (occurring during atrial systole, ventricular systole, and early ventricular diastole); however, the rub may be absent, especially in the presence of significant pericardial effusion. Mild to moderate-sized pericardial effusion is common and can rarely lead to cardiac tamponade. ECG characteristically demonstrates diffuse ST elevation caused by inflammation of the ventricular myocardium Choice A= Autoimmune disease (e.g., SLE, RA) is a potential cause of pericarditis, but it is less common than viral pericarditis and is less likely in the absence of a personal history of autoimmune disease (pericarditis is only rarely the presenting manifestation of autoimmune disease) Choice B= MI due to coronary artery disease can lead to peri-infarction pericarditis (a localized reaction to transmural myocardial necrosis) within 2-4 days or to Dressler syndrome (an autoimmune response to infarction-induced antigens) within weeks to months. Despite a family history of CAD, MI is relatively unlikely in this man age <40 years Choice C= Gram-positive cocci (e.g., S. aureus, S. pneumoniae) in rare cases can cause pericarditis (purulent pericarditis) via direct extension of pneumonia or hematogenous bacterial spread. Affected patients are typically quite ill and present with high fever and sepsis Choice D= Malignant pericarditis can occur in the setting of metastatic cancer (e.g., lung cancer, breast cancer); however, it is less likely to present with pain and inflammation and more likely to present with symptoms of fluid accumulation (i.e., dyspnea due to subacute cardiac tamponade). Malignant pericarditis is also relatively uncommon compared to viral pericarditis and is less likely in this relatively young man

-Age: 45 years man -Comes to the ED because of severe chest pain, diaphoresis, and palpitations. The patient dies two hours after the onset of his symptoms -Autopsy reveals 100% occlusion of the left anterior descending artery. At the time of the patient's death, light microscopy of the affected myocardium would most likely demonstrate which of the following? a)Edema and punctuate hemorrhages b)Myocyte hypereosinophilia c)Dense interstitial neutrophil infiltrate d)Extensive macrophage phagocytosis of the dead cells e)Fibrovascular granulation tissue with neovascularization f)Dense collagen scar g)Normal myocardium

Answer: Choice G (Normal myocardium) -The cause of death was most likely an acute MI. In MI, changes on light microscopy are usually not apparent until 4 hours after the onset of severe ischemia. Although a variable waviness of myofibrils at the border of the infarct (due to myofibril relaxation) might be observed before this, more definite signs of early coagulation necrosis, such as cytoplasmic eosinophilia and nuclear pyknosis, take at least 4 hours to develop -Other potentially lethal causes of chest pain, diaphoresis, and palpitations/tachycardia include aortic dissection and/or rupture, massive pulmonary embolism, and tension pneumothorax. The chest pain of aortic dissection is typically tearing in nature and often radiates to the back. Dyspnea is typically the most prominent symptom in the event of a pulmonary embolus or tension pneumothorax Choices A and B= Cytoplasmic hypereosinophilia on light microscopy is one of the earliest signs of coagulative necrosis of cardiac myocytes. It begins approximately 4 hours after the onset of lethal ischemia. Light microscopy may also reveal edema and punctate hemorrhages in the infarcted myocardium starting also at about 4 hours after the ischemic event Choice C= An interstitial infiltrate of neutrophils around a zone of MI is not seen until 1 to 3 days after the onset of severe ischemia Choice D= Extensive macrophage phagocytosis of the dead cells generally does not develop until at least 5 days after MI, and is more prominent during post-infarction days 7 to 10 days Choice E= Fibrovascular granulation tissue with neovascularization generally begins to develop 7 days after MI, and is most prominent on post-infarction days 10 to 14 Choice F= Increased collagen deposition and decreased cellularity in a zone of infarcted myocardium is not generally evident until 2 weeks post-infarction. Fibrosis continues during weeks 2 to 8, producing a dense collagenous scar by 2 months post-infarction

-Age: 1 week boy -Brought to the office for his first primary care visit following an uncomplicated vaginal delivery at 40 weeks gestation. The boy was born to a 30 years woman who took prenatal vitamins throughout the pregnancy. -She was diagnosed with gestational diabetes mellitus at 28 weeks gestation, which was treated with dietary modification and exercise. -The nursery course was uncomplicated, and the boy was discharged at around 30 hours of life following observation of appropriate breastfeeding, voiding, and stooling. -His weight, length, and head circumference (50th percentile). Physical exam: grade II/VI harsh, holosystolic murmur best heard at the left mid to lower sternal border. -Birth records show that no murmur was heard by 2 different health care providers in the newborn nursery. Diagnosis? a)Coarctation of the aorta b)Ebstein's anomaly c)Patent ductus arteriosus d)Peripheral pulmonary stenosis e)Primum-type atrial septal defect f)Secundum-type atrial septal defect g)Tetralogy of Fallot h)Wolff-Parkinson-White syndrome i)Ventricular septal defect

Answer: Choice I (ventricular septal defect) -This patient's presentation is consistent with a VSD, the most common congenital heart lesion. Echocardiography should be performed to confirm the location and size of the VSD -Patients with small VSDs, typically have a loud, "blowing", holosystolic murmur at the mid to lower left sternal border and no symptoms. The murmur is not usually detectable at birth but becomes audible around age 4-10 days as pulmonary vascular resistance (PVR) continues to decline, permitting significant left-to-right shunting. Most small VSDs are hemodynamically insignificant and close spontaneously. -In contrast, moderate to large VSDs usually present with heart failure, failure to thrive, and diaphoresis with feeding. Large VSDs may have no murmur as RV pressures are similar to those of the LV. In addition, large VSDs rarely close spontaneously and can result in chronic pulmonary hypertension and cyanosis (Eisenmenger syndrome) if the defect is not closed early in life Choice A= Coarctation of the aorta is a non-cyanotic congenital heart disease that can present in infancy with shock when the patent ductus arteriosus closes. Children and adults who present in later life tend to have milder coarctation that manifests as upper body hypertension and collateral intercostal vessels Choice B= Ebstein's anomaly is characterized by displacement of a malformed tricuspid valve into the RV. Soon after birth, it presents as cyanosis and heart failure from severe tricuspid regurgitation Choices C, E, and F= ASDs (wide, fixed splitting of S2) and patent ductus arteriosus (continuous murmur) are non-cyanotic congenital heart diseases. These initially cause left-to-right shunting, although untreated large defects may become complicated by Eisenmenger syndrome Choice D= Peripheral pulmonary stenosis (i.e., pulmonary branch stenosis) is a common cause of innocent murmur that is heard after the first day of life when the patent ductus arteriosus is closed. The murmur results from the relative hypoplasia of branch pulmonary arteries compared to the relatively large main pulmonary artery. Auscultation may reveal a low-grade, mid-systolic, high-pitched, or blow ejection murmur that is best heard on the pulmonary area with radiation to the axilla and back Choice G= Tetralogy of Fallot symptoms depend on the severity of RV outflow obstruction. Those with severe obstruction present with profound cyanosis in infancy, but patients with moderate obstruction have "tet" spells later in childhood. Choice H= The hallmark of WPW syndrome is preexcitation, which manifests on ECG as a short PR interval, delta wave, and widening of the QRS complex. Arrhythmias are associated with WPW but not with murmurs

-Age: 5 weeks boy -Evaluated for a week-long history of rapid breathing and tiring with feeds. -The infant was born at home after an uneventful pregnancy. The mother declined all prenatal testing and ultrasound evaluations. -Normal temp., BP (76/38), pulse (124/min), RR (66/min). Cardiovascular examination is notable for a hyperdynamic precordium, and a mid-diastolic rumble at the left sternal border, and a 3/6 holosystolic murmur in the apex that radiates to the left axilla. -An echocardiogram shows defects in the lower part of the interatrial septum and the interventricular septum. This patient's condition is most likely associated with? a)22q11.2 deletion b)Autosomal trisomy c)Fibrillin mutation d)Frataxin mutation e)Hamartin gene mutation f)Sex chromosome monosomy

Answer: Choice B (Autosomal trisomy) -This patient's echocardiogram suggests a complete atrioventricular (AV) canal defect, the most common type of cardiac defect in patients with Down syndrome, an autosomal trisomy. Failure of endocardial cushion fusion results in ostium primum atrial septal defect, a ventricular septal defect and a single AV valve. Significant left-to-right shunting and AV valve regurgitation lead to excessive pulmonary blood flow and symptoms of heart failure (e.g., tachypnea, poor feeding). Auscultator findings of AV valve regurgitation (holosystolic, best heard at apex) and increased pulmonary venous return (mid-diastolic rumble) are characteristic. Choice A= DiGeorge syndrome (22q11.2 deletion) is characterized by thymic aplasia (T cell deficiency) and hypoparathyroidism (hypocalcemia). It is associated with tetralogy of Fallot, truncus arteriosus, and transposition of the great vessels Choice C= Marfan syndrome (fibrillin-1 mutation) is associated with cystic medial necrosis of the aorta, which may result in dissecting aortic aneurysms and aortic valve regurgitation. Mitral valve prolapse is also common, but septal defects are not Choice D= Mutations in frataxin, a mitochondrial protein important in iron homeostasis and respiratory function, cause Friedreich ataxia. It is characterized by spinocerebellar degeneration and is associated with hypertrophic cardiomyopathy, but not septal defects Choice E= Mutations in tuberin and hamartin are seen in tuberous sclerosis. These patients may develop cardiac rhabdomyomas in ventricular walls and AV valves, cutaneous angiofibromas (adenoma sebaceum), central nervous system hamartomas, and renal cysts. However, septal defects are not characteristic Choice F= A missing sex chromosome (monosomy) is seen in Turner syndrome (45, XO), which is associated with bicuspid aortic valve (most common cardiac lesion) and coarctation of the aorta

-Age: 56 years woman -Unknown medical history is brought to the ED in an unresponsive state. Paramedics were called for sudden-onset, left-sided weakness, and when they arrived at the patient's house, she was having a generalized tonic-clonic seizure. -Antiseizure medications were given, and the patient was subsequently intubated. Evaluation shows a comatose woman with left hemiplegia. Neuroimaging reveals a large infarction in the right middle cerebral artery territory with cerebral edema. -Despite aggressive interventions, the patient dies from brain herniation. Autopsy= multiple, small, nondestructive masses attached to the edges of the mitral valve leaflets. Microscopy= masses are composed of platelet-rich thrombi, but no organisms are present. Which of the following is most closely associated with the patient's heart valve findings? a)Advanced malignancy b)Chronic liver failure c)Dilated cardiomyopathy d)IV drug use e)Large-vessel vasculitis

Answer: Choice A (Advanced malignancy) -The autopsy finding of sterile platelet-rich thrombi attached to the mitral valve leaflets is characteristic of nonbacterial thrombotic endocarditis (NBTE) (marantic endocarditis). The pathogenesis of NBTE is though to begin with valvular endothelial injury caused by circulating inflammatory cytokines, which triggers platelet deposition in the presence of an underlying hypercoagulable state. NBTE is most commonly associated with advanced malignancy (especially mucinous adenocarcinoma) and SLE (Libman-Sacks endocarditis); less commonly, it can occur with antiphospholipid syndrome, DIC, and extensive burns -Histologically, NBTE vegetations consist of bland thrombus with strands of fibrin, immune complexes, and mononuclear cells (white thrombus). The vegetations typically affect the left-sided heart valves (mitral or aortic) and are often asymptomatic; however, systemic embolization (e.g., stroke, acute limb ischemia) can occur and is the most common presentation of NBTE Choice B= Chronic liver failure (e.g., cirrhosis) can be complicated by portal vein thrombosis; however, it is not associated with NBTE Choice C= DCM is characterized by ventricular enlargement and reduced systolic function. It can occasionally lead to LV mural thrombus with risk of systemic embolization, but it is not associated with NBTE Choice D= IV drug use increases the risk of bacterial endocarditis, not NBTE. Injected particulate matter (e.g., dirt) is thought to cause microdamage to the right-sided heart valves before being filtered out by the lungs, facilitating bacterial attachment to primarily the tricuspid valve Choice E= Temporal (giant cell) arteritis, a large vessel vasculitis that typically presents with headaches and fever, is associated with an increased risk of venous thromboembolic disease. Although associated NBTE has been reported, it is quite rare

-Age: 30 years man -Brought to the ED due to left-sided weakness. His left arm was sluggish after awakening, and he had difficulty walking to the bathroom -The patient has no known medical conditions but has a 2-month history of positional dyspnea. Vital signs are within normal limits. Exam= strength of the left upper and left lower extremity is reduced, and touch/temperature sensation is similarly diminished. Cardiovascular exam= intermittent murmur at the apex without carotid bruits -Echocardiography: mobile, 3-cm left atrial mass obstructing the mitral opening during diastole. Histology of mass will show? a)Amorphous extracellular matrix b)Malignant epithelial cells with necrotic debris c)Malignant vascular spindle cells d)Organized thrombus e)Well-organized, mature adipocytes

Answer: Choice A (Amorphous extracellular matrix) -This patient's left atrial mass on echo is consistent with an atrial myxoma, this benign tumor is the most common primary cardiac neoplasm. Approximately 80% originate in the left atrium, and they can present with systemic embolization (e.g., stroke, as in this patient) from tumor fragments passing into the systemic circulation. The tumors may also lead to constitutional symptoms (e.g., fatigue, weight loss, low-grade fever) resulting from cytokine release, and they may cause transient mitral valve obstruction, leading to symptoms that can mimic mitral valve stenosis (e.g., dyspnea, hemoptysis) -Physical exam often reveals an intermittent or positional mid-diastolic murmur ("tumor plop") that results from the motion of the tumor mass obstructing the mitral valve orifice. Histopathologic examination of a myxoma reveals amorphous extracellular matrix with scattered stellate or globular myxoma cells within abundant mucopolysaccharide (myxoid) ground substance containing chondroitin sulfate and hyaluronic acid. Because of their high vascularity, these tumors often demonstrate areas of hemorrhage accompanied by brown, hemosiderin-laden macrophages Choices B and C= Primary malignant cardiac tumors are quite rare; those that occur are usually either sarcomas or lymphomas. Malignant vascular spindle cells suggest primary cardiac angiosarcoma, which usually arises in the RA and is exceedingly rare. Malignant epithelial cells with necrotic debris suggest secondary involvement of the heart from a metastatic malignancy (e.g., melanoma, lung cancer, breast cancer). Metastatic spread to the heart usually involves the pericardium and rarely creates a mobile intracardiac mass Choice D= An organized thrombus can form in the LA or left atrial appendage in patients with atrial flutter or fibrillation and may cause stroke due to systemic embolization. However, thrombi are typically adherent to the cardiac endothelium; a mobile mass in the LA is more characteristic of a myxoma Choice E= Lipomas are benign tumors composed of well-organized, mature adipocytes. They are the 2nd most common type of primary cardiac neoplasm; however, unlike myxomas, lipomas, rarely embolize

-A researcher center studying cardiovascular pathology is conducting trials in which experimental rabbits are fed sweet peas containing substances that alter connective tissue synthesis and maturation -The animals are monitored for several weeks prior to euthanasia. Autopsy shows myxomatous degeneration with pooling of proteoglycans in the media layer and an intact intimal layer in large arteries. The pathologic findings seen in experiment are most similar to? a)Aortic aneurysm b)Atherosclerosis c)Berry aneurysm d)False aneurysm e)Giant cell arteritis f)Malignant hypertension

Answer: Choice A (Aortic aneurysm) -Myxomatous changes in the media of large arteries is found in cystic medial degeneration, a form of pathologic weakening of connective tissue that occurs in large arteries. Medial degeneration is characterized by the fragmentation of elastic tissue and separation of the elastic and fibromuscular components of the tunica media by small, clefted spaces that become filled with amorphous extracellular matrix (i.e., mucopolysaccharides) -Marfan syndrome is a frequent cause of cystic medial degeneration in younger patients. It is characterized by an autosomal dominant defect in the extracellular glycoprotein fibrillin-1, which is a major component of the extracellular matrix microfibrils that form the scaffolding for elastic fibers. Mutation of the fibrillin-1 gene results in elastic tissue fragmentation, which predisposes affected patients to aortic aneurysms (with an intact intima) and dissections (with an intimal tear) -Beta-aminopropionitrile (a chemical found in certain kinds of sweet peas) causes inhibition of lysyl oxidase, an enzyme responsible for cross-linking elastin fibers and collagen fibers Ingestion of this compound can cause connection tissue disruption in the aorta that mimics the myxomatous degeneration seen in patients with Marfan syndrome Choice B= Atherosclerosis is the formation of intimal atheromas in response to chronic or repetitive endothelial injury. Myxomatous degeneration does not play a role in the pathogenesis of atherosclerosis Choice C= The pathogenesis of berry aneurysms is multifactorial. Contributing factors include hemodynamic stress, hypertension, and certain connective tissue disorders. Type IV Ehlers-Danlos syndrome (which results in abnormalities in type III collagen) predisposes patients to development of berry aneurysms. Myxomatous degeneration is not associated with berry aneurysms Choice D= A false aneurysm, or pseudoaneurysm, is the result of breach in the continuity of all the layers of a blood vessel, leading to blood leakage outside the vessel wall (e.g., hematoma). The intima would not be intact in a false aneurysm Choice E= Giant cell arteritis involves granulomatous inflammation of the media and fragmentation of the internal elastic lamina, perhaps due to autoimmunity to elastin. It is not related to myxomatous changes of the arterial wall Choice F= Hyperplastic arteriolosclerosis of the renal arterioles is associated with malignant hypertension. It results from concentric lamellar reduplication of the intimal smooth muscle cells ("onion-skinning"). Myxomatous degeneration is not a contributing factor.

-Age: 37 years man -Found unresponsive under a tree during a thunderstorm. He is not breathing when paramedics arrive on the scene. On examination, his pupils are fixed and dilated. Several cutaneous erythematous marks in a fern-leaf pattern are seen on his lower-extremities. 2nd degree burns are present on both arms -Cardiopulmonary resuscitation is started; however, the patient does not respond and is pronounced dead on arrival at the hospital. Which of the following is most likely to be the primary cause of his death? a)Cardiac arrhythmias b)Extensive deep-tissue burns c)Intracranial hemorrhage d)Myocardial contusion e)Pulmonary barotrauma

Answer: Choice A (Cardiac arrhythmias) -The finding of an unconscious person on wet ground or under tress during a thunderstorm suggests a lightning injury. This patient's Lichtenberg figures (erythematous cutaneous marks in a fern-leaf pattern) and 2nd degree burns confirm the suspicion. -Although lightning injuries are rare, they are associated with a 25% fatality rate, and more than 70% of survivors have long-lasting complications. 2/3 of lightning-related deaths occur within the first hour following injury; the most common causes are fatal arrhythmias and respiratory failure. A comprehensive and thorough physical examination should always be performed on lightning strike victims as injury patterns are widely variable and skin findings can significantly underestimate the extent of internal injury Choice B= Superficial burns are common in lightning injury. In contrast, deep-tissue burns are relatively rare due to the short duration of electrical contact and the "flashover effect", the tendency of lightning to travel over the skin surface and discharge to the ground. This effect is responsible for the formation of Lichtenberg figures Choice C= Intracranial hemorrhage only rarely occurs following lightning injury. Lightning strikes can damage the autonomic system and cause fixed and dilated or asymmetric pupils. These signs may not accurately reflect the extent of neurologic injury and should not be a reason to stop resuscitation Choices D and E= Lightning induces rapid heating of the surrounding air, generating shock waves that can travel through the body and cause mechanical trauma. As a result, patients should be thoroughly assessed for skeletal fractures. Myocardial contusion and pulmonary barotrauma occur less frequently and would be extremely rare causes of death

-Age: 61 years woman -Hospitalized due to chest pressure and shortness of breath. 2 days ago, her husband died in a car accident. The patient's medical history: hypertension, hyperlipidemia, and gout. -Five years ago= diagnosed with breast cancer and treated with surgery and chemotherapy. BP (165/90), pulse (95 and regular). Physical exam: unremarkable -ECG: normal sinus rhythm with T wave inversions in the anterior leads. Echocardiogram= hypokinesis of the apical wall with decreased LV ejection fraction -Diagnostic coronary angiography= no evidence of obstructive coronary artery disease. The patient is treated medically and discharged home. 4 weeks later, she has no symptoms. Repeat echocardiography= LV wall motion and ejection fraction. Cause of patient's initial presentation? a)Catecholamine-induced myocardial stunning b)Ischemia-induced transmural myocardial necrosis c)Myocardial hypertrophy and fibrosis due to uncontrolled hypertension d)Myocardial infiltration by mature lymphocytes e)Toxin-induced myocardial injury

Answer: Choice A (Catecholamine-induced myocardial stunning) -This patient likely has stress-induced (takotsubo) cardiomyopathy, which is characterized by hypokinesis of the mid and apical segments and hyperkinesis of the basal segments of the LV leading to systolic dysfunction and reduced ejection fraction. The condition is likely caused by a catecholamine surge in the setting of physical or emotional stress (e.g., death of a loved one; heart-broken syndrome). Catecholamines may cause microvascular spasm leading to ischemia and myocardial stunning or they may cause direct myocardial dysfunction. The resulting segmental LV dysfunction creates a characteristic balloon shape on echocardiogram that mimics that of an octopus trap (takotsubo means "octopus trap" in Japanese) -Stress-induced cardiomyopathy most commonly affects postmenopausal women. Patients typically have chest pain that can mimic an MI and may also have symptoms of heart failure (e.g., dyspnea, lower extremity swelling). ECG often shows evidence of ischemia (e.g., ST elevation, T wave inversion) in the anterior precordial leads; however, coronary angiography typically reveals an absence of obstructive coronary artery disease. The condition usually resolves within several weeks with supportive treatment only Choice B= Ischemia-induced transmural myocardial necrosis is seen in patients with acute MI, which is unlikely in this patient with no evidence of obstructive coronary artery disease on coronary angiography Choice C= LV hypertrophy due to uncontrolled hypertension typically leads to diastolic dysfunction without systolic wall motion abnormalities Choices D and E= Myocardial infiltration with lymphocytes is seen in patients with acute viral myocarditis. Toxin-induced myocardial injury can occur with exposure to a variety of agents, including alcohol, cocaine, and certain chemotherapy drugs (e.g., anthracyclines). These myocardial insults typically lead to dilated cardiomyopathy with global (rather than segmental) LV systolic dysfunction

-Age: 66 years man -Routine office visit; history: hypertension and osteoarthritis -Smoker (daily for 40 years); high BP, pulse (normal); cardiopulmonary exam: normal -There is a pulsating, central abdominal mass on physical exam. Which of the following pathological conditions is the most likely underlying cause of this patient's abnormal findings? a)Chronic transmural inflammation b)Cystic medial necrosis c)Focal intimal tear d)Malignant endothelial proliferation e)Microbial infection of the aortic wall f)Vasa vasorum endarteritis

Answer: Choice A (Chronic transmural inflammation) -This patient's pulsating, central abdominal mass most likely represents an abdominal aortic aneurysm (AAA), which is a focal dilation of the abdominal aorta that typically occurs below the renal arteries -AAA is associated with severe risk factors (e.g., age> 60 years, smoking, hypertension, male sex, family history) that leads to oxidative stress, vascular smooth muscle apoptosis, and chronic transmural inflammation of the aorta. Inflammatory cells (particularly macrophages) release matrix metalloproteinases and elastases that degrade extracellular matrix components (e.g., elastin, collagen) leading to weakening and progressive expansion of the aortic wall. Furthermore, ischemia of the tunica media may play a role as the infrarenal abdominal aorta has a tenuous vasa vasorum, and atherosclerotic thickening of the intimal layer increases as the diffusion distance for oxygen Choice B= Cystic medial necrosis is characterized by loss of smooth muscle collagen, and elastic tissue with formation of cystic mucoid spaces in the aortic media. Patients with Marfan syndrome develop cystic medial degeneration of the aortic root at a young age, predisposing them to ascending aortic aneurysm and dissection Choice C= An intimal tear in the aortic wall is the primary even in the pathophysiology of aortic dissection. Patients typically present with tearing chest pain that radiates to the back. Proximal dissections involving the ascending aorta can cause unequal blood pressure in the arms, aortic valve regurgitation (e.g., diastolic decrescendo murmur at the right sternal border), and cardiac tamponade Choice D= Malignant endothelial proliferation is characteristic of angiosarcoma, a neoplasm that arises from blood or lymphatic vessels in the subcutaneous tissue and most frequently affects the scalp and face Choice E= Microbial infection (usually bacteria) can cause localized dilation of an artery due to destruction of the vessel wall. Trauma, bacteremic seeding, or septic emboli (mycotic aneurysm) can cause infected aneurysms, which typically present with painful, pulsatile masse and systemic signs such as fever and malaise Choice F= Vasa vasorum endarteritis (endarteritis obliterans) in the thoracic aorta is thought to be the mechanism by which syphilis causes thoracic aortic aneurysm

-Age: 32 years man -Comes to the ED due to severe headaches and vomiting. Soon after, he becomes comatose and expires despite extensive resuscitation efforts. Autopsy= ruptured cerebral aneurysm with extensive intracranial hemorrhage and a congenital heart defect -The patient's presentation is most likely associated with which of the following abnormalities? a)Coarctation of the aorta b)Patent ductus arteriosus c)Primum-type atrial septal defect d)Secundum-type atrial septal defect e)Tetralogy of Fallot f)Ventricular septal defect

Answer: Choice A (Coarctation of the aorta) -The patient died from an intracranial hemorrhage due to a ruptured cerebral aneurysm. Hypertension, genetic conditions (e.g., Ehlers-Danlos syndrome), and lifestyle factors (e.g., cigarette smoking) increase the risk of aneurysm development. Coarctation of the aorta is a congenital heart defect associated with secondary hypertension and is a risk factor for cerebral aneurysm development -Coarctation of the aorta is a narrowing of the aortic arch near the ligamentum arteriosum (remnant of ductus arteriosus) with localized medial and intimal hyperplasia. Critical coarctation (severe narrowing) often presents in the neonatal period with heart failure and cardiogenic shock. However, less severe narrowing allows for distal (but decreased) perfusion to the lower extremities and may present only with asymptomatic upper extremity hypertension in children or adults. Additional symptoms due to this differential in BP include headache, epistaxis, chest pain, and lower extremity claudication -Complications of aortic coarctation include cerebral and aortic aneurysm. Cerebral aneurysm with life-threatening rupture causing subarachnoid hemorrhage is likely due to chronic hypertension. Aortic aneurysms usually develop near the site of coarctation due to embryologic vessel wall abnormalities and may lead to dissection or fatal rupture Choices B, C, D and F= PDA and ASDs and VSDs create a left-to-right shunt that, if untreated, may lead to right-to-left shunting (i.e., Eisenmenger syndrome) due to pulmonary hypertension. This may increase risk of a paradoxical thromboembolism from the venous to arterial circulation directly to the brain, causing an ischemic stroke. However, neither condition is associated with cerebral aneurysm or intracranial hemorrhage Choice E= Tetralogy of Fallot is characterized by pulmonary stenosis, VSD, an overriding aorta, and RV hypertrophy. Almost all patients have cyanosis in infancy or hypercyanotic episodes (i.e., tet spells) in early childhood. The patient's age makes this diagnosis unlikely. In addition, risk of intracranial hemorrhage is not increased with ToF, regardless of age

-Age: 58 years man -Comes to the ED with abrupt onset, severe chest pain that radiates to his back -BP (220/130 in the left arm; 180/100 in right arm); Initial labs: normal serum troponin levels -Electrocardiogram is negative for ST-segment changes; patient's acute condition is triggered by? a)Intimal fatty streak formation b)Intimal tearing c)Medial calcification d)Medial inflammation e)Vasa vasorum obliteration

Answer: Choice B (Intimal tear) -The patient's presentation is most consistent with aortic dissection, which is characterized by severe retrosternal chest pain radiating to the mid-to-upper back that can move downward as the dissection progresses -It is initiated by a tear in the aortic intima that typically extends for about 1-5cm in a transverse or oblique direction. The dissection can progress both proximally and distally as blood is forced through the tear, bisecting through the aortic media. -Dissections involving any portion of the ascending aorta are classified as type A; those confined to the descending aorta are classified as type B. As the dissecting intramural hematoma spreads along the aortic wall, it can compress major arterial branches. This patient's brachial BP discrepancy suggests compromise of the brachiocephalic trunk servicing his right arm -Hypertension is the primary risk factor for aortic dissection. In many patients with longstanding hypertension, there is medial hypertrophy of the aortic vasa vasorum and, consequently, reduced blood flow to the aortic media. This can cause medial degeneration with a loss of smooth muscle cells, leading to aortic enlargement and increased wall stiffness. Both of these changes exacerbate aortic wall stress, which is already increased due to the hypertension itself. This synergistic increase in aortic wall stress greatly increases the risk of intimal tearing and subsequent development of aortic dissection Choice A= An intimal fatty streak is the initial lesion of atherosclerosis. Some fatty streaks progress to frank atheromas, which can progressively weaken the underlying media of the aortic wall. However, atherosclerosis predisposes more to aortic aneurysm formation than to aortic dissection Choice C= Monckeberg sclerosis (medial calcific sclerosis; bottom pic) is a form of arteriolosclerosis characterized by calcific deposits in the medial layer of muscular arteries. These lesions generally occur in patients age >50 and are thought to cause isolated systolic hypertension due to arterial hardening Choice D and E= Aortitis with medial inflammation can be caused by autoimmune conditions such as Takayasu and giant cell arteritis. Tertiary syphilis can also cause a form of aortitis characterized by obliterative endarteritis of the vasa vasorum. These conditions weaken the aortic wall and can predispose to aortic aneurysms, and rarely, dissections. However, hypertension is a more important risk factor for the development of intimal tearing, the primary triggering event of an aortic dissection

-Age: 56 years man -Comes to the clinic for a routine checkup. History: hypertension, type 2 diabetes, hyperlipidemia, and mild intermittent asthma -The patient currently takes no medications and has not seen a physician in 7 years. He reports feeling well. BP (152/101) and pulse (87). Waist circumference (110cm). Lab results are as follows: low density lipoprotein (161mg/dl), fasting blood glucose (201mg/dl), Hb1Ac (7.4%) -Which of the following vascular beds is most likely to carry the highest atherosclerotic burden in this patient? a)Coronary arteries b)Internal thoracic arteries c)Mesenteric arteries d)Popliteal arteries e)Renal arteries

Answer: Choice A (Coronary arteries) -This patient has multiple risk factors (e.g., hypertension, diabetes mellitus, hyperlipidemia) that contribute to atherosclerosis. Atherosclerosis is a pathophysiologic process that begins in childhood with the development of focal intimal thickening due to accumulation of lipid-laden macrophages and extracellular matrix ("fatty streaks"). With advancing age, fatty streaks transition into atherosclerotic plaques, such as fibrous cap atheromas and fibrous plaques. More advanced lesions develop a lipid-rich necrotic core and areas of calcification -Atherosclerosis is a chronic disease of the arterial walls and involves all major vascular beds. The most susceptible vascular regions involve bends and branch points that encourage turbulent blood flow, which disrupts vascular wall integrity and leads to endothelial cell dysfunction. Turbulent blood flow also leads to decreased shear stress on the vascular walls and prolonged endothelial contact with cholesterol particles. -The hemodynamics of the lower abdominal aorta and the coronary arteries make these vascular beds the most susceptible to atherosclerosis of all major vascular beds in the body. In fact, atherosclerotic lesions (e.g., intimal thickening, fatty streaks) occur in these vessels as early as the 2nd decade of life Choice B= The internal thoracic arteries (previously known as the internal mammary arteries) are relatively resistant to developing atherosclerosis and are often used as conduits in patients undergoing coronary artery bypass grafting Choices C and E= The mesenteric and renal arteries are susceptible to developing atherosclerosis; however, the burden of disease tends to be less severe than in the coronary circulation Choice D= Along with the carotid arteries, the popliteal arteries are among the most susceptible to atherosclerosis in the body. However, these arteries are not as susceptible to atherosclerosis as the aorta and coronary arteries

-Age: 36 years man -Comes to ED after briefly losing consciousness while watching TV half an hour ago -The patient had no preceding chest pain or shortness of breath, but he has been having recurrent palpitations over the past several days. -2 weeks ago= non-itchy rash on his thigh that he treated with an over-the counter steroid cream. Medical history= unremarkable -No family history of heart disease or sudden cardiac death. The patient recently began working as a forest ranger in Pennsylvania. He doesn't smoke -BP (normal), pulse (46); The lungs are clear on auscultation and heart sounds are normal -ECG= complete AV block; Echocardiography= normal ventricular function with no major valvular disease. Underlying cause? a)Disseminated spirochetal infection b)Granulomatous myocardial inflammation c)Missense mutation of a sarcomere protein d)Myocyte potassium channel mutation e)Viral infection-induced cardiac injury

Answer: Choice A (Disseminated spirochetal infection) -The patient most likely has early disseminated Lyme disease. Lyme disease is caused by the bacterial spirochete Borrelia burgdorferi, which is transmitted by the Ixodes tick that is endemic to the northeastern and midwestern US. Most cases initially present with a non-itchy, bull's eye rash, known as erythema migrans, at the site of infection. Left untreated for several weeks or more, a small percentage of those infected will develop cardiac involvement (Lyme carditis) -Lyme carditis most commonly manifests with varying degrees of AV conduction block. Many patients remain asymptomatic; however, patients who develop complete AV block are likely to have symptoms of dyspnea, lightheadedness, or syncope. Treatment for symptomatic AV block due to Lyme carditis typically requires hospitalization with temporary pacing and the administration of IV antibiotics (ceftriaxone is the drug of choice) Choice B= Granulomatous myocardial inflammation occurs in cardiac sarcoidosis, and AV conduction block is the most common disease manifestation. However, this patient's recent rash and residence in an endemic area make Lyme carditis more likely Choice C= A missense mutation affecting the cardiac sarcomere protein beta-myosin heavy chain is a common cause of hypertrophic cardiomyopathy. Myocardial disarray creates a propensity for ventricular arrhythmias, but AV conduction block is not typical Choice D= Congenital long QT syndrome results from a mutation affecting cardiomyocyte potassium channels. It can cause syncope or sudden cardiac death, typically via the triggering of torsades de pointes (polymorphic ventricular tachycardia). AV conduction block would be an unusual manifestation Choice E= Viral infection of the heart typically manifests as myocarditis. The most common symptomatic presentation is decompensated heart failure due to dilated cardiomyopathy. AV block is not typical

-Age: 75 years man -Comes to the office due to worsening dyspnea and fatigue on exertion over the last 6 months. Recently, he has had severe lightheadedness during physical activity. BP (125/65) and pulse (65, regular). -Physical exam reveals a harsh ejection-type systolic murmur at the base of the heart radiating to the neck. The 2nd heart sound is diminished in intensity. A 4th heart sound is heard at the cardiac apex. Cause of this patient's heart condition? a)Extensive valve calcification with impaired leaflet mobility b)Fusion of valve commissures due to repetitive inflammation c)Hypertrophy of the basal interventricular septum with subaortic obstruction d)Infectious vegetations attached to the valve cusps e)Myxomatous valve degeneration with leaflet prolapse

Answer: Choice A (Extensive valve calcification with impaired leaflet mobility) -This patient's presentation (fatigue, progressive dyspnea, presyncope with exertion, and a cardiac murmur)= aortic valve stenosis (AS). In AS, the heart is unable to adequately increased cardiac output during exercise, causing pulmonary vascular congestion and reduced mean arterial pressure. These changes cause dyspnea and lightheadedness, respectively. -AS can also chronically lower cardiac output, causing an increased tendency toward fatigue in general. AS murmur is usually a harsh ejection-type systolic murmur heard best at the base of the heart in the "aortic area" (second right intercostal space) with radiation to the carotid arteries. Second heart sound (S2) is diminished in intensity due to reduced mobility of the aortic leaflets, and a fourth heart sound (S4) may be heard due to decreased compliance of the hypertrophic myocardium. -Calcific degeneration of the trileaflet aortic valve is the most common cause of valvular AS in developed nations. AS is characterized by progressive aortic valve leaflet thickening and calcification, leading to restricted leaflet excursion and mobility. Congenital heart disease, such as bicuspid aortic valve, can also lead to AS due to accelerated valve degeneration. On the other hand, rheumatic valve disease is a common cause of AS in developing nations and is characterized by fusion of valve commissures due to repetitive inflammation (Choice B). Rheumatic valve disease usually presents at an earlier age and involves the mitral valve as well. Choice C= Hypertrophy of the basal interventricular septum with subaortic obstruction is the pathophysiologic process in patients with hypertrophic cardiomyopathy. Physical examination reveals a harsh crescendo-decrescendo murmur heard best at the apex and lower sternal border. The murmur does not radiate to the neck, and S2 is usually normal Choice D= Infective endocarditis can lead to aortic regurgitation (AR) due to inadequate coaptation of the aortic leaflets. AR murmur is an early, decrescendo, high pitched, blowing diastolic murmur that begins immediately after A2. It is best heard along the left sternal border at the 3rd and 4th intercostal spaces Choice E= Myxomatous degeneration of the mitral valve leaflets leads to mitral valve prolapse. Cardiac auscultation usually reveals a non-ejection click and mid-to-late systolic murmur of mitral regurgitation

-Age: 54 years man -Comes to the ED due to severe fatigue and dyspnea. Long history of progressively worsening heart failure that has been resistant to treatment (including diuretics) -He was treated with chest radiation 10 years ago for non-Hodgkin lymphoma and has been in remission since then. The patient is admitted to the hospital, but his condition continues to deteriorate despite aggressive therapy -He dies 3 days later, and autopsy is performed. Gross inspection= dense, thick, fibrous tissue in the pericardial space between the visceral and parietal pericardium -Which of the following signs would most likely have been detected during a physical exam of this patient just prior to his death? a)Kussmaul sign b)Loud P2 c)Paradoxical splitting of S2 d)S3 gallop e)Sustained left parasternal lift

Answer: Choice A (Kussmaul sign) -The autopsy finding of thick, fibrous tissue in the pericardial space is consistent with constrictive pericarditis, a potential complication of chest radiation therapy for NHL. The dense, rigid pericardial tissue encases the heart and restricts ventricular filling, causing low cardiac output (manifesting with fatigue and dyspnea on exertion) and progressive right-sided heart failure (manifesting with hepatomegaly and peripheral edema) -Physical examination in constrictive pericarditis typically shows elevated jugular venous pressure (JVP) with prominent x and y descents and a pericardial knock (early diastolic sound that occurs before S3) and may also demonstrate pulsus paradoxus (>10mmHg drop in SBP during inspiration). In addition, Kussmaul sign may be present. Under normal circumstances, the decrease in intrathoracic pressure during inspiration increases the venous pressure to the right side of the heart and lowers JVP. However, in constrictive pericarditis, the rigid pericardium prevents the right side of the heart from accommodating increased venous return, which leads to a paradoxical rise in JVP during inspiration, referred to as Kussmaul sign Choice B= A loud pulmonic component of the second heart (P2) is heard in patients with pulmonary hypertension Choice C= During inspiration, venous return to the right side of the heart is increased, resulting in later closure of the pulmonic valve and physiologic splitting of the aortic (A2) and pulmonic components of S2. Paradoxical splitting occurs when cardiac pathology (e.g., aortic stenosis, LBBB) delays closure of the aortic valve, causing A2 to occur noticeably later than P2 Choice D= An S3 gallop occurs due to the sudden deceleration of blood as it enters a dilated ventricle, it is typically heard in patients with dilated cardiomyopathy or severe mitral regurgitation Choice E= A sustained left parasternal lift is most commonly a sign of RV hypertrophy, such as occurs with chronic RV pressure overload (e.g., pulmonary hypertension, pulmonic stenosis) or volume overload (e.g., tricuspid regurgitation)

-Age: 16 years boy -Suddenly collapses while jogging and dies despite resuscitation efforts. He has no history of medical problems; however, his family history is significant for an uncle who died suddenly at age 20 -Autopsy reveals significant left ventricular hypertrophy predominantly affecting the interventricular septum. There are no valvular abnormalities. Assuming the boy died of an inherited condition, which of the following proteins was most likely affected? a)beta-myosin heavy chain b)Dystrophin c)Fibrillin-1 d)IK membrane potassium channel e)Transthyretin

Answer: Choice A (beta-myosin heavy chain) -This presentation- sudden cardiac death (SCD) in a young patient with a family history of SCD and autopsy findings of asymmetric septal hypertrophy- is suggestive of hypertrophic cardiomyopathy (HCM). HCM is an autosomal dominant genetic disorder of the cardiac myocytes caused by mutations in one of several sarcomere genes encoding the myocardial contractile proteins. Common mutations include single-point missense mutations in the genes for beta-myosin heavy chain and myosin-binding protein C Choice B= Mutations in the dystrophin gene are responsible for X-linked familial dilated cardiomyopathy. Dystrophin mutations are also found in the common skeletal myopathies (Duchenne and Becker muscular dystrophy), which are both associated with cardiac involvement Choice C= Marfan syndrome is an autosomal dominant defect of the glycoprotein fibrillin-1 in the connective tissue that causes abnormalities in the skeleton (e.g., disproportionately long extremities, pectus deformity, scoliosis), eyes (lens dislocation) and cardiovascular system (e.g., aortic root dilation, aortic regurgitation) Choice D= Genetic mutations in cardiac K+ channel proteins are seen in patients with congenital long QT syndrome (LQTS). A decrease in repolarizing potassium current leads to prolongation of the QT interval, which predisposes to development of polymorphic ventricular tachycardia (torsades de pointes) and sudden cardiac death. However, septal hypertrophy is not a feature of congenital LQTS Choice E= Mutations in the transthyretin gene are responsible for a hereditary form of cardiac amyloidosis. Amyloid deposition in the myocardium results in infiltrative cardiomyopathy, which typically presents with progressive dyspnea, peripheral edema and ascites

-Age: 52 years man -Periodic substernal chest heaviness. The pain is precipitated by fast walking, especially uphill and against the wind, and remits after 5 mins of rest -The patient has history of hypertension, smoker (daily); high BP and normal pulse -Lung are clear to auscultation, and no heart murmurs are heard. There is no peripheral edema. Which of the following pathologic states is most likely to be present in this patient? a)Atherosclerotic plaque obstructing 50% of the coronary artery lumen, no thrombus b)Atherosclerotic plaque obstructing 80% of the coronary artery lumen, no thrombus c)Ruptured atherosclerotic plaque with fully obstructive thrombus d)Significant coronary artery vasospasm causing flow obstruction e)Ulcerated atherosclerotic plaque with partially obstructive thrombus

Answer: Choice B (Atherosclerotic plaque obstructing 80% of the coronary artery lumen, no thrombus) -This patient with multiple risk factors for coronary artery disease (e.g., age, hypertension, smoking) has intermittent substernal chest heaviness consistent with stable angina pectoris. Stable angina is characterized by substernal or left-sided chest pressure, tightness, or pain that is reliably produced by exertion and relieved by rest. It occurs due to temporary myocardial ischemia resulting from demand-supply mismatch of oxygen-rich blood to the myocardium -Myocardial oxygen demand is determined by the heart rate, BP (afterload), left ventricular end-diastolic volume (preload) and cardiac contractility, whereas myocardial oxygen supply is determined by coronary blood flow. The demand supply mismatch of stable angina results from restricted coronary blood flow due to a fixed atherosclerotic lesion that obstructs >70% of the coronary artery lumen. These lesions allow sufficient blood flow to the myocardium during rest but insufficient blood flow to meet myocardial oxygen demands during exertion Choice A= An atherosclerotic plaque obstructing 50% of the lumen is unlikely to produce symptomatic myocardial ischemia. Symptoms of stable angina do not become prominent until approximately 70% of the cross-sectional area of the coronary artery lumen is occluded Choice C= Ruptured atherosclerotic plaque with overlying fully obstructive thrombus describes the pathophysiology of ST elevation MI (STEMI). The abrupt cessation of myocardial blood flow leads to transmural infarction and present with persistent chest pain that is not relieved by rest Choice D= Coronary artery vasospasm (vasospastic angina or Prinzmetal angina) results from hyperactivity of vascular smooth muscle, usually in the absence of significant atherosclerotic coronary artery disease. Transient anginal chest pain is typical, however, the pain not precipitated by exertion or relieved by rest Choice E= Ulcerated atherosclerotic plaque with overlying partially obstructive thrombus is likely to produce acute coronary syndrome in the form of non-ST elevation (partial thickness; NSTEMI) myocardial infarction or unstable angina. Unstable angina describes anginal pain occurring at rest or with markedly increased frequency, intensity, or duration compared to the patient's baseline stable angina

-Age: 42 years man -Comes to the ED due to severe chest pain that started abruptly 2 hours ago. The pain is midline, constant, and 10/10 in intensity. -He has had no fever, cough, or shortness of breath. History: hypertension. On exam: the patient is diaphoretic and is in severe distress due to pain -Temp. (37C), pulse (116), RR (24). SBP (82 in right arm, 60 in left arm); Jugular veins are distended with an estimated pressure of 13cm H2O. The lungs are clear to auscultation -The point of maximal impulse is not palpable. The extremities are cold with no peripheral edema. The patient develops cardiac arrest and dies soon after arrival. Autopsy would reveal? a)Air within the pleural space b)Blood in the pericardial space c)Ruptured posteromedial papillary muscle d)Thrombus at the pulmonary artery bifurcation e)Transmural necrosis of the anterolateral myocardium

Answer: Choice B (Blood in the pericardial space) -This patient's presentation is most consistent with an ascending aortic dissection, complicated by cardiac tamponade. Dissections can affect the ascending aorta (Type A) or may involve only the descending aorta (Type B). The most common symptom is sharp tearing chest and back pain that is severe and sudden. If an ascending dissection extends proximally to the aortic root, it can affect the coronary ostia or aortic valve, resulting in myocardium ischemia or aortic regurgitation, respectively. Other signs and symptoms include BP asymmetry, stroke, or paraplegia, depending on the vessels and structures involved (e.g., subclavian artery, carotid artery, spinal arteries) -In addition, a dissection may extend proximally into the pericardium, causing blood to fill the pericardial space and leading to acute tamponade. As blood accumulates the rising pericardial pressure can rapidly supersede right-sided filling pressures and restrict venous return, resulting in a precipitous drop in cardiac output and obstructive shock that can lead to cardiac arrest. Signs of tamponade include jugular venous distention, muffled heart sounds, and hypotension (Beck triad), as well as reduced point of maximal impulse, cold extremities and clear lungs Choice A= Air in the pleural space is expected with tension pneumothorax, which can cause obstructive shock due to mediastinal shifting with vena cava collapse and impaired venous return to the heart. Although sudden-onset chest pain is common, shortness of breath and unilaterally absent breath sounds are expected Choice C= A ruptured posteromedial papillary muscle causes acute mitral regurgitation, which can abruptly lead to a cardiogenic shock. Patient are expected to have shortness of breath and lung crackles due to acute pulmonary edema Choice D= A saddle pulmonary thromboembolism can cause obstructive shock with RV failure and jugular venous distention. However, it would not account for upper extremity blood pressure asymmetry or distant heart sounds Choice E= Transmural necrosis of the anterolateral myocardium can cause LV free wall rupture with patients developing acute tamponade as blood rapidly fills the pericardium. This etiology of tamponade would not explain upper extremity BP asymmetry

-Age: 35 years man -Brought to the ED after being involved in a motor vehicle collision. He was an unrestrained driver and hit his chest against the steering wheel during the collision -When the paramedics reached him, he reported chest discomfort but was hemodynamically stable. Shortly after arrival at the ED, the patient experiences worsening respiratory distress and becomes hypotensive. -On physical examination, the patient appears anxious and is tachycardic and tachypneic -The trachea is midline; the jugular veins are distended. The anterior chest wall is bruised and tender to palpation. Vesicular breath sounds are present bilaterally. The abdomen is soft and nontender. Cause of patient's clinical deterioration? a)Aortic rupture b)Cardiac tamponade c)Diaphragmatic rupture d)Hemothorax e)Tension pneumothorax

Answer: Choice B (Cardiac tamponade) -This patient experienced significant blunt chest trauma when his chest struck the steering wheel, evidenced by anterior chest wall bruising and tenderness on examination. Such trauma can lead to several life-threatening cardiopulmonary injuries; this patient with progressive hypotension, tachycardia, and jugular venous distention most likely has acute cardiac tamponade -Blunt cardiac injury can cause rapid accumulation of blood within the pericardium. Because the pericardium is relatively noncompliant and does not have time to adapt, even a small amount of pericardial fluid (e.g., 100-200ml) can dramatically increase intrapericardial pressure and lead to tamponade. The increased pericardial pressure compresses the relatively low-pressure, right-sided heart chambers and restricts diastolic filling of the right side of the heart. This elevates right atrial pressure, which is transmitted backward to the vena cava, the central venous system, and the jugular veins (causing distention) -With decreased right-sided filling, less blood is pumped through the pulmonary circulation to the left atrium and left ventricle, resulting in decreased cardiac output. This results in systemic hypotension, despite compensatory tachycardia. Impaired peripheral perfusion leads to unmet oxygen demand in the tissues with resulting dyspnea, increased respiratory drive and tachypnea. Choice A= Blunt deceleration during chest trauma can cause tearing of the aorta (i.e., aortic rupture). However, aortic rupture usually causes massive internal hemorrhage with rapidly developing hypovolemic shock (e.g., flat jugular veins). Most affected patients do not survive transport to the hospital Choice C= Diaphragmatic rupture can occur with blunt trauma, typically from a sudden increase in intraabdominal pressure. It may result in intrathoracic herniation of abdominal organs and respiratory symptoms (e.g., dyspnea, tachypnea), but it often causes abdominal tenderness and does not typically cause significant bleeding or hypotension Choice D= Blunt chest trauma could cause a massive hemothorax with hypotension and tachycardia (i.e., hypovolemic shock) as well as tachypnea due to the mass effect on the lung. However, flat jugular veins and absent basilar breath sounds are expected Choice E= Tension pneumothorax can result from blunt chest trauma that injures the visceral pleura or tracheobronchial tree. It can cause tachypnea and obstructive shock (e.g., hypotension, tachycardia, jugular venous distention); however, unilateral absent breath sounds are expected, and tracheal deviation is often present.

-Age: 18 years woman -Referred to cardiologist after a heart murmur is discovered during a routine checkup -Patient is healthy and has no symptoms. Medical history is unremarkable -She runs daily and wants to start actively training for a half marathon. The patient is concerned that the murmur is a sign of heart disease -She has no family history of sudden cardiac death. Auscultation reveals a midsystolic click followed by a short late-systolic murmur at the cardiac apex. The murmur disappears with squatting. This patient's condition is most likely related to an abnormality in which of the following tissue? a)Cardiac muscle b)Connective tissue c)Coronary vessels d)Endocardial lining e)Parietal pericardium

Answer: Choice B (Connective tissue) -The presence of a midsystolic click followed by a systolic murmur at the cardiac apex that disappears with squatting is most consistent with mitral valve prolapse (MVP). The click is caused by sudden tensing of the chordae tendineae as they are pulled taut by the ballooning valve leaflets, and the murmur results from regurgitation of blood due to malalignment of the valve margins with prolapse -During systole, there is a critical LV size at which prolapse occurs, therefore, maneuvers that change LV end-diastolic volume affect the timing of the click and murmur. Squatting increases venous return and LV volume, increasing the tension and decreasing the slack in the mitral valve apparatus. Because ethe ventricular walls have farther to travel to reach the critical LV size at which prolapse occurs, prolapse occurs later in systole or not at all -Primary MVP is most commonly a sporadic disorder and is characterized by myxomatous degeneration (i.e., pathologic deterioration of the connective tissue) affecting the mitral valve leaflets and chordae tendineae. Secondary MVP is associated with inherited connective tissue disorders, including Marfan or Ehlers-Danlos syndrome and osteogenesis imperfecta Choice A= Hypertrophic cardiomyopathy (HCM) presents with a systolic murmur at the left sternal border that decreases in intensity with squatting (due to increased LV volume and decreased outflow tract obstruction). A midsystolic click is not heard with HCM Choice C= Coronary artery disease can cause myocardial ischemia or infarction leading to papillary muscle dysfunction and acute mitral regurgitation. In such patients, squatting from a standing position increases murmur intensity due to a higher blood volume of regurgitation. In addition, such a murmur is not preceded by a midsystolic click Choice D= Endomyocardial fibrosis and endocardial fibroelastosis are disorders that affect the endocardium and are not associated with MVP Choice E= Pericardial disease may infrequently result in cardiac tamponade or constrictive pericarditis. Pericarditis may produce a friction rub, however, it is not associated with a cardiac murmur

-Age: 32 years man -Dies suddenly in his sleep. Easy fatigability (past 6 months), tired all the time, and needed to take naps during the day. He didn't drink or use drugs. Smoker (daily- 10 years). His father died from a heart problem at age 40 -Autopsy= heart grossly enlarged. Layered mural thrombus is seen in the left ventricular apex -Coronary atherosclerosis is present, with 20% narrowing of the mid-left anterior descending artery and 25% narrowing of the left circumflex artery. Cause of death? a)Antiphospholipid antibody syndrome b)Dilated cardiomyopathy c)Hypertrophic cardiomyopathy d)Ischemic heart disease e)Restrictive cardiomyopathy

Answer: Choice B (Dilated cardiomyopathy) -This patient most likely died due to dilated cardiomyopathy (DCM). A primary insult to the myocardium leads to impaired myocardial contractile function with dilation of one or both ventricles. The structural changes to the ventricular myocardium places these patients at risk of sudden cardiac death from ventricular arrhythmia (i.e., ventricular tachycardia, ventricular fibrillation). In addition, the global hypokinesis of the LV leads to stagnation of blood flow and the possible development of LV mural thrombus and subsequent systemic embolization -Causes of DCM include infection (e.g., viral myocarditis, Chagas disease), chemical toxicity (e.g., alcohol), and inherited disease. The premature death of this patient's father from a heart problem suggests familial DCM, which is typically transmitted in an autosomal dominant inheritance pattern. Truncating mutations affecting the TTN gene, which encodes for the cardiac sarcomere protein titin, are the most common cause of familial DCM. Choice A= Antiphospholipid antibody syndrome is an autoimmune disorder characterized by venous and/or arterial thrombosis. Cardiac manifestations can include valvular thickening, nonbacterial vegetations, and increased risk of coronary atherosclerosis; however, LV cavity enlargement and mural thrombus are not typical Choice C= Hypertrophic cardiomyopathy is an autosomal dominant genetic disorder caused by mutation in genes encoding for one of several cardiac sarcomere proteins (e.g., beta-myosin heavy chain, myosin-binding protein C). It is a common cause of SCD in young individuals, usually during exertion. Autopsy typically shows asymmetric septal hypertrophy of the LV Choice D= Ischemic heart disease can lead to myocardial dysfunction and LV enlargement that mimics DCM; however, this patient's coronary arteries had only minor atherosclerotic narrowing (i.e., 20% and 25%) that would not have caused clinically significant myocardial ischemia Choice E= Restrictive cardiomyopathy is typically the early cardiac manifestations of an infiltrative process (e.g., amyloidosis, sarcoidosis) and is characterized by nondilated ventricles with enlargement of both atria

-Age: 33 years woman -Comes to the ED due to sudden-onset shortness of breath and left-sided chest pain -The symptoms began an hour ago while she was doing yard work, and she nearly passed out at symptom onset; takes no medication/supplements other than ibuprofen for occasional headaches. She doesn't smoke -Temp. (37.1C), BP (84/58), pulse (122), RR (24). Pulse oximetry is 86% on room air. On physical exam, the patient is in moderate distress. The jugular veins are distended. -Heart sounds are rapid and regular with no murmurs. Lungs are clear to auscultation bilaterally with no crackles or wheezes. Arterial blood gas results are pH 7.52, PaCO2 28mmHg, and PaO2 54mmHg. Most likely to be seen on beside echocardiography? a)Enlarged left ventricular cavity b)Enlarged right ventricular cavity c)Left atrial enlargement d)Left ventricular wall thickening e)Right ventricular wall thickening

Answer: Choice B (Enlarged right ventricular cavity) -This patient most likely has an acute pulmonary embolism (PE). The classic presentation is acute-onset chest pain and shortness of breath that may be accompanied by syncope or near-syncope. -Patients usually have tachycardia, tachypnea, jugular venous distention, and clear lungs. Arterial blood gas typically show hypoxemia and acute respiratory alkalosis due to hyperventilation. Up to 30% of cases may present with no apparent risk factors (e.g., hypercoagulability) -Massive PE can lead to hypotension and obstructive shock. There is a rapid increase in pulmonary arterial resistance that leads to an increase in pulmonary arterial and RV pressure. The rapid pressure increase causes RV cavity enlargement due to increased RV wall tension and cardiac muscle stretching. RV myocardial oxygen demand increases and coronary artery perfusion decreases, leading to a supply/demand mismatch and RV ischemia. Consequent RV dysfunction then leads to an inability to pump blood through the pulmonary circulation, resulting in decreased left-sided preload and decreased cardiac output. Such RV failure is caused by an increase in pulmonary vascular resistance is sometimes called cor pulmonale Choice A= An enlarged LV cavity usually results from eccentric hypertrophy in the setting of chronic volume overload, such as occurs in dilated cardiomyopathy or severe aortic or mitral valve regurgitation. In acute PE, the LV cavity is either normal or somewhat reduced in size due to reduced blood flow coming from the right side of the heart Choice C= Left atrial enlargement is typical of severe mitral stenosis, which usually occurs in the setting of rheumatic heart disease. Undiagnosed mitral stenosis can sometimes present acutely with decompensated heart failure; however, a rumbling diastolic murmur and evidence of pulmonary edema (e.g., crackles on lung auscultation) are expected Choice D= LV wall thickening is expected with the concentric hypertrophy that occurs with prolonged systemic hypertension or severe aortic stenosis Choice E= RV wall thickening is seen with the concentric hypertrophy that occurs in patients with chronic pulmonary hypertension. In acute PE, there is no time for compensatory wall thickening to occur in response to the increased pressure load, the rapid increase in pressure causes RV dilation that may be accompanied by RV failure

Question 2 of 2 -Blood cultures obtained from this patient on admission grow Streptococcus species. Which of the following processes was the most likely initiating step in the pathogenesis of this patient's condition? a)Endocardial fibrosis b)Fibrin deposition c)Liquefactive necrosis d)Myxomatous degeneration e)White blood cell infiltration

Answer: Choice B (Fibrin deposition) -The initial process involved in the pathogenesis of IE is a disruption of normal endocardial surface. This occurs most commonly at the areas of maximal turbulence to blood flow in preexisting valvular lesions, typically the atrial surface of the incompetent AV valves, or the ventricular surface of incompetent semilunar valves. This sis followed by focal adherence of fibrin and platelets, forming a sterile fibrin-platelet nidus -During bacteremia from any cause, microorganisms colonize the sterile nidus on the endothelial surface with subsequent microbial growth leading to further activation of the coagulation system. Streptococci infect the cardiac valves with preexisting endothelial lesions. In contrast, Staphylococcus aureus can adhere to damaged or normal endothelial cells. Macroscopic vegetations consist of fibrin and platelets on the surface, with RBC debris, leukocytes, and clusters of microorganisms embedded deep within the lesion Choice A= Endomyocardial fibrosis is a restrictive cardiomyopathy characterized by thickening and fibrosis of the apical endocardial surface. It occurs most commonly in tropical regions (e.g., Uganda) Choices C and E= Neutrophil-rich inflammatory infiltrates with subsequent abscess formation and/or liquefactive necrosis of underlying cardiac tissue are common in advanced lesions Choice D= Myxomatous degeneration of the mitral valve is characterized by thickened and redundant mitral leaflets with elongated chordae leading to prolapse of one or both mitral leaflets (mitral valve prolapse). Myxomatous degeneration can predispose patients to the development of endocarditis; however, endocarditis vegetations themselves are not formed through myxomatous degeneration

-Age: 66 years woman -Worsening chest discomfort and shortness of breath for the past 4 weeks. There is no history of recent upper respiratory illness, fever, cough, palpitations, or syncope -History: hypertension, breast adenocarcinoma (treated with surgery and chemotherapy 6 years ago). The patient doesn't smoke, and hasn't traveled recently -Temp. (37C), BP (110/70), pulse (90). Physical exam: changes consistent with prior right mastectomy. The right axillary lymph nodes are enlarged. -The lungs are clear to auscultation. Heart sounds are distant with no murmur. There is no extremity edema. Bedside echocardiography= large pericardial effusion. Pericardiocentesis in this patient will show? a)Exudative fluid containing acid-fast bacilli b)Hemorrhagic fluid with atypical cells c)Purulent fluid with Gram-positive cocci d)Serosanguinous fluid positive for viral PCR e)Turbid fluid containing fungal elements

Answer: Choice B (Hemorrhagic fluid with atypical cells) -This patient with a history of right breast cancer, right axillary lymph node enlargement (i.e., likely metastatic disease) and a large pericardial effusion (e.g., distal heart sounds) most likely has malignant pericardial effusion -Malignancy (e.g., breast cancer) is a common cause of pericardial effusion and may be the initial manifestation of cancer recurrence. Pericardial involvement occurs by either direct tumor extension or metastatic spread through the blood or lymphatics; this often results in subacute accumulation (e.g., over weeks) of pericardial fluid. Subacute (vs. rapid) accumulation gives the pericardium time to progressively stretch, keeping intrapericardial pressure stable; during this period, patients often have only nonspecific symptoms (e.g., dyspnea, chest discomfort). However, large volumes can eventually exceed the pericardium's stretch capacity, increase intrapericardial pressure, and compromise cardiac function -When the cause of pericardial effusion is not clear, pericardiocentesis can be used to acquire fluid for laboratory analysis to help determine etiology. In malignant effusion, pericardial fluid is often hemorrhagic due to inflammation associated with metastatic invasion that causes bleeding from irritated capillaries. Cytologic analysis often reveals atypical malignant cells Choice A= Some patients with pulmonary TB develop tuberculous pericarditis, which can be accompanied by pericardial effusion containing fast-acid bacilli. However, patients with TB pericarditis typically have symptoms of extrapulmonary TB, including fever, night sweats, and weight loss. In addition, most patients are from, or have traveled to, areas where TB is endemic. Choices C and E= Bacteria (e.g., S aureus) and fungi can cause purulent pericarditis, which may be accompanied by pericardial effusion containing gross pus (i.e., purulent fluid) or microscopic purulence (i.e., turbid fluid). However, most patients with purulent pericarditis are acutely ill (e.g., fever, tachycardia) and have predisposing risk factors such as recent or ongoing infection, immunosuppression, or thoracic surgery Choice D= Viral infection is a common cause of pericarditis, which can result in pericardial effusion. However, viral pericarditis is often preceded by viral respiratory or GI symptoms. In this patient with evidence of metastatic breast cancer (e.g., enlarged axillary lymph nodes) and no recent illness, malignant effusion is more likely

-Age: 16 years boy -Evaluated in the office due to leg cramps. He plays soccer but frequently stops in the middle of practice due to cramps in both legs. The patient is afebrile -BP (170/90) bilaterally in the upper extremities. Point of maximal impulse is nondisplaced. S1 is normal, S2 is physiologically split. An S4 is heard, as well as continuous murmur best auscultated over the back at the left paravertebral area. Distal pulses in the radial arteries are prominent. Dorsalis pedis and posterior tibial pulses are difficult to palpate. The rest of the exam is unremarkable -Which of the following is the most likely cause of this patient's continuous murmur? a)Increased blood flow across the tricuspid valve b)Increased blood flow through collateral blood vessels c)Increased cardiac output due to low systemic vascular resistance d)Increased pulmonary arterial blood flow e)Persistence of the ductus arteriosus

Answer: Choice B (Increased blood flow through collateral blood vessels) -This patient has upper extremity hypertension with diminished lower extremity pulses. These findings are consistent with coarctation of the aorta, a congenital heart defect characterized by narrowing of the aorta -Patients with severe aortic narrowing develop cardiogenic shock in the neonatal period; children and adults with more mild aortic narrowing may have asymptomatic hypertension and bounding pulses of the upper extremities with hypotension and diminished pulses in the lower extremities. Decreased distal perfusion can also lead to lower extremity claudication, as seen in this patient. -To provide blood flow beyond the site of the coarctation, a compensatory network of collaterals often develops gradually. For instance, distal branches of the ascending aorta (e.g., internal thoracic artery) can supply blood to the intercostal arteries below the obstruction, blood can then flow retrograde through the intercostal arteries into the descending aorta to bypass the defect. Flow through collateral blood vessels accounts for the continuous murmur auscultated over the back in this patient. LVH also can develop due to the aortic obstruction, resulting in an S4 gallop as seen in this case Choices A and D= An atrial septal defect is characterized by a left-to-right intracardiac shunt and right-sided volume overload, causing increased blood flow through the tricuspid valve and pulmonary arteries. Cardiac examination reveals a fixed and widely split S2 and a systolic ejection murmur best heard over the second left intercostal space (i.e., pulmonic flow murmur) Choice C= Increased cardiac output due to low systemic vascular resistance can occur during septic shock. In contrast, aortic coarctation causes increased systemic vascular resistance due to obstructed aortic blood flow Choice E= Patent ductus arteriosus causes a continuous murmur best heard over the left infraclavicular area. However, differential hypertension and diminished pulses in the lower extremities would not be seen.

-Age: 32 years man -Dies suddenly in his sleep. Autopsy= left ventricle with a large area of apical thinning composed of fibrotic scar tissue. The mid and basal left ventricular segments show normal myocardial thickness -The coronary arteries show no evidence of obstructive atherosclerosis. The right ventricle appears normal in size. No apparent valve damage is present. -Dilation and wall-thickening of the esophagus is also noted. Which of the following factors in this patient's medical history would be most helpful in determining the cause of death? a)Alcohol consumption b)Location of residence c)Childhood vaccinations d)Family history of cardiomyopathy e)Occupational chemical exposure

Answer: Choice B (Location of residence) -This man with a large area of localized cardiac apical wall thinning likely died of chronic Chagas cardiomyopathy. Chagas disease is caused by infection with the protozoan Trypanosoma cruzi, which is transmitted by the triatomine "kissing" bug (Reduviid family) found throughout the Americas as far north as the Southern US. Infection is rare in developed regions, but the disease is endemic throughout Central and South America where widespread open-air and thatched-roof housing exposes individuals to routine contact with the triatomine vector -Years following initial infection, some individuals develop serious end-organ damage affecting the cardiac, GI and/or CNS. Cardiac disease results from a chronic parasite-induced and immune-mediated myocarditis that leads to dilated cardiomyopathy (DCM). Chronic Chagas cardiomyopathy is relatively unique among DCMs in that there is characteristically localized apical wall thinning with the development of a large apical aneurysm. In addition to impairing ventricular systolic function, the aneurysm can harbor mural thrombus that may systemically embolize and cause stroke. Damage to the cardiac conduction system can also trigger ventricular arrhythmias (e.g., ventricular tachycardia, ventricular fibrillation) that can lead to sudden cardiac death (the likely cause of death in this patient) -The GI manifestations of Chagas disease result from destruction of the myenteric plexus, which can lead to progressive disruption and dysfunction of the esophagus, and less commonly, the colon Choices A, C, D and E= Excessive alcohol consumption can lead to DCM. Measles, mumps and rubella are typically vaccinated against in childhood and are potential causes of viral DCM. Genetic mutations of the gene encoding for the cardiac sarcomere titin can lead to familial DCM. Miners and metal workers who are exposed to high levels of cobalt can develop DCM. These DCMs typically involve diffuse LV wall thinning, unlike the characteristic localized apical wall thinning and aneurysms development that occurs in chronic Chagas cardiomyopathy. DCM due to IHD may sometimes involve localized wall thinning and aneurysm development, but it is unlikely in this young man with no evidence of atherosclerotic coronary artery disease. Esophageal enlargement is also not expected with these other etiologies of DCM

-Age: 10 years boy -Recently immigrated from overseas with his family is brought to the office due to exertional dyspnea and fatigability. The boy tires easily when walking and cannot keep up with his peers at the playground. According to his parents, he was diagnosed with congenital heart disease in infancy, but corrective surgery was unavailable. -They cannot recall the details of his diagnosis. He takes a daily multivitamin and no medications. Pulse oximetry of the right hand shows 99% on room air. Physical examination reveals no murmurs. There is bilateral cyanosis and clubbing of the toes. The fingers are normal. All extremity pulses are full and equal. Diagnosis? a)Atrial septal defect b)Patent ductus arteriosus c)Tetralogy of Fallot d)Transposition of the great arteries e)Ventricular septal defect

Answer: Choice B (Patent ductus arteriosus) -The patient with cyanosis and clubbing of the feet but normal peripheral pulses most likely has a large patent ductus arteriosus (PDA). A PDA initially results in a left-to-right shunt with increased pulmonary blood flow. Over time, this can lead to increased pulmonary vascular resistance (PVR) due to progressive vascular remodeling. If the shunt is not corrected, PVR can eventually exceed systemic resistance and cause reversal of the shunt to right-to-left blood flow (i.e., Eisenmenger syndrome), resulting in arterial hypoxemia, cyanosis, and eventually clubbing -Differential clubbing and cyanosis in the lower extremities occurs when there is a significant decrease in postductal oxygen saturation (the arteries perfusing the head and upper extremities arise proximal to the ductus). In patients with a PDA and Eisenmenger syndrome, the upper extremities are supplied by oxygenated blood from the left ventricle, but oxygenated blood flows right-to-left across the PDA and supplies the descending aorta, resulting in lower body hypoxemia and cyanosis of the toes. -The right-to-left shunting in Eisenmenger syndrome does not typically cause a murmur, but some patients may develop a murmur of tricuspid regurgitation due to progressive right ventricular enlargement Choices A and E= Atrial and ventricular septal defects initially involve left-to-right intracardiac shunting and are therefore acyanotic at birth. Large septal defects are at risk for shunt reversal and Eisenmenger syndrome; however, since the right-to-left shunt is intracardiac, cyanosis and clubbing affect the upper and lower extremities equally Choice C= Tetralogy of Fallot is generally characterized by spells of cyanosis due to transient right-to-left shunting across the ventricular septal defect. Because the right-to-left shunt is intracardiac, cyanosis typically involves the whole body Choice D= Differential clubbing and cyanosis can occur with uncorrected transposition of the great arteries when there is extracardiac shunting through a PDA to allow mixing between the 2 parallel circuits. However, the cyanosis will be present in the upper extremities and not lower extremities, as oxygenated blood will flow across the PDA into the descending aorta (i.e., postductal oxygen saturation is higher)

-Age: 42 years woman -Comes to ED for evaluation of chest pain. She was moving furniture in her summer house 2 days ago when she experienced sharp pain in the left side of the sternum that quickly subsided. Since then, the patient has had episodic pain with deep inspiration or trunk movement. She has no fever or cough. -History: hypertension; father died of MI at age 67. Not smoker. High BP (same BP in right and left arm), pulse and RR (normal) -Localized tenderness to palpation at the left sternal border. Lungs are clear to auscultation, and cardiac exam (normal hear sounds without gallops or murmurs). Abdomen is soft and nontender. No peripheral edema. Cause? a)Acute pericarditis b)Aortic dissection c)Costochondritis d)Esophageal spasm e)Gastroesophageal reflux disease f)Panic disorder g)Pneumonia h)Pulmonary arterial hypertension i)Pulmonary embolism j)Unstable angina

Answer: Choice C (Costochondritis) -This patient's chest pain is most likely due to Costosternal syndrome (also known as costochondritis or anterior chest wall syndrome) involving regional chest wall. It usually occurs after repetitive activity and involves the upper costal cartilage at the costochondral or Costosternal junctions. The pain is typically reproduced with palpation and worsened with movement or changes in position (e.g., horizontal arm flexion). Patients typically do not have palpable warmth, swelling or erythema Choices A, G and I= Conditions causing inflammation of the pleura or pericardium can cause sharp pain worsened with inspiration. Pericarditis often follows an upper respiratory viral syndrome. The pain of pericarditis is typically worse when lying flat and relieved by leaning forward. Examination may show a pericardial friction rub. Pulmonary embolism can cause pleural inflammation with chest pain. However, such cases are typically due to large emboli and are associated with significant hypoxia and shortness of breath. Pneumonia usually presents with fever, cough, and shortness of breath. None of these conditions would be associated with focal tenderness of the chest wall Choice B= Aortic dissection causes sudden, tearing chest pain radiating to the back and BP disparity between the arms (>10mmHg). This patient's episodic pain makes this less likely. BP will normally vary mildly between arms; a difference in systolic or diastolic BP <10mmHg is not considered significant Choice D and E= GERD typically causes postprandial or nocturnal burning pain in the upper abdomen and chest. Esophageal spasm can cause chest pain with meals or swallowing Choice F= Panic disorder is usually a diagnosis of exclusion and can present with episodic chest pain, shortness of breath, and palpitations. Patients usually have intense fear/anxiety, and chest wall tenderness is not seen Choice H= Pulmonary arterial hypertension usually presents with exertional dyspnea, lethargy and fatigue. Other symptoms, such as exertional chest pain, syncope, and peripheral edema, are generally seen in advanced disease Choice J= Unstable angina is characterized by exertional chest pressure, often radiating to the arm or jaw. Local chest tenderness does not rule out a cardiac cause of chest pain but is unusual

-Age: 27 years man -Comes to the ED due to acute-onset chest pain and shortness of breath. The pain started suddenly an hour ago while he was lifting a box of books. The patient has no chronic medical conditions, and he was in the usual state of health prior to the onset of symptoms. -He doesn't smoke. Height is 182cm and weight is 72kg. On physical exam, the patient is in acute respiratory distress. Temp. (37C), BP (90/60), pulse (120, regular), and RR (26). Pulse oximetry shows 86% saturation on room air. -Cardiopulmonary examination reveals pectus excavatum, bilateral pulmonary crackles, and a decrescendo diastolic murmur at the right upper sternal border. The abdomen is soft and nontender. There is no peripheral edema. Subsequently, the patient develops cardiac arrest and dies. Histology changes on autopsy? a)Atherosclerotic thickening of the intima b)Coagulative necrosis of the left ventricular wall c)Fragmentation of the elastic lamellae in the aortic media d)Necrosis of the papillary muscles e)Perivascular inflammation around the vasa vasorum

Answer: Choice C (Fragmentation of the elastic lamellae in the aortic media) -This young patient likely had Marfan syndrome complicated by aortic root disease, leading to aortic dissection with acute aortic regurgitation and heart failure. Aortic dissection typically presents with sudden-onset, severe chest or back pain. In Marfan syndrome, the dissection almost always affects the ascending aorta; it can propagate proximally to impair aortic valve closure, causing acute aortic regurgitation (evidenced by a decrescendo diastolic murmur) and heart failure (evidenced by pulmonary edema) -Marfan syndrome results from a mutation that disrupts the synthesis, secretion, and incorporation into the extracellular matrix of fibrillin, a protein that provides the glycoprotein scaffolding for elastin structure. Although affected patients are often tall in stature, the syndrome is more reliably recognized by an increased ratio of limb length to trunk length -The most common histologic findings in Marfan-related aortic root disease include fragmentation and loss of the elastic lamellae with fibrosis and cystic medial degeneration (replacement of collagen, elastin, and smooth muscle by a basophilic mucoid extracellular matrix with irregular fiber cross-linkages and cystic collections of mucopolysaccharide). Cystic medial degeneration also occurs with normal aging but is accelerated in Marfan syndrome. Choices A and B= Fatty streaking (the early stages of atherosclerosis) is often present in male patients in their 20s, but significant atherosclerotic thickening of the intima is not expected. Coagulative necrosis of the LV wall typically results from atherosclerotic plaque rupture leading to MI, which is unlikely in the absence of significant atherosclerotic disease Choice D= MI can cause papillary muscle necrosis and rupture. The mechanical complication typically occurs acutely or within 3-5 days of MI and leads to acute mitral regurgitation with a holosystolic-rather than diastolic decrescendo-murmur Choice E= Inflammation of the vasa vasorum can lead to aortitis and aneurysm formation in syphilis and certain vasculitides (e.g., Behcet disease). However, such diseases are associated with multiple other manifestations (e.g., rash, ulcer, airway disease) and would not explain this patient's pectus excavatum

-Age: 64 years man -Difficult walking for 6 months; experiencing muscle cramps in his right thigh after walking a block on level ground -The cramps subside quickly with rest. He has also had decreased sexual performance. -Medical history: hypercholesteremia and MI; smokes (daily) -Afebrile, normal BP and pulse; cardiac auscultation (normal S1 and S2 and equal vesicular breath sounds over bilateral lungs) -Abdomen soft (without masses), no peripheral edema, nor muscle tenderness, nor muscle weakness; cause of patient's symptoms? a)Granulomatous inflammation of the arterial media b)Homogenous acellular thickening of the arteriolar wall c)Lipid-filled arterial intimal plaques d)Medial band-like arterial calcification e)Onion-like concentric thickening of the arteriolar walls

Answer: Choice C (Lipid-filled arterial intimal plaques) -Patient's muscle pain (occurs with exercise and remits with rest)= intermittent claudication; smoking is an important risk factor -Claudication is usually due to atherosclerosis of the large arteries, specifically resulting from fixed stenoses caused by lipid-filled intimal plaques that bulge into arterial lumen (atheromas). These stenoses prevent a sufficient increase in blood flow to muscles during exercise, leading to ischemic muscle pain. The pain is rapidly relieved by rest, as residual blood flow is adequate to meet the metabolic demands of resting, but not exercising muscle -Thigh claudication is suggestive of occlusive disease of the ipsilateral external iliac artery or its more distal branches (i.e., common femoral, superficial femoral, profunda femoris arteries). Accompanying impotence and/or gluteal claudication suggests more proximal aortoiliac occlusion (so called Leriche syndrome), which, in addition to affecting the external iliac artery, also diminishes blood flow to the internal pudendal and gluteal branches of the internal iliac artery Choice A= Granulomatous inflammation of the arterial media occurs in temporal (giant cell) arteritis, the most common form of systemic vasculitis in adults. The condition typically affects patients age >50 years and causes headache, facial pain, jaw claudication and visual loss Choice B= Hyaline arteriolosclerosis involves homogenous deposition of hyaline material in the intima and media of small arteries and arterioles, typically in patients with hypertension or diabetes mellitus (both absent in this patient). The underlying structure of the vessel wall is maintained, but the intima is thickened, and the arterial lumen narrowed Choice D= Medial band-like calcifications occur in medial calcific sclerosis (Monckeberg arteriosclerosis). The condition is characterized by calcified deposits in muscular arteries and typically affects individuals age >50. Although visible radiologically and often palpable on physical examination, these calcifications are clinically asymptomatic because they do not narrow the vessel lumen Choice E= Onion-like concentric thickening of arteriolar walls due to layers of smooth muscle cells and reduplicated basement membrane is seen in hyperplastic arteriolosclerosis, which can occur in severe chronic hypertension (e.g., BP >180/120mmHg). Although arterioles in all tissues may be affected, those in the kidneys, retinas, and intestines are most common; skeletal muscle is less likely to be symptomatically involved

-Age: 44 years man -Occasional chest discomfort that is not consistently related to exertion. History: hypertension, hyperlipidemia. Grandfather experienced MI at age 50. -Coronary CT angiography= several nonobstructive atherosclerotic plaques in the coronary arteries. One plaque in the proximal left anterior descending artery appears extensive, has a large hypodense core, and occupies 40% of the lumen. No intervention is performed -One year later, the patient comes to the ED with acute severe chest pain and is found to have thrombotic occlusion of the proximal left anterior descending artery. High intraplaque activity of which of the following enzymes most likely resulted in this patient's MI? a)Hydroxymethylglutaryl CoA reductases b)Lysyl oxidase c)Metalloproteinases d)Procollagen peptidases e)Prolyl hydroxylase

Answer: Choice C (Metalloproteinases) -This patient with coronary atherosclerosis has likely developed plaque rupture with thrombotic occlusion of the left anterior descending artery, resulting in acute coronary syndrome (unstable angina, MI) -Patients with coronary atherosclerosis can be asymptomatic but typically develop symptoms of stable angina (exertional chest pain relieved nitroglycerin/rest) if progressive enlargement of the atherosclerotic plaque causes >70% luminal stenosis. -Acute coronary syndrome usually occurs due to plaque rupture, which leads to superimposed thrombosis and vessel occlusion. The likelihood of plaque rupture is typically related to plaque stability rather than plaque size or degree of luminal narrowing. Plaque stability largely depends on the mechanical strength of the fibrous cap. Thin-cap fibroatheromas are generally unstable and more vulnerable to rupture. -During the chronic inflammatory progression of an atheroma, the fibrous cap is continually being remodeled. The balance of collagen synthesis and degradation determines the mechanical strength of the cap. Thin-cap fibroatheromas are characterized by a large necrotic core covered by a thin fibrous cap. Activated macrophages infiltrating the atheroma contribute to the breakdown of extracellular matrix proteins (e.g., collagen) by secreting metalloproteinases. Ongoing intimal inflammation can destabilize the mechanical integrity of the plaque through release of these metalloproteinases, resulting in plaque rupture and consequent acute coronary syndrome Choice A= HMG-CoA reductase is the rate-limiting enzyme in cholesterol synthesis and is mostly expressed in the liver, intestine, adrenal cortex, and reproductive tissues. Patients with hyperlipidemia are typically treated with stating (e.g., atorvastatin), which competitively inhibit HMG-CoA reductase Choice B= Lysyl oxidase strengthens extracellular collagen fibers by mediating cross-link formation between lysine and hydroxylysine residues (requires copper). Therefore, a high activity of lysyl oxidase would favor atheroma stabilization by strengthening collagen in the fibrous cap Choice D= Procollagen peptidase cleaves the terminal regions of procollagen molecules exocytosed by fibroblasts or smooth muscle cells, transforming procollagen into insoluble tropocollagen. Tropocollagen then aggregates to form collagen fibrils Choice E= Prolyl hydroxylase is responsible for the hydroxylation of proline on procollagen chains (requires vitamin C), which is a necessary step in the formation of a stable collagen triple helix

-Age: 54 years Caucasian male -Comes to the ED with retrosternal chests pain of 30 mins duration. The patient also complains of sweating and mild dyspnea. -A single tablet of nitroglycerin is delivered sublingually, and the patient's pain decreases significantly. The patient has experienced several similar episodes of pain over the last 12 hours, all of which resolved spontaneously. -Which of the following ultrastructural changes would most likely indicate irreversible myocardial cell injury in this patient? a)Myofibril relaxation b)Disaggregation of polysomes c)Mitochondrial vacuolization d)Disaggregation of nuclear granules e)Triglyceride droplet accumulation f)Glycogen loss

Answer: Choice C (Mitochondrial vacuolization) -The appearance of vacuoles and phospholipid-containing amorphous densities within mitochondria generally signifies irreversible injury and implies a permanent inability to generate further ATP via oxidative phosphorylation. When the mitochondria are injured irreversibly, the cell cannot recover. Simple mitochondrial swelling may be associated with reversible cellular injury, however. Choice A= Myofibril relaxation is an early sign of reversible injury in cardiac myocytes, which occurs within the first 30 mins of severe ischemia. Myofibril relaxation corresponds with intracellular ATP-depletion and lactate accumulation due to anaerobic glycolysis during this period. Choice B= Disaggregation of polysomes denotes the dissociation of rRNA from mRNA in reversible ischemic/hypoxic injury. Depletion of intracellular ATP is thought to promote the dissolution of polysomes into monosomes as well as the detachment of ribosomes from the rough endoplasmic reticulum. Disaggregation of polysomes results in impaired protein synthesis. Choice D= Disaggregation of granular and fibrillar elements and fibrillar elements of the nucleus is associated with reversible cell injury. Another common nuclear change associated with reversible cell injury is clumping of nuclear chromatin, perhaps secondary to a decrease in intracellular pH Choice E= Triglyceride droplet accumulation is characteristic of reversible cell injury, especially in hepatocytes, and also in striated muscle cells and renal cells. This fatty change may result from the decreased synthesis of intracellular proteins that occurs with cell injury. In the hepatocyte, decreased production of lipid acceptor proteins prevents the normal incorporation of triglycerides into lipoproteins. Since triglycerides cannot be rapidly exported from the cell in the form of lipoproteins, they accumulate intracellularly Choice F= Glycogen loss is another early and reversible cellular response to injury. As a result of lowered mitochondrial ATP production, ATP must be supplied to the cell via anaerobic glycolysis of glucose derived from the cell's glycogen stores. Myocardial glycogen stores may be completely depleted within 30 mins of the onset of severe ischemia

-Age: 65 years man -Admitted to hospital with an acute ST-segment elevation MI. The patient undergoes coronary angiography with stent placement in the left circumflex artery, and he is started on therapy with a beta blocker and antiplatelet agents. -On day 3 of hospitalization, he experiences chest pain. The pain, described as sharp and radiating to his neck and shoulders, is exacerbated by coughing and swallowing -He has no shortness of breath, lightheadedness, or abdominal pain. Temp (37.5), BP (130/80) pulse (90) and regular, RR (20). The lungs are clear on auscultation, no peripheral edema -Cause of patient's chest pain? a)Pericardial inflammation due to autoimmune reaction to necrotic tissue b)Pericardial inflammation due to concomitant viral infection c)Pericardial inflammation overlying the necrotic segment of myocardium d)Recurrent thrombosis of the culprit coronary vessel e)Thrombosis of a nonculprit coronary vessel

Answer: Choice C (Pericardial inflammation overlying the necrotic segment of myocardium) -The sharp and pleuritic nature of this patient's new chest pain suggests pericardial involvement. The exacerbation with swallowing indicates that the posterior pericardium may be involved, and the radiation to the neck suggest involvement of the inferior pericardium, which is adjacent to the phrenic nerve afferents supplying the diaphragm. The patient's low-grade fever indicates that this is an inflammatory process -A fibrinous or serofibrinous pericarditis, known as peri-infarction pericarditis (PIP), develops in about 10-20% of patients between 2-4 days following a transmural MI. This pericarditis is a reaction to necrosis of the myocardium near the epicardial surface, delayed treatment presentation increases the risk of PIP due to higher levels of necrosis. Inflammation of the visceral and parietal pericardium is usually localized to the areas overlying the necrotic myocardial segment. The condition is generally short-lived and resolves with several days of supportive care; sometimes therapy with high-dose aspirin is needed Choice A= Dressler syndrome (postcardiac injury syndrome) is an autoimmune-mediated pericarditis that is likely provoked by antigens exposed or created by infarction and necrosis of the cardiac muscle. It is less common and has a later onset than PIP, typically occurring a week to a few months after an MI. The pericardium is often diffusely affected in Dressler syndrome Choice B= Viral pericarditis usually follows an antecedent upper respiratory infection. However, a viral cause for this patient's pericarditis is less likely than PIP as a direct complication of his transmural infarction Choices D and E= Recurrent thrombosis of the recently stented vessel or thrombosis of another coronary vessel would be expected to cause similar pain to the patient's original, anginal type of pain. The pain of myocardial ischemia is not typically sharp or pleuritic, but rather constant, substernal and crushing

-Age: 6 years boy -Difficulty hearing. The patient has no ear pain, discharge, or upper respiratory symptoms. Initial testing suggests that he has bilateral sensorineural hearing loss. He has no motor deficits or cerebellar signs. His paternal uncle died suddenly at age 12 -Examination of the ears, nose, and throat is normal. ECG shows normal sinus rhythm with a prolonged QT interval (520msec). Echocardiogram shows normal left and right ventricular function with no significant valvular disease. -A genetic defect affecting which of the following is most likely present in this patient? a)Calcium channels b)Membrane anchoring protein c)Potassium channels d)Sodium channels e)Sodium-potassium ATPase

Answer: Choice C (Potassium channels) -This patient's clinical presentation is suggestive of Jervell and Lange-Nielsen syndrome, an autosomal recessive disorder characterized by profound bilateral sensorineural hearing loss and congenital long QT syndrome, which predisposes individuals to syncope and sudden cardiac death. -This syndrome occurs secondary to mutations in genes (e.g., KCNQ1, KCNE1) that encodes the alpha and beta subunits of voltage-gated potassium channels. -These subunits contribute to the slow-acting component of the outward-rectifying potassium current, which is responsible for ventricular repolarization during phase 3 of the cardiac action potential. Mutations in the potassium channel lead to a decrease in potassium current with prolongation of action potential duration and the QT interval. QT interval prolongation predisposes to the development of life-threatening ventricular arrhythmias, such as torsades de pointes and ventricular fibrillation Choices A and D= Brugada syndrome is an autosomal dominant condition that can be associated with mutations in cardiac sodium or L-type calcium channels, leading to characteristic ECG changes (e.g., pseudo right bundle branch block, ST-segment elevation in leads V1-V3) and an increased risk of ventricular tachyarrhythmias and sudden cardiac death Choice B= Duchenne muscular dystrophy is an X-linked disorder caused by mutations in dystrophin, a structural membrane protein that stabilizes the plasma membrane of myocytes. Patients typically present with progressive proximal muscle degeneration and weakness. Additional features may include cardiomyopathy and conduction abnormalities Choice E= Genetic defects in the sodium-potassium ATPase pump typically are not associated with long QT syndrome. Cardiac glycosides (e.g., digoxin) inhibit the sodium-potassium ATPase in myocardial cells, resulting in a mild increase in intracellular sodium and subsequent inhibition of the sodium-calcium exchanger. This leads to increased intracellular calcium and cardiac contractility, which can help reduce symptoms in patients with advanced systolic heart failure

Question 1 of 2 -Age: 63 years man -Brought to ED due to an episode of syncope. The patient was recently discharged back to a nursing home after being hospitalized for a urinary tract infection. -History: coronary artery disease and takes aspirin, clopidogrel, and atorvastatin -Medical conditions: type 2 diabetes, hypertension; doesn't use tobacco -Low BP (68/32) and pulse (118). Invasive hemodynamic monitoring reveals an elevated right atrial pressure of 17mmHg and a pulmonary capillary wedge pressure of 7mmHg (normal 8-10mmHg); most likely cause? a)Anaphylactic shock b)Gastrointestinal bleeding c)Right ventricular infarction d)Septic shock e)Severe aortic stenosis

Answer: Choice C (Right ventricular infarction) -This patient is in shock (e.g., severe hypotension, end organ dysfunction (syncope)) due to an unknown cause. Hemodynamic monitoring (e.g., pulmonary artery catheterization) shows decreased pulmonary capillary wedge pressure (PCWP) and increased right atrial pressure (RAP), which indicated impaired blood flow from the right ventricle to the left side of the heart (right-sided heart failure). Common causes of right-sided HF= obstruction of pulmonary circulation (e.g., pulmonary hypertension, pulmonary embolism) and right ventricular infarction (likely in this patient due to multiple cardiovascular risk factors) -In right ventricular infarction, the right ventricle is unable to effectively pump blood forward into the pulmonary arteries, causing increased pressure in the right atrium (i.e., elevated central venous pressure). The decreased RV outflow also lowers PCWP (an indicator of left-sided preload), which subsequently reduces cardiac output and causes hypotension. A reflexive increase in heart rate is mostly intact in this patient, though it is often impaired in RV infarction due to frequent ischemic involvement of the conduction system Choices A and D= Anaphylactic and septic shock are associated with peripheral vasodilation and massive fluid shift to the extravascular space. The depleted intravascular volume lowers the mean circulatory filling pressure, reducing both RAP and PCWP, while hypotension caused by widespread peripheral vasodilation leads to a compensatory increase in cardiac output Choice B= GI bleeding can cause hypovolemic shock with decreased RAP, PCWP, and cardiac output. Decreased cardiac output triggers an increase in sympathetic tone with peripheral vasoconstriction (increased systemic vascular resistance) to help maintain perfusion pressure Choice E= Aortic stenosis causes left ventricular outflow obstruction and would increase pressure in the left atrium (increased PCWP) and pulmonary circulation. In severe cases, right-sided heart failure can also develop, leading to increased RAP. Systemic vascular resistance is increased with aortic stenosis in response to a reduction in cardiac output

-Brought to the ED due to severe substernal chest pain that is accompanied by sweating and mild dyspnea. The pain started as midline chest discomfort several hours ago but then progressively worsened such that the patient could not fall asleep -He received sublingual nitroglycerin in the ambulance without significant pain relief. His medical history includes hypertension, type 2 diabetes, hyperlipidemia -ECG= normal sinus rhythm, ST-segment elevation in leads I, aVL, and V1-V4, with deep Q wave development over the next several hours. Cardiac catheterization in this patient would most likely show which of the following? a)Atherosclerotic plaque obstructing 50% of the coronary artery lumen, no thrombus b)Atherosclerotic plaque obstructing 80% of the coronary artery lumen, no thrombus c)Ruptured atherosclerotic plaque with fully obstructive thrombus d)Significant coronary artery vasospasm causing flow obstruction e)Ulcerated atherosclerotic plaque with partially obstructive thrombus

Answer: Choice C (Ruptured atherosclerotic plaque with fully obstructive thrombus) -This patient with persistent substernal chest pain and ECG demonstrating ST elevation in the anterolateral leads has an ST-elevation MI (STEMI). In the evolution of STEMI on ECG, peaked T waves in the affected leads are typically the first manifestation, followed by the appearance of ST elevation within several minutes, then Q waves within hours to days. Patients with STEMI typically have sudden-onset substernal chest pain that may be accompanied by nausea, diaphoresis, or dyspnea; the pain does not improve with rest or the administration of short-acting nitrates (e.g., nitroglycerin) -MI is usually caused by acute atherosclerotic plaque rupture with release of lipids, collagen, and tissue factor into the blood stream; because these contents are highly thrombogenic, there is rapid formation of overlying thrombus. In STEMI, the overlying thrombus is extensive enough to fully occlude the coronary artery lumen, completely blocking blood flow to a portion of the heart and leading to transmural (full-thickness) infarction of the myocardial wall. Partially obstructive thrombus (Choice E) is likely to produce NSTEMI or unstable angina. NSTEMI is associated with impaired blood flow to only the subendocardial region; the symptomatic presentation is similar to STEMI, but ECG reveals ST segment and T wave abnormalities with no ST-elevation Choice A= Atherosclerotic plaque obstructing 50% of the coronary artery lumen is likely to be asymptomatic. Symptoms of stable angina do not typically become prominent until >70% of the cross-sectional area of the coronary artery lumen is occluded. Choice B= Atherosclerotic plaque obstructing 80% of the coronary artery lumen is likely to cause stable angina, characterized by substernal or left-sided chest pressure, tightness, or pain that is precipitated by exertion and relieved by rest or short-acting nitrates Choice D= Coronary artery vasospasm (vasospastic or Prinzmetal angina) results from hyperactivity of intimal smooth muscle, usually in the absence of significant atherosclerotic coronary artery disease. Transient anginal chest pain with corresponding ST elevation on ECG can occur; however, the chest pain should rapidly resolve with nitroglycerin

-A group of forensic pathologists are analyzing tissue samples of adolescents age 13-18 to study the aging process. -Autopsy of a 14-year-old boy who died in a motor vehicle accident shows several minimally raised yellow spots on the inner surface of the abdominal aorta. The rest of the cardiovascular findings during the autopsy are unremarkable -He had no known medical problems. There was no family history of cardiovascular disease or sudden cardiac death. -Which of the following is most likely to be the predominant cell type in these lesions on light microscopy? a)Eosinophils b)Fibroblasts c)Macrophages d)Mast cells e)Neutrophils

Answer: Choice C (macrophages) The lesions described during autopsy on the inner surface of the abdominal aorta are consistent with fatty streaks. These are the earliest lesions in the progression to atherosclerosis and can be seen as early as the second decade of life. Atherosclerosis is a pathophysiologic process that begins in childhood with the development of focal intimal thickening and lipid accumulation. It progresses in the following sequence: -Endothelial injury/dysfunction causing increased vascular permeability and enhanced leukocyte adhesion -Accumulation of lipoproteins (LDL) within the vessel wall -Monocyte adhesion to the endothelium, followed by migration and transformation into lipid-laden macrophages (foam cells) -Release of growth factors by adherent platelets, activated macrophages and endothelial cells -Recruitment of T cells and smooth muscle cell proliferation, which lead to extracellular matrix production With advancing age, the chronic inflammatory process initiated by endothelial injury transitions from fatty streaks within the intima (composed mainly of lipid-laden foam cells) into atherosclerotic plaques, such as fibrous cap atheromas and fibrous plaques

-Age: 22 years woman -Comes to the office due to worsening dyspnea. This patient has also experienced low-grade fevers, a 7kg weight loss, syncopal episodes over the last 3 months -Her shortness of breath worsens when sitting and improves when lying down; Medical history is insignificant. The patient doesn't smoke -Temp. (37.4C), BP (115/62), pulse (75, regular), RR (12). Lungs are clear. Cardiac auscultation= low-pitched, mid-diastolic rumble at the cardiac apex. The remainder of physical exam is normal. ECG= left atrial enlargement, echocardiography= large, pedunculated mass in the left atrium. Histology will show? a)Coarse, filamented, branching septate hyphae b)Gram-positive cocci in clusters c)Malignant myeloid cells d)Scattered cells within a mucopolysaccharide stroma e)Solid mass of platelets and fibrin

Answer: Choice D (Scattered cells within a mucopolysaccharide stroma) -This patient with a large, pedunculated mass in the LA most likely has an atrial myxoma. Myxomas are the most common primary cardiac neoplasm, and approximately 80% arise within the LA -Left atrial myxomas frequently obstruct blood flow from the LA to the LV, leading to a murmur mimicking that of mitral stenosis (i.e., mid-diastolic rumble at the apex). The obstruction can also lead to a decrease in cardiac output that manifests as dyspnea, lightheadedness, or syncope. Because the mass is typically, mobile, obstructive symptoms may be transient and influenced by position (i.e., upright posture exacerbates mitral obstruction, whereas lying down alleviates it). In some patients, fragments of the mass may embolize into the systemic circulation (e.g., resulting in stroke or acute limb ischemia). In addition, some myxomas can produce cytokines (e.g., IL-6) that lead to constitutional symptoms including fever and weight loss -Histologically, these tumors demonstrate scattered myxoma cells within a mucopolysaccharide stroma and blood vessels, which may be encircled by myxoma cells. Myxomas produced a large amount of VEGF, which contributes to the angiogenesis, hemorrhaging (seen as brown hemosiderin deposits), and friability characterizing these tumors Choices A and B= Coarse, filamented, branching septate hyphae are found in fungal endocarditis, whereas gram positive cocci in clusters are consistent with bacterial endocarditis. Both are unlikely in this young woman with no history of valvular disease or IV use. In addition, endocarditis typically causes a regurgitant murmur, with valvular vegetations seen on echocardiography. Choice C= Acute myeloid leukemia (AML) rarely may involve the heart. Signs and symptoms related to anemia (e.g., weakness, fatigue), neutropenia (e.g., infections), and thrombocytopenia (e.g., ecchymoses, epistaxis) are typically present with AML Choice E= Nonbacterial thrombotic endocarditis involves the valvular deposition of sterile platelets and fibrin and most commonly occurs in the setting of advanced malignancy or SLE. The disease usually presents with systemic embolization, and heart murmurs are often absent; if a murmur is present, it is typically regurgitant rather than stenotic

-Age: 42 years man -Comes to the ED after a syncopal episode preceded by palpitations and lightheadedness. The patient has no significant medial history and takes no medications (doesn't smoke) -The patient is a computer programmer and spends most of his time indoors. Vital signs are normal with no orthostatic changes. Heart and lung sounds are normal -The liver span is increased with no tenderness. The skin appears darkly tanned. ECG demonstrates sinus node dysfunction, and echocardiography reveals abnormal diastolic relaxation of the left ventricle. Which of the following is the most likely cause of this patient's condition? a)Amyloid deposition b)Cortisol deficiency c)Granuloma formation d)Human leukocyte antigen gene mutation e)Iron accumulation f)Lysosomal glycogen accumulation

Answer: Choice E (Iron accumulation) -This patient has syncope associated with diastolic dysfunction, hepatomegaly and skin hyperpigmentation. This presentation is suggestive of hereditary hemochromatosis (HH), an autosomal recessive disorder characterized by excessive intestinal iron absorption with accumulation of the excess iron in parenchymal tissues -Cardiac manifestations of HH include heart failure and conduction system disease. In early stages, myocardial iron overload is expressed as diastolic LV dysfunction with restrictive filling pattern. As the disease advances, progressive ventricular modeling leads to dilated cardiomyopathy and reduced LV ejection fraction. Iron deposition can also affect the cardiac conduction system. Sinus node dysfunction (i.e., sick sinus syndrome) can lead to presyncope or syncope; malignant arrhythmias with sudden cardiac death occasionally occur -On light microscopy, cardiac hemochromatosis is visible as brown, granular deposits in myocytes that stain strongly with Prussian blue. Cardiomyopathy can be the first presenting manifestation of HH, although careful assessment can usually find evidence of iron deposition elsewhere (e.g., elevated hepatic transaminases, hepatomegaly). Treatment includes therapeutic phlebotomy to remove excess iron stores, which can lead to regression of cardiomyopathy in some patients. Choice A= Amyloidosis is caused by extracellular deposition of misfolded amyloid proteins. Multiorgan involvement (e.g., cardiomyopathy, hepatomegaly) is common, but amyloidosis frequently involves the kidney (e.g., nephrotic syndrome), nerves (e.g., neuropathy) and muscles (e.g., macroglossia). It is not associated with hyperpigmentation Choice B= Primary adrenal insufficiency (i.e., cortisol deficiency) can cause hyperpigmentation due to cosecretion of melanocyte-stimulating hormone with ACTH (both derived from POMC). However, hepatomegaly is not seen, and although hypotension is common, sinus node and diastolic dysfunction are not Choice C= Sarcoidosis results from widespread formation of noncaseating granulomas. Cardiac infiltration can cause conduction blockade, and liver involvement may result in hepatomegaly. However, lung symptoms (e.g., dyspnea, cough) are more common, and skin manifestations include papules and nodules rather than hyperpigmentation Choice D= HH is associated with mutations in the HFE gene, the HFE protein binds to the transferrin receptor and regulates intestinal and hepatic iron uptake. HLA mutations are associated with autoimmune disorders (e.g., celiac disease, type 1 diabetes) Choice F= Glycogen storage disease type II (i.e., Pompe disease) is caused by a deficiency in alpha-glucosidase (acid maltase). It manifests shortly after birth with hypertrophic cardiomyopathy, hepatomegaly, muscle weakness and failure to thrive. Skin hyperpigmentation is not typical

-Age: 68 years man -Brought to ED with chest pain. For the last year, the patient has had exertional chest pain that has progressively worsened. He was shoveling snow this morning when the pain become unbearable. He has smoked a pack of cigarettes daily for 40 years -High BP; normal pulse; exam: mild diaphoresis -Lungs: clear to auscultation. ECG= ST and T wave changes suggestive of ischemia. Cardiac enzymes are elevated. Emergent coronary angiography is performed, which demonstrates significant atherosclerotic involvement of the left anterior descending and circumflex arteries. Development of these vascular lesions most likely involved growth factors released from which of the following sources? a)B lymphocytes b)Eosinophils c)Erythrocytes d)Mast cells e)Neutrophils f)Platelets

Answer: Choice F (Platelets) The development of atherosclerotic plaque, or atheroma, involves a multistep process: -Initially, factors including hyperlipidemia, hypertension, hyperglycemia, and smoking triggers endothelial injury and/or dysfunction. This leads to increased vascular permeability, enhanced leukocyte adhesion, and a higher propensity for thrombosis. -Lipoproteins (i.e., LDL and oxidized LDL) enter the arterial wall intima and begin to accumulate. Monocytes adhere to the endothelial wall and enter into the intima as well; these cells transform into macrophages and engulf lipid particles to become foam cells. Platelets adhere to the abnormal endothelium and become activated -Growth factors, namely platelet-derived growth factors (PDGF), are released from platelets, activated macrophages, and endothelial cells. This triggers smooth muscle (SMC) recruitment from the media and proliferation in the intima Over time, progressive SMC proliferation and accumulation of necrotic debris (due to macrophage/foam cell and SMC death) lead to growth of the atheroma. SMCs encourage plaque stability by synthesizing collagen, whereas activated inflammatory cells break down collagen and contribute to plaque instability. HDL likely extracts lipids from the intima back into the bloodstream and helps slow atheroma development Choice A= B lymphocytes are not known to play a significant role in atheroma development. However, T lymphocytes in the arterial intima likely play a role in activating macrophages, endothelial cells, and SMCs by releasing inflammatory cytokines (e.g., IFN-gamma) Choices B and D= Eosinophils and mast cells are important in IgE mediated (type I) hypersensitivity reactions; mast cells release histamine in the early phase, and eosinophils release leukotrienes and major basic protein in the late phase of the reaction. However, these cells are not significantly involved in atheroma development Choices C and E= Neither erythrocytes nor neutrophils release growth factors or play a significant role in atheroma development

-Age: 24 years man -Evaluated for episode of syncope. He was out jogging when he felt lightheaded and passed out, but he didn't sustain any head injury -The patient has had 2 similar episodes of lightheadedness while jogging over the last year, but this was the first time he passed out. He considers himself in good health and has no other medical problems -The patient does not smoke or do drugs. His father suddenly at age 30. On physical exam, he has a harsh systolic murmur. Transthoracic echocardiography shows asymmetric interventricular septal hypertrophy. The patient's symptoms are most likely explained by left ventricular outflow obstruction created by which of the following structures? a)Aortic valve cusp and interventricular septum b)Aortic valve cusp and papillary muscle c)Aortic valve cusps d)Ascending aorta and interventricular septum e)Mitral valve leaflet and interventricular septum

Answer: Choice E (Mitral valve leaflet and interventricular septum) -This patient's presentation suggests hypertrophic cardiomyopathy (HCM), an autosomal dominant disorder resulting from mutations in cardiac sarcomere proteins. HCM is characterized by asymmetric ventricular septal hypertrophy and variable, dynamic left ventricular outflow (LVOT) obstruction. Systolic anterior motion of the mitral valve toward the interventricular septum can cause eccentric mitral regurgitation and exacerbate LVOT obstruction -Many patients are asymptomatic and may be diagnosed with an abnormal ECG or murmur during routine evaluation. They can also present with exertional dyspnea, chest pain, fatigue, palpitations, dizziness, syncope or sudden cardiac death (due to ventricular arrhythmias). Examination often reveals a harsh crescendo-decrescendo systolic murmur at the apex and left lower sternal border, which changes in intensity with physiologic maneuvers Answer: Choice E (Mitral valve leaflet and interventricular septum) -This patient's presentation suggests hypertrophic cardiomyopathy (HCM), an autosomal dominant disorder resulting from mutations in cardiac sarcomere proteins. HCM is characterized by asymmetric ventricular septal hypertrophy and variable, dynamic left ventricular outflow (LVOT) obstruction. Systolic anterior motion of the mitral valve toward the interventricular septum can cause eccentric mitral regurgitation and exacerbate LVOT obstruction -Many patients are asymptomatic and may be diagnosed with an abnormal ECG or murmur during routine evaluation. They can also present with exertional dyspnea, chest pain, fatigue, palpitations, dizziness, syncope or sudden cardiac death (due to ventricular arrhythmias). Examination often reveals a harsh crescendo-decrescendo systolic murmur at the apex and left lower sternal border, which changes in intensity with physiologic maneuvers

-Age: 69 years man -Comes to the clinic due to a squeezing pressure in his midline chest and neck that occurs when walking. History: hypertension and type 2 diabetes. -Ex-smoker (20-pack-ear history). Coronary angiography shows extensive atherosclerosis and near-total occlusion of the left anterior descending artery. Further testing= normal resting LV ejection fraction with no regional wall motion abnormalities -The absence of myocardial necrosis and scarring despite vessel occlusion in this patient can be best explained by which of the following features of the occluding plaque? a)Active peripheral inflammation b)Lipid-rich core c)Low calcium content d)Ostial location e)Slow growth rate f)Thin fibrous cap

Answer: Choice E (Slow growth rate) -In response to progressive reductions in blood flow caused by atherosclerotic coronary artery disease, ischemic myocardium may release growth factors (e.g., VEGF) to stimulated formation and maturation of collateral vessels. In this patient, the slow development of stable atherosclerotic plaque in the left anterior descending artery (LAD) likely allowed collateral vessels from the right coronary artery to supply the ischemic myocardium located distal to the LAD occlusion. Because there is low hydrostatic pressure distal to the LAD occlusion, a pressure gradient is created that facilitates blood flow through the small collateral vessels. -An unstable atherosclerotic plaque is less likely to allow the development of viable collateral vessels, as it is more likely to rupture before it becomes large enough to cause significant chronic ischemia, rupture leads to overlying thrombus and abrupt arterial occlusion with acute myocardial ischemia/infarction and cellular necrosis. The rapid development of ischemia allows insufficient time for collateral vessels to develop and mature Choices A, B, and F= Atherosclerotic plaques with active inflammation (containing activated macrophages that secrete metalloproteinases and degrade collagen), high core concentration of lipid, and/or thin fibrous cap are unstable and more likely to rupture. All of these properties are less likely in an atherosclerotic lesion that has grown to create near-total occlusion without rupturing Choice C= A patient's total coronary artery calcium content correlates with the total atherosclerotic plaque burden, and calcium scoring by cardiac CT scan is sometimes used to estimate the severity of coronary artery disease. However, low calcium content (or immature calcification) with an individual plaque may decrease stability of that plaque and increase the likelihood of rupture Choice D= Ostial atherosclerotic plaques (those located at a branch point of 2 arteries) are more likely to occlude blood flow to multiple areas of myocardium, thereby limiting the availability of nearby well-perfused arteries from which viable collateral vessels can develop

Question 2 of 2 -IV fluids and vasopressors are administered, but the patient remains hypotensive and dies 3 hours later. Autopsy examination is performed and reveals bilateral hippocampal necrosis. Which of the following areas of the colon would be most likely to also demonstrate necrosis? a)Ascending colon b)Cecum c)Hepatic flexure d)Ileocolic junction e)Splenic flexure

Answer: Choice E (Splenic flexure) -This patient's bilateral hippocampal necrosis is characteristic of the effect of global cerebral ischemia due to systemic hypotension. The hippocampus is particularly vulnerable during shock because of the high metabolic demand of its CA1 pyramidal neurons. Watershed areas located between the distal terminal branches of 2 different vascular territories (border zones) are also susceptible to ischemia because they have a low baseline perfusion pressure and poorly tolerate sustained decreases in systemic BP Important watershed areas in the body include: -Gastrointestinal - Splenic flexure: the border zone supplied by branches of the superior and inferior mesenteric arteries -Brain: the borders of the areas supplied by the anterior cerebral artery, middle cerebral artery and posterior cerebral artery

-Age: 10 years boy -Brought to the physician by parents due to restlessness and involuntary jerking. He takes no medications and vaccinations are up to date -His parents do not recall any recent injuries or illnesses other than a sore throat 3 months ago. On examination, the patient has rapid, irregular jerking movements involving his face, arms, and legs. This patient is at greatest risk for developing which of the following? a)Deforming polyarthritis b)Early dementia c)Parkinson disease d)Renal failure e)Valvular heart disease

Answer: Choice E (Valvular heart disease) -This patient's restlessness and purposeless jerking movements 3 months after having a sore throat are consistent with Sydenham chorea, a hyperkinetic extrapyramidal movement disorder that is the most common acquired chorea of childhood. Sydenham chorea is a neurologic manifestation of acute rheumatic fever that occurs 1-8 months after group A beta-hemolytic streptococcal infection -It is caused by a delayed onset autoimmune reaction involving anti-streptococcal antibodies that cross-react with the basal ganglia. Because Sydenham chorea is a manifestation of acute rheumatic fever, patients with this disorder at risk for chronic rheumatic heart disease Choice A= Although migratory polyarthritis is an early manifestation of acute rheumatic fever, it is generally transient, and rarely, if ever, deforming Choice B= Early-onset Alzheimer dementia is associated with down syndrome (trisomy 21). Increased expression of the amyloid precursor protein gene located on chromosome 21 is thought to be the cause. Choice C= Parkinson disease is an extrapyramidal hypokinetic movement disorder consisting of tremor, rigidity, akinesia, and postural instability. Jerking extremity movements are generally not seen in Parkinson disease, except in cases of levodopa overdose Choice D= Poststreptococcal GN occurs following infection with specific nephritogenic strains of group A Streptococcus. However, the nephritis manifests within 1-6 weeks and is typically self-limited. Renal failure is not part of the clinical syndrome associated with acute rheumatic fever

Question 1 of 2 -Age: 53 years man -Comes to the ED due to fever and progressive weakness over the last 2 weeks. Yesterday, he also developed shortness of breath. The patient emigrated from Eastern Europe 2 years ago and says he was diagnosed with "heart disease" in the distant past, but does not recall details -He doesn't smoke. Despite receiving appropriate medical care, the patient expires during hospitalization. On autopsy, gross examination of his heart shows large, friable masses on the mitral valve with extensive destruction of the cuspal tissue -Which of the following is the most likely predisposing factor for this patient's presenting condition? a)Myocardial hypertrophy b)Myocardial thinning and fibrosi c)Pericardial effusion d)Rupture of chordae tendineae e)Valvular inflammation and scarring

Answer: Choice E (Valvular inflammation and scarring) -This patient's presentation- fever, progressive weakness, dyspnea (due to valvular regurgitation)- along with the presence of vegetations on the mitral valve is consistent with infective endocarditis (IE). Predisposing factors for IE include valvular abnormalities (e.g., rheumatic heart disease, mitral valve prolapse, prosthetic valves, congenital heart disease) and conditions that promote bacteremia/fungemia (e.g., IV drug use, dental procedures) -This patient likely had a remote history of rheumatic heart disease (immigrant from developing region, history of heart disease) with underlying degeneration of the mitral valve due to chronic valvular inflammation and scarring Choice A= Myocardial hypertrophy is seen in patients with uncontrolled hypertension, aortic stenosis, and hypertrophic cardiomyopathy and, in general, does not predispose to IE Choice B= Myocardial fibrosis is associated with infiltrative cardiomyopathy (sarcoidosis, amyloidosis), Chagas disease, myocarditis, and prior MI. It does not lead to increased risk of IE Choice C= Pericardial effusion occurs as a result of inflammation of the pericardium (acute or chronic pericarditis) and has no association with the risk of IE Choice D= Rupture of chordae tendineae is a complication of endocarditis or MI, it is not a risk factor for development of IE


Ensembles d'études connexes

Grammar test. Present and Past tenses.

View Set

Cardiovascular med practice questions - ATI

View Set

Econ--Supply and Demand and Price Elasticity

View Set

CAPM Leftover Definitions (some are in other areas, use only for final study)

View Set